Sunteți pe pagina 1din 300

Edicin y correccin: Dania Relova Fernndez

Diseo de interior y cubierta: Frank Herrera


Diagramacin: Israel de Jess Zaldvar Pedroso
Carlos L. Menndez Gutirrez y Jess Garca, 2007

Versin impresa
ISBN 978-959-16-07-0339-3
Flix Varela

Versin electrnica
ISBN 978-959-16-0619-8
Editorial Universitaria

Procesos para el tratamiento de aguas residuales / Carlos Menendez Gutierrez y Jess M. Prez Olmo

PRESENTACIN

En 1984, los autores, en colaboracin con el Dr. Jan Chudoba, del Instituto de
Tecnologa Qumica de Praga, publicaron el libro Fundamentos Tericos de
Algunos Procesos para la Purificacin de Aguas Residuales, con el propsito de que sirviera de apoyo a aquellos profesionales de habla hispana que
realizaban su trabajo de doctorado en la Repblica de Checoslovaquia. Dos
aos despus se public la segunda edicin de ese ttulo.
Para continuar supliendo las crecientes necesidades de los profesionales de
este campo de trabajo, los autores publicaron en Cuba en 1991, bajo el ttulo de
Procesos para el Tratamiento Biolgico de Aguas Residuales Industriales
un texto enfocado fundamentalmente, tal como indica su ttulo, en los procesos
biolgicos para el tratamiento de aguas residuales. Doce aos despus se presenta esta segunda edicin con el mismo propsito.
Como generalmente ocurre, una vez que concluy la preparacin de esta
nueva edicin, es que los autores se consideraron en condiciones de iniciar el
trabajo. Esto es consecuencia de la inconformidad natural que se experimenta,
a partir de la conviccin de que toda obra humana es perfectible. Tambin esto
forma parte de las enseanzas que nos leg Jan Chudoba.
La justificacin que nos anima ahora, como en las ocasiones anteriores,
estimulados por los resultados ya logrados, es continuar contribuyendo a la superacin de los profesionales dedicados a esta esfera de trabajo, con temas
actualizados y expuestos de manera ms completa que en la primera edicin.
Otra justificacin quizs sea el de siempre haber pensado que el camino por
recorrer era ms largo del que en realidad ha sido y que sentimos la necesidad
de expresarnos. Ambas justificaciones son igualmente vlidas.
Agradecemos a todos los que han colaborado en este nuevo empeo, en
especial a la profesora Julia Mara Hernndez por su apoyo en la preparacin de
los materiales, y a nuestras familias por la paciencia manifestada una vez ms.
Deseamos dedicar esta obra, que es a su vez un nuevo punto de partida a la
memoria de Jan Chudoba, que sigue estando entre nosotros.
a JAN
III

Ciudad de La Habana : Editorial Flix Varera - Editorial Universitaria, 2007. ISBN 978-959-16-0619-8

Procesos para el tratamiento de aguas residuales / Carlos Menendez Gutierrez y Jess M. Prez Olmo

LOS AUTORES

Carlos L. Menndez Gutirrez


Graduado de Ingeniero Qumico en la Universidad de La Habana
y de Especialista en Saneamiento Ambiental en el Centro Nacional de Investigaciones Cientficas de Cuba. Obtuvo el ttulo de
Doctor en Ciencias Tcnicas en el Instituto de Tecnologa Qumica de Praga. Ha impartido cursos en el tema de tratamiento de
aguas residuales en universidades cubanas y latinoamericanas.
Miembro del Comit Doctoral del Instituto Tecnolgico de Toluca, Mxico, y del Tribunal de Grados Cientficos para Ingeniera
Qumica de Cuba. Miembro de varias sociedades cientficas. Es
autor de artculos tcnicos.
Jess M. Prez Olmo
Graduado de Ingeniero Qumico en la Universidad de La Habana
y Mster en Ingeniera Saneamiento Ambiental. Ha impartido
cursos sobre tratamiento de aguas residuales en universidades
cubanas. Es Especialista Principal en el tratamiento de aguas
residuales de la Compaa Especializada en Soluciones Integrales Geogrficas y Medio Ambientales (CESIGMA S.A.) y ha
brindado asesora en plantas cubanas y de otros pases latinoamericanos.

Edicin auspiciada por:


Environmental Capacity Enhancement Project in Cuba.
Proyecto internacional establecido entre la University of Guelph y el
Instituto Superior Politcnico Jos Antonio Echeverra
Financiado por:
Canadian International Development Agency (CIDA)
IV

Ciudad de La Habana : Editorial Flix Varera - Editorial Universitaria, 2007. ISBN 978-959-16-0619-8

Procesos para el tratamiento de aguas residuales / Carlos Menendez Gutierrez y Jess M. Prez Olmo

NDICE

CAPTULO 1. Indicadores de la contaminacin / 1


1.1. Introduccin / 1
1.2. Contenido de slidos / 1
1.3. Demanda bioqumica de oxgeno (DBO) / 3
1.4. Demanda terica de oxgeno (DTO) y demanda qumica
de oxgeno (DQO) / 11
1.5. Carbono orgnico total (COT) / 13
1.6. Nitrgeno / 17
1.7. Fsforo / 19
1.8. Caractersticas de las aguas residuales / 20
1.9. Mediciones del caudal / 29
1.10. Disminucin del volumen y fortaleza de los residuales / 35
CAPTULO 2. Pretratamiento de aguas residuales / 39
2.1. Compensacin / 39
2.2. Sedimentacin / 53
CAPTULO 3. Oxidacin biolgica nitrificacin / 77
3.1. Principios de la oxidacin biolgica / 77
3.2. Metabolismo / 78
3.3. Biodegradabilidad de las aguas residuales / 82
3.4. Cintica del crecimiento biolgico y remocin de sustrato / 87
3.5. Nitrificacin-desnitrificacin / 98
CAPTULO 4. Lodo activado / 119
4.1. Generalidades / 119
4.2. Parmetros bsicos / 123
4.3. Principales modificaciones tecnolgicas / 128
V

Ciudad de La Habana : Editorial Flix Varera - Editorial Universitaria, 2007. ISBN 978-959-16-0619-8

Procesos para el tratamiento de aguas residuales / Carlos Menendez Gutierrez y Jess M. Prez Olmo

4.4.
4.5.
4.6.
4.7.
4.8.
4.9.

Aplicacin de los modelos cinticos / 135


Factores que inciden en la eficiencia de purificacin / 138
Presencia de compuestos orgnicos en el efluente / 144
Produccin de biomasa y lodo / 146
Determinacin de constantes a escala de laboratorio / 157
Requerimientos nutricionales / 160

CAPTULO 5. Transferencia de oxgeno / 165


5.1. Introduccin / 165
5.2. Consideraciones fundamentales de la transferencia
de oxgeno / 165
5.3. Requerimiento de oxgeno / 173
5.4. Implementos de aereacin / 180
5.5. Consumo de potencia / 192
CAPTULO 6. Filtros percoladores / 199
6.1. Introduccin / 199
6.2. Partes de las que consta un filtro percolador. Mecanismo
de remocin de la DBO / 201
6.3. Parmetros bsicos del proceso. Definiciones / 206
6.4. Clasificacin de los filtros percoladores / 207
6.5. Recirculacin / 208
6.6. Factores que inciden en la eficiencia de purificacin / 209
6.7. Diseo de filtros percoladores / 214
6.8. Clculo de las constantes del modelo de primer orden / 224
CAPTULO 7.
7.1.
7.2.
7.3.
7.4.
7.5.
7.6.
7.7.
7.8.

Lagunas de estabilizacin / 235


Caractersticas generales / 235
Lagunas aerobias / 235
Lagunas anaerobias / 236
Lagunas facultativas / 236
Rgimen de flujo en las lagunas / 240
Diseo de lagunas de estabilizacin / 241
Relacin rea: volumen para las lagunas / 247
Remocin de patgenos / 248
VI

Ciudad de La Habana : Editorial Flix Varera - Editorial Universitaria, 2007. ISBN 978-959-16-0619-8

Procesos para el tratamiento de aguas residuales / Carlos Menendez Gutierrez y Jess M. Prez Olmo

7.9. Balance hdrico de las lagunas / 248


7.10. Criterios para la operacin y mantenimiento de las lagunas
de estabilizacin / 251
CAPTULO 8.
8.1.
8.2.
8.3.
8.4.
8.5.
8.6.
8.7.

Digestin anaerobia / 255


Introduccin / 255
Mecanismo de la digestin anaerobia / 256
Distintas variantes de los procesos de digestin anaerobia / 257
Factores que controlan el proceso de digestin / 265
Parmetros indicadores del curso de la digestin / 269
Mtodos de diseo / 272
Puesta en marcha de los digestores anaerobios / 282

CAPTULO 9.
9.1.
9.2.
9.3.
9.4.
9.5.
9.6.

Manejo de lodos / 285


Introduccin / 285
Contenido de humedad y volumen de los lodos / 285
Lodos primarios y secundarios. Criterios de estimacin / 287
Espesamiento por gravedad / 290
Lechos de secado / 291
Volumen de agua producida / 293

ANEXO 1. Volmenes mnimos de muestras y criterios


para su conservacin / 295
ANEXO 2. Distribucin normal acumulativa / 296

VII

Ciudad de La Habana : Editorial Flix Varera - Editorial Universitaria, 2007. ISBN 978-959-16-0619-8

Indicadores de la Contaminacin

CAPTULO 1

INDICADORES DE LA CONTAMINACIN

1.1. INTRODUCCIN
Generalmente el nivel de contaminacin de las aguas residuales no
se mide a partir del conocimiento de la concentracin de los distintos
constituyentes de un agua residual que pueden ser considerados contaminantes, sino determinando parmetros globales como son la demanda
bioqumica de oxgeno (DBO) y la demanda qumica de oxgeno (DQO)
entre otros. En ocasiones, fundamentalmente cuando se trabaja con
residuales industriales, las caractersticas de estos son tales que se
requiere conocer constituyentes especficos como los metales pesados,
tensoactivos, fenoles y otros.
En algunos casos, por ejemplo, para la determinacin de trazas de contaminantes orgnicos en aguas superficiales y para beber, se emplean tcnicas analticas especiales a saber la cromatografa y la espectrometra de
masas.

1.2. CONTENIDO DE SLIDOS


El conocimiento del contenido de slidos de un agua o agua residual aporta
valiosa informacin sobre sus caractersticas, lo cual permite conocer de forma
general su naturaleza y si estas requieren de algn tratamiento en particular que
facilite su remocin o eliminacin.
Cuando se habla de slidos contenidos en un agua residual se est haciendo
referencia a aquello que permanece como residuo despus de la evaporacin y
secado de la muestra a 103 oC.
Los slidos en las aguas residuales pueden estar en forma suspendida, coloidal y disuelta. Todos ellos a su vez son de naturaleza inorgnica u orgnica. La
fraccin orgnica de los slidos se determina mediante la prdida por ignicin a
600 oC.
2

Indicadores de la Contaminacin

1.3. DEMANDA BIOQUMICA DE OXGENO (DBO)


La DBO es uno de los parmetros ms utilizados en la caracterizacin de
los contaminantes orgnicos. Esta determinacin brinda un estimado del oxgeno disuelto requerido por los microorganismos en la degradacin de los compuestos biodegradables.
El mecanismo presente durante la prueba de laboratorio de la DBO es esencialmente biolgico y no depende solo del oxgeno presente y utilizado en un
momento dado, sino tambin de la velocidad a la que este se consume, y por
tanto, del tiempo. As, es necesario fijar un tiempo y temperatura de incubacin
estndar, que normalmente es de cinco das, y 20 qC como temperatura de referencia.
En muchos casos se necesita conocer la demanda bioqumica de oxgeno total de un agua residual (DBO ltima). En tal sentido se requiere determinar previamente la velocidad a la cual ocurre la reaccin en toda su
extensin.
La variacin en el tiempo de la demanda bioqumica de oxgeno de un
agua residual sigue una cintica de primer orden, segn se observa en la figura 1.2.
La determinacin de la DBO implica conocer el oxgeno consumido por
las bacterias en un intervalo de tiempo, ya que existe una relacin cuantitativa entre el oxgeno consumido y la concentracin del material orgnico que
se transforma en dixido de carbono y amonaco. Esta relacin puede ser
representada por la siguiente ecuacin generalizada:

C n H a Ob N c  n 

a
4

3
a 3
 c O 2 o n CO 2   c H 2O  c NH 3
2 4
2 2

(1.1)

La diferencia en el contenido de oxgeno al inicio y al final del anlisis,


por litro de muestra utilizada constituye la DBO de la muestra expresada en
mg de O2 L .
Lgicamente la velocidad y extensin a la cual ocurre esta degradacin
depende de la temperatura y del tiempo, de ah que se ha aceptado para
estos dos parmetros 20 o C y cinco das respectivamente, tal como ya fue
expresado.

Menndez Gutirrez, C. y J. Prez Olmo

Fig. 1.2. Variacin en el tiempo de la DBO: a) remanente, b) satisfecha.

1.3.1. Cintica de la DBO


Los estudios cinticos sobre la DBO han indicado que para propsitos
prcticos esta puede ser considerada, con un buen grado de aproximacin,
como una reaccin de primer orden, lo cual se expresa matemticamente
mediante la ecuacin:

dLt
dt

 k cLt

(1.2)

donde:
Lt: DBO remanente, mg L.
t: tiempo, das.
k: constante de velocidad, da.
Como toda constante de velocidad de reaccin, kc es funcin de la temperatura. Aunque el anlisis tpico o estndar se realiza a 20 oC el valor de kc puede
calcularse a otra temperatura mediante la relacin:
kt

en la que

k 20 T 20

(1.3)

T = 1,056

La integracin de la ecuacin 1.2 conduce a:

Lt

Lo10 k t

Lo e - kc t

(1.4)

Indicadores de la Contaminacin

donde:
Lo: DBO ltima o total de la muestra, mg L.
k: k/2,3
Tal como se aprecia en la ecuacin anterior, el conocimiento de la DBO
remanente de la muestra en un instante dado exige conocer Lo y k. La determinacin de Lt no es lo ms comn en el trabajo rutinario. Generalmente lo que
ms interesa conocer es aquella parte de la demanda bioqumica de oxgeno que
se ha satisfecho en un intervalo de tiempo y a una temperatura dada. As, si Lo
es la demanda total o ltima de un agua residual, y Lt es la demanda remanente
en un instante t dado, la demanda satisfecha o ya ejercida (Y) ser:
Y

Lo  Lt

DBOt

Lo 1  10  k t

(1.5)

Para el caso del anlisis realizado a 200 C y cinco das:

Lo  L5

DBO5 d , 20qC

Lo 1  105 k

(1.6)

Estas expresiones describen los lugares geomtricos representados en la


figura 1.3.

Fig. 1.3. Relacin entre Lo y Lt.


Como ya fue expresado, es la DBO ejercida la que se determina
normalmente en el periodo de incubacin de cinco das y a la temperatura de
referencia de 20 oC. El conocimiento de la DBO total (Lo), se requiere solamente
en casos especficos y conlleva un tratamiento matemtico especial de los
datos experimentales.
Para que la muestra de agua residual sea analizada exitosamente es necesario que contenga una determinada poblacin microbiana o en su defecto esta
5

Menndez Gutirrez, C. y J. Prez Olmo

sea suministrada, a fin de que la materia orgnica pueda ser degradada y tenga
lugar consumo de oxgeno.
Este aspecto reviste una gran importancia porque puede ocurrir que el agua
residual objeto de anlisis contenga bacterias del gnero Nitrosomonas y
Nitrobacter, las cuales no degradan la materia orgnica, sino que oxidan diferentes formas del nitrgeno con el consiguiente consumo de oxgeno (fenmeno
este denominado nitrificacin), y por lo que se falsean los resultados. En las
siguientes ecuaciones se muestra la estequiometra del proceso de nitrificacin.

2NH3  3O2 
o 2NO2   2H2 O  2H
2NO2   O2 
o 2NO3
Siendo la reaccin total:
2NH 3  4O 2 
o 2NO 3   2H   2H 2 O

(1.7)

En aquellos casos en que se analizan efluentes de plantas de tratamiento


que utilizan filtros percoladores o lodos activados estn presentes las bacterias
nitrificantes, por lo que al realizar el anlisis de la DBO se debe aadir alguna
sustancia que las inhiba. El compuesto ms comnmente empleado para inhibir
el crecimiento de organismos nitrificantes es el alilthiourea (ATU).
Cuando estos tipos de bacterias estn presentes, la curva de la DBO puede
mostrar un comportamiento similar al representado en la figura 1.4.

Fig. 1.4. Efecto de la nitrificacin sobre la DBO.


Obsrvese que de estar presente el fenmeno de la nitrificacin la DBO determinada resultar mayor que la que corresponde a la degradacin de la materia
orgnica, lo cual introduce un error por exceso con respecto a lo deseado.
6

Indicadores de la Contaminacin

1.3.2. Influencia de la constante de velocidad


en la determinacin de la DBO
La constante de velocidad desempea un rol importante en la fraccin de la
DBO que puede ser estimada en un periodo de cinco das. Cuanto mayor sea el
valor de esta constante, mayor ser el valor de la DBO determinada.
Esta influencia se puede apreciar en la figura 1.5 donde a manera de ejemplo se han considerado cuatro valores de k.

Fig. 1.5. Influencia de k sobre la DBO.


En la figura 1.5 puede apreciarse cmo para el mismo tiempo, por ejemplo
cinco das, la DBO ser distinta en funcin del valor de la constante de velocidad k.
La variacin que puede experimentar la constante k ha sido motivo de amplias discusiones, pero se pueden considerar como las principales causas:
La naturaleza de la materia orgnica.
La habilidad de los microorganismos para utilizar esa materia orgnica.
As por ejemplo, se puede sealar que en general la parte soluble es degradada ms rpidamente que aquella que se presenta en forma coloidal o en suspensin.
Otro aspecto de importancia en la determinacin de la DBO es la concentracin de microorganismos que aporte el inculo, ya que de ser muy baja, la
DBO ejercida en el tiempo tambin ser baja debido a que las bacterias tardan
un tiempo apreciable en multiplicarse como para ejercer una oxidacin notable.
Este fenmeno se destaca en la figura 1.6. En el caso mostrado por la curva A,
la concentracin de microorganismos es tres rdenes de magnitud mayor que en
B. Esta es la razn por la cual este es un aspecto que debe ser tenido en cuenta
en el momento de realizar el anlisis de la DBO. En el caso de efluentes de
plantas de tratamiento o residual humano este aporte est garantizado de por s,
pudiendo no ocurrir lo mismo cuando la muestra tiene otra procedencia.
7

Menndez Gutirrez, C. y J. Prez Olmo

Fig. 1.6. Efecto de la concentracin del inculo sobre la DBO.


Otro factor no menos importante en la determinacin de la DBO es el grado
de aclimatacin de las bacterias utilizadas en el curso del anlisis para el sustrato
en particular que presenta la muestra. Por supuesto este fenmeno se presenta
cuando la muestra no contiene microorganismos y resulta necesario inocularlos.
En la figura 1.7 se muestra el caso de un inculo no adaptado al sustrato de
inters. De ah que se presente retraso en la DBO ejercida en la etapa inicial.
Por el contrario, en el ejemplo representado en la figura 1.8, se ilustra una situacin en la que el inculo est perfectamente adaptado al sustrato y no se presenta la fase inicial de retraso.

Fig. 1.7. Efecto de la aclimatacin del inculo sobre la DBO.


Se puede apreciar que las curvas que aparecen en las figuras 1.6, 1.7 y 1.8
presentan una tendencia que no se corresponde exactamente con lo que predice
una cintica de primer orden (figura 1.4), ya que las primeras presentan una
meseta ms o menos acentuada y que pudiera ser falsamente interpretado como
8

Indicadores de la Contaminacin

un proceso de nitrificacin. La explicacin de la aparicin de estas mesetas


pudiera ser la siguiente: durante el primer y segundo da de incubacin, la materia orgnica soluble es rpidamente oxidada y la remanente es convertida en
FpOXODV<D HQ HVWD IDVH DSDUHFH XQD PHVHWD FDUDFWHUtVWLFD GH XQD UHGXFLGD
velocidad de oxidacin, lo cual lleva asociada una fase de respiracin endgena.
Posteriormente se aprecia un incremento en la velocidad de oxidacin, que es
atribuible a un incremento en la poblacin de protozoos, los cuales son predadores
de las bacterias. La presencia y duracin de esta meseta entre estas fases
depende del intervalo de tiempo entre el pico de la poblacin bacteriana y el de
la poblacin de protozoos.

Fig. 1.8. Efecto sobre la DBO de un inculo adaptado.


Se ha comprobado que mientras menor sea la constante de autooxidacin
de la biomasa, la meseta ser ms acentuada. Por otra parte, mientras sean los
valores mayores de esta constante la meseta tiende a no aparecer.
Otro aspecto importante relacionado con la DBO est en el hecho de que
por mucho tiempo se pens que despus de un largo periodo de incubacin (por
ejemplo, 20 das) la DBO determinada era la DBO ltima o total de la muestra
(Lo), y que al mismo tiempo era igual a la DQO terica, cuando en realidad
existe discrepancia entre el valor determinado y la demanda terica de oxgeno.
El siguiente ejemplo pondr en evidencia esta discrepancia.
La oxidacin de la glucosa a dixido de carbono y agua requiere de 192 g de
oxgeno por mole de glucosa (180 g/mol) 1,065 mg de oxgeno por miligramo
de glucosa segn la estequiometra de la ecuacin 1.8:
C 6 H12 O 6

 6 02 
o 6 CO 2

 6 H2 O

(1.8)

As, una solucin de glucosa de 300 mg Ltiene una demanda terica de


oxgeno de 320 mg L, sin embargo la determinacin de la DBO con periodos
9

Menndez Gutirrez, C. y J. Prez Olmo

de incubacin de 20 das ha reportado valores de Lo en el intervalo de 250 a


285 mg L. De aqu resulta evidente que no toda la glucosa ha sido oxidada a
dixido de carbono y agua, interpretndose en los siguientes trminos este fenmeno: en el curso de la degradacin de la glucosa las bacterias obtienen energa
para su crecimiento y reproduccin, lo cual implica que parte de la materia
orgnica es convertida en tejido celular, por lo que no experimenta oxidacin
completa, hasta que ocurre la respiracin endgena. Cuando las bacterias mueren este tejido celular se convierte en alimento de otras y prosigue la conversin
de la materia orgnica en dixido de carbono y agua. Una parte de las bacterias
vivas y muertas sirven de alimento a los protozoos y contina la degradacin de
la materia orgnica.

Fig. 1.9. Secuencia de degradacin de los compuestos orgnicos.


En cada una de las etapas por las que transcurre la oxidacin de la materia
orgnica se producen desechos del metabolismo que resultan resistentes a la
degradacin biolgica y que pueden explicar la discrepancia entre la demanda
terica y prctica. En el diagrama de la figura 1.9 se ilustra esta secuencia de
fenmenos.

1.3.3. Clculo de la constante de oxigenacin


A partir de la importancia que existe, a la hora de caracterizar un agua
residual, del conocimiento de la relacin entre la DBO cinco das, DBO ltima y
la constante de velocidad, k, han sido propuestos muchos mtodos para el
10

Indicadores de la Contaminacin

clculo de k.1, 3 A continuacin se presenta uno que, aunque no muy exacto, es


sencillo y permite tener una idea del valor de esa constante antes de pasar a una
determinacin ms exacta, pero tambin ms laboriosa.
El mtodo de Rhame para determinar k, o mtodo de los dos puntos de
Rhame,4 se basa en la relacin entre los valores de la DBO para dos tiempos
dados. El segundo tiempo es el doble del primero. Con el mtodo de Rhame
pueden calcularse la DBO ltima y la constante de de-oxigenacin:
X2
2 X  Z

(1.9)

1
X
log
T t
ZX

(1.10)

donde:
L: DBO ltima.
X: DBO en el tiempo t.
Z: DBO en el tiempo T = 2t.
Segn Rhame, para t = 10 das, los valores de L y k que se obtienen son
comparables a los que se estiman cuando se usan otros mtodos.
En la tabla 1.1 se comparan los valores de la DBO ltima y la constante de
de-oxigenacin para residuales teneros, calculados por diferentes mtodos.
Tabla 1.1. Valores de k y Lo de residuales teneros calculados por diferentes
mtodos

1.4. DEMANDA TERICA DE OXGENO (DTO)


Y DEMANDA QUMICA DE OXGENO (DQO)
Cualquier compuesto orgnico tericamente puede ser oxidado hasta la obtencin final de productos estables como H2O , CO2 , NH3 y SO 24 . La cantidad
11

Menndez Gutirrez, C. y J. Prez Olmo

de oxgeno necesaria para la oxidacin se denomina demanda terica de oxgeno (DTO). La DTO puede ser calculada considerando las relaciones
estequiomtricas:

y z
y
C x H y O z  x   O 2 o x CO 2  H 2O

4 2
2

DTO

4x  y  2z
MM

(1.11)

(1.12)

donde:
MM: Masa molecular del compuesto.
Para compuestos que contienen N en su estructura, la ecuacin para la
demanda terica de oxgeno es:

y3p z
Cx H y Oz N p  x 
 O 2 o x CO 2 

4
2
y 3p
H 2 O  pNH 3
2

DTO

4 x  y  3 p  2 z 8
MM

(1.13)

(1.14)

Si adems el azufre participa en su composicin,

y  3 p  2q
z
 2 q  O2 
C X H y O z N p Sq  x 

4
2
y3p
H 2O  p NH 3  q H 2SO 4
x CO 2 
2

4 x  y  6q  3 p  2z
8
DTO

MM

(1.15)

(1.16)

La demanda qumica de oxgeno brinda una medida ms real de la cantidad


de oxgeno requerida para la oxidacin de los compuestos orgnicos a CO2 y
H2O, de acuerdo con las ecuaciones 1.11, 1.13 y 1.15, cuando se utiliza un
oxidante fuerte.
12

Indicadores de la Contaminacin

El mejor mtodo para determinar la DQO es aquel que arroja valores iguales o cercanos a la DTO. Actualmente este mtodo es el estndar del dicromato. 5
Este mtodo utiliza una solucin de dicromato de potasio cuya concentracin
molar de equivalente es 0,25, en 50 % de cido sulfrico. Bajo estas condiciones, con sulfato de plata como catalizador, la mayora de los compuestos son
oxidados entre 90 y 100 %.6
No obstante, algunos compuestos no son oxidados o lo son solo parcialmente cuando se utiliza el mtodo estndar con dos horas de reflujo. Esto se ha
comprobado en algunas bases fuertes nitrogenadas como las metilaminas,
etilaminas, piridinas y sus derivados (tabla 1.2).
Tabla 1.2. Algunos compuestos parcialmente oxidables por el dicromato
en medio cido

Compuesto

Oxidacin
%
3,7
Metilamina
1,9
Dimetilamina
5,2
Trimetilamina
4,4
Piridina
30,0
metil piridina
27,8
3 metil piridina

Compuesto
Etilamina
Dietilamina
Trietilamina
2,4 dimetilpiridina
2,4,6 trimetilpiridina

Oxidacin
%
36,1
27,8
37,4
58,0
85,2

Otras bases heterocclicas nitrogenadas como la quinolina, pirol, pirolidina,


indol, inidiazol, purina y pirimidina, son oxidadas fcil y completamente. Sin embargo, en los compuestos heterocclicos que contienen dos o ms tomos de
nitrgeno en la molcula, solamente una parte de ellos se desprende como amonaco y el resto como nitrgeno.7
El azufre de los compuestos orgnicos es oxidado a sulfato. La excepcin
est constituida por el grupo CH36TXHHVR[LGDGRDiFLGRPHWLOVXOIyQLFR
Durante la prueba de la DQO se determinan tanto los compuestos
degradables como los no degradables biolgicamente. Tambin se incluyen aquellos compuestos inorgnicos reductores que pueden ser oxidados por el dicromato
en medio cido, como es el caso comn del anin cloruro.

1.5. CARBONO ORGNICO TOTAL (COT)


Hoy da el COT es utilizado como una medida del contenido de sustancias
orgnicas en el agua. Sin embargo, este parmetro no brinda informacin acerca del oxgeno necesario para la oxidacin de las mismas. As, por ejemplo, la
13

Menndez Gutirrez, C. y J. Prez Olmo

glucosa y el cido caproico poseen en su molcula el mismo nmero de tomos


de carbono, no obstante la demanda terica de oxgeno (DTO) de ambos compuestos es diferente:
C6 H12 O 6  6O 2 o 6CO 2  6H 2 O DTO 1,07 gg 1

(1.14)

2,21 gg 1

(1.15)

C6 H12 O 2  8O 2 o 6CO 2  6H 2 O DTO

En el primer caso se requieren 2,66 g de oxgeno para la oxidacin de 1 g de


carbono. En el segundo caso cada gramo de carbono necesita 3,55 g de oxgeno
para la oxidacin total.
En el orden terico existe una relacin entre la DTO y DQO, as como entre
la DTO y el COT.1 Por tanto, existe tambin una relacin entre la DQO y el
COT. En el mismo plano terico cabe suponer que la relacin DTO/COT vara
entre los lmites de cero, cuando los compuestos orgnicos no son oxidables por
el dicromato, hasta 5,33, que es la relacin para el metano segn la ecuacin:
CH 4  2 O 2 o CO 2  2 H 2 O

(1.16)

En la tabla 1.3 se representa esta relacin para algunos compuestos orgnicos comunes.
Tabla 1.3. Relacin DTO/COT de algunos compuestos8
Compuesto
cido actico
Glucosa
Sacarosa
Piridina
Benceno
Etilamina
Metano

Relacin DTO / COT


2,67
2,67
2,67
3,33
3,33
4,00
5,33

El valor lmite de 5,33 para la relacin DTO/COT es tambin vlido para


mezclas de compuestos, aunque en la medida que aumenta en complejidad la
composicin del residual, la DTO va perdiendo su inters prctico y la relacin
DTO/COT llega a ser inoperante.
Para las aguas residuales, constituidas por mezclas de sustancias cuya composicin no siempre se conoce y de concentracin variable, la DTO tiene escaso valor prctico. De ah la importancia de la DQO y la DBO como parmetros
indicadores de la contaminacin en unidades de oxgeno.
14

Indicadores de la Contaminacin

En la prctica la relacin DQO/DTO es muy variable debido a que, como ya


se dijo, hay muchos compuestos orgnicos que no son fcilmente oxidados por
el dicromato. La relacin puede ser tan alta como 1 (para el etanol, cido oxlico
y cido benzoico)9 y tan baja como 0,02 para la piridina.8 Por tanto, las relaciones DTO/COT que aparecen en la tabla 1.3 para compuestos puros, no son
igualmente vlidas para DQO/COT.
Hay una correspondencia entre las relaciones anteriores y el nmero de
oxidacin promedio (NOP) del carbono (figura 1.10).
Este puede calcularse por la ecuacin:
NOP 4  2

nO 2
nC

(1.17)

donde:
nO 2 : Nmero de tomos de oxgeno necesarios para la oxidacin completa

de un compuesto dado en CO2 , H2O, NH3 y H2SO4.


nC: Nmero de tomos de carbono en el compuesto dado.

Fig. 1.10. Dependencia del NOP de la relacin DTO/Corg para algunos


compuestos.
15

Menndez Gutirrez, C. y J. Prez Olmo

Cuando se aplica la ecuacin 1.17 a las molculas de glucosa, formaldehdo


y cido actico, puede llegarse a la conclusin que en esos compuestos el carbono tiene el NOP igual a cero.
El carbono en todas las aminas alifticas y alcoholes tiene el NOP igual
D(QHOiFLGRFDSURLFRHVGH\HQHOiFLGRFtWULFRGH
Para los compuestos puros, la ecuacin 1.17 se transforma en:
NOP

42

DTO
Corg

(1.18)

Para aguas residuales la ecuacin 1.18 se modifica:


NOP 4  2

NOP

4

DQO / 16
Corg / 12
3 DQO
2 Corg

(1.19)

en la que tanto la DQO como la concentracin de carbono orgnico se


expresan en mg L.
En el mismo plano terico puede afirmarse que cuando el NOP del
sustrato es menor que cero se hace necesario suministrar oxgeno para la
sntesis celular:
5nC 6 H12 O 2  6nNH 3  10nO 2 o 6 C5 H 7 NO 2 n  18nH 2O

(1.20)

Para sustratos cuyo NOP es mayor que cero, el contenido de oxgeno en su


molcula es tal que no requiere un suministro adicional del mismo para que
ocurra la sntesis celular:
5nC6 H 8 O 7  6nNH 3 o 6 C5 H 7 NO 2 n 8nH 2 O  7,5nO 2

(1.21)

Como se aprecia de la ecuacin 1.21 an queda una cierta fraccin del


oxgeno que puede ser utilizado para la oxidacin.
Debe destacarse que cuando la biomasa es considerada como sustrato, su
NOP es igual a cero:

C5 H 7 NO2 n  5nO 2 o

5nCO 2  2nH 2O  nNH3

16

(1.22)

Indicadores de la Contaminacin

1.6. NITRGENO
El nitrgeno en las aguas residuales se puede presentar principalmente en
forma orgnica, formando parte de protenas, y en forma amoniacal. El nitrito
(NO2) y los nitratos (NO3) rara vez aparecen en las aguas residuales crudas,
y cuando existen, se trata fundamentalmente de aguas residuales industriales.
El conocimiento del contenido de nitrgeno en sus variadas formas resulta de gran inters. As por ejemplo, cuando un agua destinada a consumo
humano presenta nitrgeno orgnico, o nitrgeno amoniacal, es indicio de
contaminacin fecal reciente, lo cual es un alerta sobre su peligrosidad. La
urea contenida en la orina del hombre y animales, se descompone aportando
nitrgeno amoniacal segn muestra la siguiente ecuacin:
(1.23)

CON 2 H 4  H 2 O 
o CO 2  2NH 3

As mismo, las heces fecales de animales contienen cantidades apreciables


de protenas las cuales son convertidas en nitrgeno amoniacal por las bacterias
segn:
Nitrgeno orgnico  Bacterias


o

NH 3

(1.24)

El nitrgeno amoniacal puede ser posteriormente convertido en iones nitrito


por las bacterias Nitrosomonas:
bacterias

2NH 3(g)  3O 2 o 2NO 2   2H   2H 2 O

(1.25)

La presencia de iones nitrito en un agua destinada al consumo humano


puede ser considerada tambin como indicativo de contaminacin fecal.
Esta secuencia de reacciones no termina sino en los iones nitrato, que
constituyen la forma ms oxidada del nitrgeno. Esto ocurre cuando los iones
nitrito son oxidados por las Nitrobacter segn:
bacterias

2NO 2   O 2 
o 2NO 3 

(1.26)

La secuencia de la nitrificacin se puede mostrar grficamente, tal como


indica la figura 1.11.
La presencia de diferentes formas del nitrgeno tiene otra connotacin cuando
se trata de aguas residuales. En este caso es importante conocer el contenido
de nitrgeno por ejemplo cuando se requiere disear un sistema de tratamiento
biolgico donde se debe cumplir que el contenido de nitrgeno guarde relacin
definida con la DBO y el fsforo, para que el proceso de depuracin transcurra
de manera adecuada.
17

Menndez Gutirrez, C. y J. Prez Olmo

Fig. 1.11. Transformacin de diferentes formas del nitrgeno.


Por otra parte el contenido de nitrgeno y las diferentes formas en que se
puede presentar a lo largo del sistema de tratamiento aporta criterio sobre algunos de los fenmenos que pueden ocurrir.
En la tabla 1.4 se resume el impacto de diferentes formas del nitrgeno.
Tabla 1.4. Impactos causados por diferentes especies del nitrgeno
Forma del
Nitrgeno
N-NH3

N-NO3

Nitrgeno total

Impacto
La oxidacin del nitrgeno amoniacal en
un curso receptor produce disminucin
del oxgeno disuelto.
El nitrgeno amoniacal no ionizado
puede causar toxicidad en varios
organismos acuticos.
El nitrgeno en forma de nitrato en aguas
potables puede causar metahemoglobinemia en nios pequeos.
El nitrgeno como nutriente es causa de
un crecimiento excesivo de organismos
acuticos que consumen oxgeno durante
la noche y excretan sustancias que causan
olor y sabor.

18

Indicadores de la Contaminacin

1.7. FSFORO
El conocimiento del contenido de fsforo de las aguas residuales reviste
inters, pues este elemento constituye un factor imprescindible para la vida
de los organismos acuticos al entrar a formar parte de su estructura.
La presencia de compuestos de fsforo en cursos receptores induce el
crecimiento de algas. Estas afectan de forma notable la calidad de las aguas
ya que pueden ser el origen de toda una secuencia de fenmenos, dado que
este elemento es el limitante para el desarrollo de estas formas de vida. La
luz y los compuestos de nitrgeno (los cuales son los otros elementos imprescindibles) son generalmente abundantes.
El deterioro de la calidad del agua debido a la aparicin de color o sabor
desagradables no es el nico problema que se genera como consecuencia de
la presencia del fsforo en un cuerpo de agua, sino que se produce otro no
menos grave y es que al morir las algas, estas ocasionan una contaminacin
de carcter orgnico en el agua.
Por todo lo anterior es importante disponer de datos sobre la concentracin de fsforo que se vierte a los cuerpos receptores, provenientes fundamentalmente de las aguas residuales industriales y domsticas.
Las aguas residuales de origen domstico son relativamente ricas en
fsforo. Antes del desarrollo de los detergentes sintticos, el contenido
de fsforo usualmente estaba en el intervalo de 2 a 5 mgL  y las formas
orgnicas variaban de 0,5 a 1,0 mgL  . Esta contribucin humana se debe a
la ruptura de las protenas con la consiguiente eliminacin del fsforo en la
orina. En la actualidad muchos detergentes sintticos contienen grandes
cantidades de poli fosfatos, pudiendo contener hasta 10 % en su composicin. La sustitucin del jabn por estos productos ha incrementado notablemente el contenido de este elemento en las aguas residuales.

19

Menndez Gutirrez, C. y J. Prez Olmo

1.7.1. Diferentes compuestos de fsforo presentes en las aguas


residuales
En las aguas residuales se pueden encontrar compuestos de fsforo tales como:
Tabla 1.5. Diferentes compuestos de fsforo
Nombre
Ortofosfatos
Fosfato trisdico
Fosfato disdico
Fosfato monosdico
Fosfato diamnico
Polifosfatos
Hexametafosfato de sodio
Tripolifosfato de sodio
Pirofosfato tetrasdico

Frmula
Na3PO4
Na2HPO4
NaH2PO4
(NH4)HPO4
Na3(PO3)6
Na5P3O10
Na4P2O7

Todos los polifosfatos experimentan reacciones qumicas produciendo


ortofosfatos (PO4  ), siendo la velocidad con que ocurre este proceso dependiente del tiempo, la temperatura y el pH.

1.8. CARACTERSTICAS DE LAS AGUAS RESIDUALES


1.8.1. Aguas residuales municipales
Estn constituidas por materia orgnica en forma suspendida, coloidal y
disuelta. La carga contaminante de estas aguas puede ser expresada en
funcin de la contribucin per cpita.
La DBO, contenido de slidos y el gasto de agua per cpita que normalmente se reporta en la literatura vara de un pas a otro, de una zona o regin a otra
e incluso con el tiempo en una misma localizacin. Esto est, por supuesto, en
funcin de los hbitos alimentarios, de las costumbres y condiciones higinicosanitarias de la poblacin.
Los valores tpicos de la contribucin per cpita diaria a los residuales municipales son 0,5 m3 hab, 104 g hab de DBO y un contenido de slidos suspendidos de 90,7 g hab.10 Estudios parciales realizados en Cuba arrojan valores
de 30 g hab para la DBO y de 24 g hab para los slidos suspendidos.11
Como referencia, la composicin promedio de un agua residual municipal se muestra en la tabla 1.6. La incorporacin de residuales industriales
pueden modificar sustancialmente los valores de estas concentraciones. 12
20

Indicadores de la Contaminacin

Tabla 1.6. Caractersticas promedio de un residual municipal

Propiedad
pH
Slidos totales (mg L-1)
Slidos totales voltiles (mg L-1)
Slidos suspendidos (mg L-1)
Slidos suspendidos voltiles
(mg L-1)
DQO (mg L-1)
DBO (mg L-1)
Cloruros (mg L-1)

Mximo
7,5
640
503
258
208

Promedio
7,2
453
340
145
120

Mnimo
6,8
322
225
83
62

436
276
45

288
158
35

159
75
25

1.8.2. Caractersticas de los residuales industriales lquidos


(RIL)
Para tener un conocimiento real de las caractersticas de un agua residual no basta con conocer la magnitud, en trminos de concentracin, de los
parmetros indicadores seleccionados. Si bien el conocimiento de las concentraciones puede ser til en determinadas circunstancias, una mayor informacin se obtiene del conocimiento del flujo msico (kgd ).
As, por ejemplo, suponiendo dos residuales con las siguientes caractersticas:
Residual A:
Residual B:

DBO5 = 500 mgL


DBO5 = 250 mgL

Flujo = 200 m3d.


Flujo = 600 m3d

Pudiera pensarse que el residual A es ms contaminante que el B debido a


que tiene mayor concentracin de DBO. Obviamente el residual A presenta una
DBO superior al B, sin embargo el valor de su flujo o gasto es menor. El producto de la concentracin por el flujo da el valor del flujo msico. De esta manera,
para el ejemplo anterior se tiene que:
Flujo msico de A = 100 kgd
Flujo msico de B = 150 kgd
Este simple ejemplo pone de manifiesto que el conocimiento de la concentracin de un parmetro solamente no es suficiente para tener una idea real del
poder contaminante de un agua residual.
Lo anterior destaca la importancia de conocer, con el mayor grado de certeza posible, el flujo de agua residual que como promedio se genera. Por lo que se
debe utilizar un procedimiento de muestreo que responda en el mayor grado
21

Menndez Gutirrez, C. y J. Prez Olmo

posible a la realidad. Es decir, las determinaciones que se realicen deben ser


representativas, para lo cual estas deben ser proporcionales al flujo.
Para llegar a disponer de muestras integradas o compuestas, se requiere
que cada una de las tomas puntuales que constituirn la compuesta, sea obtenida teniendo en cuenta tanto el flujo promedio de agua residual como el existente
en el instante en que se toma la muestra. El flujo promedio debe ser calculado
previamente.
Algunas de las caractersticas qumicas de las aguas residuales municipales
reportadas por Walter13 aparecen resumidas en la tabla 1.7.
Para estimar el volumen de muestra a extraer en cada momento, puede
utilizarse la expresin:
Vi =

VT
Q i
Q N

(1.27)

donde:
Vi: volumen de muestra a tomar en un instante (L).
VT: volumen total de muestra a recolectar (L).
N: nmero de muestras que se van a tomar.
Q : flujo promedio determinado previamente (Lh).
Qi : flujo puntual en el instante de la toma de muestra (Lh).
En la aplicacin de este procedimiento es recomendable que durante el periodo de muestreo se tomen volmenes de muestras, se conserven en fro y se
mida el flujo puntual en el instante de tomar cada muestra.
Tabla 1.7. Caractersticas qumicas de las aguas residuales municipales

Constituyentes
cidos voltiles

cidos solubles
no voltiles
cidos grasos
superiores
Protenas y
aminocidos
Carbohidratos

Tipo
Frmico, actico,
propinico, butrico
y valrico
Lctico, gliclico,
ctrico y benzoico
Palmtico, esterico y
oleico
Al menos veinte
tipos
Glucosa, lactosa,
sacarosa

22

Concentracin
(8,5-20) mg L-1
(0,1-1,0) mg L-1
60 % del contenido
de cidos grasos
(45-50 %) del
nitrgeno total

Indicadores de la Contaminacin

La caracterizacin de un agua residual reviste dos objetivos muy importantes:


1. Inmediato. El conocimiento de las caractersticas del agua residual permite conocer su poder contaminante y decidir si el residual
debe ser sometido a tratamiento o no.
2. Mediato. Cuando las caractersticas del agua residual son tales que
es obligado su tratamiento, los datos que aporta la caracterizacin son una parte imprescindible para el diseo de la
instalacin de tratamiento.
En el curso de un proceso de caracterizacin se debe tener la mirada
puesta en la posibilidad de estudiar separadamente las diferentes corrientes
de residuales que pueden existir a fin de valorar su posible segregacin,
pues dependiendo de sus caractersticas pueden requerir de tratamientos
diferentes, por lo que no es aconsejable su mezcla. Esta ptica est, por
supuesto, condicionada en buena medida al estudio o conocimiento previo
del problema.
Para el caso particular de los residuales industriales lquidos (RIL), un
procedimiento para conocer el balance material y de flujo de todos los procesos de la industria que utilizan agua y producen residuales, as como de la
industria en su conjunto, puede resultar imprescindible.
El resultado de la caracterizacin debe brindar informacin acerca de la
fortaleza contaminante de cada corriente, as como de las alternativas de
tratamiento y de su reuso.
El procedimiento general recomendado para obtener la informacin necesaria para la caracterizacin con un mnimo de esfuerzo, y que al mismo
tiempo sea confiable, puede resumirse en cuatro etapas:

obtener el diagrama de flujo de los residuales,


elaborar el esquema de muestreo y anlisis,
efectuar el balance de flujo y materiales,
reportar la variacin estadstica de los parmetros ms significativos
de la caracterizacin.

1.8.2.1. Diagrama de flujo de los residuales


Este diagrama se construye a travs de la inspeccin de todas las operaciones del proceso y con la consulta del ingeniero de la planta. El diagrama debe
indicar los posibles puntos de muestreo y el orden de magnitud del flujo de la
corriente de aguas residuales.

23

Menndez Gutirrez, C. y J. Prez Olmo

1.8.2.2. Elaboracin del esquema de muestreo y anlisis


El muestreo ptimo es el continuo, con volmenes que son una fraccin del
flujo promedio. Sin embargo, esto pocas veces es factible.
Otra posibilidad es la de las muestras integradas o compuestas con una
frecuencia de muestreo que se establece de acuerdo con el proceso que se
necesita caracterizar. El periodo de la composicin de la muestra y la frecuencia de muestreo se establece de acuerdo con la naturaleza del proceso cuya
agua residual se estudia. Algunos procesos en operacin continua pueden
muestrearse cada hora e integrarse por periodos de 8, 12 y hasta de 24 h. Aquellos procesos en los que hay mucha variacin deben integrarse en periodos de
una o dos horas. A los procesos a templa se les toma muestras en el periodo de
vaciado.
Los anlisis a realizar dependen de las caractersticas del parmetro que se
est midiendo. Por ejemplo, el pH debe medirse puntualmente, ya que es posible
en muchos casos que la integracin resulte en una neutralizacin de elementos
cidos y bsicos perdindose durante la integracin una informacin valiosa. La
carga de DBO puede requerir integracin de las muestras durante 8 h cuando
se pretende disear tratamientos de tiempos de retencin cortos, aunque muestras integradas durante 24 h resultan suficientes para tratamientos de varios
das de retencin como el de las lagunas.
Cuando se miden constituyentes como nitrgeno y fsforo para determinar
si hay necesidad o no de adicin de nutrientes en tratamientos biolgicos, son
suficientes muestras integradas de 24 h, ya que los sistemas biolgicos tienen
alta capacidad de amortiguamiento. Si se conoce la presencia de sustancias
txicas en el agua residual, estas deben ser muestreadas continuamente.
Un aspecto importante en este punto, al que ya se hizo referencia, es el de la
medicin del flujo.

1.8.2.3. Balance de flujo y materiales


Despus de realizados el muestreo y los anlisis, debe efectuarse el balance
de flujo y materiales tomando en consideracin todas las fuentes de contaminacin que sean significativas. En la medida en que el balance coincida con el
efluente total, se confirmar el cuidado con el que se realiz la caracterizacin.

1.8.2.4. Variacin estadstica de los parmetros


Esta informacin se obtiene de diversas formas, una de ellas consiste en
graficar en papel de probabilidades la frecuencia de ocurrencia.
24

Indicadores de la Contaminacin

La data de la caracterizacin de los RIL generalmente es muy variable y


susceptible de anlisis estadstico. La informacin puede ser reportada en trminos de frecuencia de ocurrencia de una propiedad en particular, que es el
valor que se espera que tenga esa propiedad o lo exceda 10, 50 y 90 % del
tiempo. Los valores obtenidos en la caracterizacin se ordenan en orden creciente de magnitud. Por ejemplo, si se trata de la DBO,
donde:
n: Nmero total de valores de DBO.
m: Nmero consecutivo y en orden creciente asignado a cada uno de los
valores de DBO, desde uno hasta n.

m
n  1 : posicin de cada valor en la tabla y que es equivalente al porcentaje
de ocurrencia.
Tabla 1.8. Porcentaje de ocurrencia
m

DBO
-1
(mg L )

m
n 1

Ocurrencia
(%)

1
2
3
4
5
6
7
8
9

175
225
285
340
350
375
405
460
515

0,1
0,2
0,3
0,4
0,5
0,6
0,7
0,8
0,9

10
20
30
40
50
60
70
80
90

Al graficar estos resultados en papel de probabilidades, tal como se hace en


la figura 1.12, la DBO promedio es la que corresponde a frecuencia de ocurrencia del 50 %. Por otro lado,
aceptando que 68 , 27 % de los valores estn
_
incluidos en el intervalo: X r sn 1 , se tiene que:
sn 1

X 84 ,1  X 15 ,9
2

Para el ejemplo anterior:


DBO promedio = 347,8 mg L
25

(1.28)

Menndez Gutirrez, C. y J. Prez Olmo

sn 1

450  230
110
2

s: desviacin estndar.

Fig. 1.12. Porcentaje del tiempo que la DBO ser igual o menor al valor
indicado.
Cuando el nmero de observaciones es muy grande (de cincuenta a cien) se
recomienda agrupar la data por incrementos. Estos incrementos pueden tomarse de 50 mg L.
Ejemplo 4.1
Suponga una data que tiene noventa y seis observaciones agrupadas por
intervalos de 50 mg L.
donde:
n: Nmero total de muestras
m: Nmero de muestras en el intervalo analizado y los anteriores
m
100 : posicin al graficar..
n 1

26

Indicadores de la Contaminacin

Tabla 1.9. Intervalos y nmeros de muestras


Intervalo
200 - 249
250 - 299
300 -349
.
.
.
1150 1 199

Muestra
en el intervalo
5
4
6
.
.
.
4

m
5
9
15
.
.
.
96

Posicin
en el grfico
5,2
9,3
15,5
.
.
.
99,0

Las mediciones del caudal de aguas residuales durante el estudio pueden


llevarse a cabo por diversos mtodos, en dependencia de la accesibilidad del
lugar. Estos pueden ser, entre otros:
1. Instalacin de vertederos, se usa en canales o conductoras abiertos.
2. Cubo o cubetas y cronmetro: aplicable cuando hay bajo caudal.
3. Medicin de la duracin del bombeo: hay que hacer uso de la curva caracterstica de la bomba.
4. Uso de medidores de flujo de paleta.

1.8.3. Empleo de ndices en la caracterizacin


Tomando en cuenta que los procedimientos de caracterizacin son generalmente complejos y exigen de importantes recursos de tiempo y de tipo
financiero, la utilizacin de indicadores o ndices de produccin y consumo
se presenta como un importante enfoque a considerar, dentro de los procedimientos utilizados para determinar, al menos aproximadamente, los niveles
de carga contaminante que son generados por las diferentes actividades de
produccin y servicios.
Los ndices de produccin constituyen un instrumento que permite determinar la efectividad de medidas y programas orientados a la disminucin de la
contaminacin, facilitando la identificacin y establecimiento de prioridades de
gestin, con arreglo a la evaluacin cuantitativa de las tendencias que se manifiestan en la calidad del medio.
En este sentido, es importante indicar que esta herramienta de trabajo
no sustituye ni exime, la necesidad de realizar investigaciones ms completas.

27

Menndez Gutirrez, C. y J. Prez Olmo

Tabla 1.10. Indicadores de carga contaminante de algunos procesos

Proceso
Instalaciones de
cra de reses
Instalaciones de
cra de pollos
Granjas lecheras
Mataderos
Sin recuperar
sangre
Sin recuperar
panza se agrega
Proceso de aves
de corral
Enlatado de
frutas y verduras
Refinacin de
aceite vegetal
Produccin de
vino
Produccin de
cerveza

Unidad

DBO
(kg/u)

DQO
(kg/u)

Slidos
suspendidos
(kg/u)

250

1716

1,4

14,6

539
6,4

5,2

11

4,7

11,9

22.4

12,7

12,5

4,3

12,9

21

16,4

0,26

8,6

14,7

cabezas

PVS

1000 aves
t de
produccin
m3 de vino
3

m de
cerveza

PVS: Peso vivo sacrificado.


Fuente: Evaluacin rpida de fuentes de contaminacin de aire, agua y suelo, OMS, Mxico, 1988.

El indicador de carga contaminante o ndice es la relacin entre el nivel


de produccin y la carga contaminante que se genera en esa actividad. El
ndice define una cantidad de sustancia determinada que se obtiene en la
actividad productiva en un tiempo considerado, refiriendo a l la contaminacin generada durante el proceso de obtencin.
Los ndices han de ser el resultado de experiencias anteriores, sean propias u obtenidas de la literatura.
En la tabla 1.10 se presentan ejemplos de los ndices de produccin referidos a la contaminacin que generan.
De igual manera puede ser de importancia el ndice de consumo de agua.
Ejemplos de este tipo de ndice se reflejan en la tabla 1.11.

28

Indicadores de la Contaminacin

Tabla 1.11. Consumo de agua de la industria azucarera y derivados


Industria
Central azucarero
Destilera
Levadura

ndice de consumo
0.5 m3/t caa molida
1.6 m3/HL de alcohol
70 m3/t de levadura
(tecnologa francesa)
80 m3/t levadura
(tecnologa austraca)

Fuente: Programa integral de desarrollo de los derivados de la caa de azcar. Subprograma: Alcohol
y levaduras, septiembre 1985, Cuba.

1.9. MEDICIONES DEL CAUDAL


El conocimiento del gasto, flujo o caudal es importante no solo en el
momento de proyectar una instalacin, su sistema de tuberas y de bombeo.
Se encuentran diversas formas comunes de conocer un caudal. Desde
trabajar por datos de proyecto, pasando por el manejo de ndices de consumo por unidad de produccin o por habitante, hasta la medicin directa. De
las mediciones directas del caudal de aguas residuales trata este epgrafe.
Los medidores de caudal ms usuales que pueden ser encontrados en
las plantas de tratamiento de aguas residuales son:
Canales de aforo Parshall.
Vertedores.
Medidores de caudal en lneas de presin.

1.9.1. Canal Parshall


El canal Parshall est formado por una seccin de entrada de paredes
verticales convergentes y fondo a nivel, un estrechamiento de paredes paralelas y fondo descendente. El canal posee adems una seccin de salida de
paredes divergentes y fondo ascendente. En la figura 1.13 se muestra un
esquema de un canal Parshall.
Este tipo de medidor es especialmente til para aguas que contienen slidos
en suspensin, debido a que la presencia de estos no afecta las mediciones. Por
otra parte, el aumento de la velocidad del agua a su paso por la parte ms
estrecha del canal, la garganta, dificulta la sedimentacin de las partculas.
La determinacin del caudal se realiza considerando la altura alcanzada
por el agua (H), tomada en una arqueta aneja conectada por un tubo
piezomtrico.
29

Menndez Gutirrez, C. y J. Prez Olmo

El caudal instantneo en cada momento se obtiene por aplicacin de la


ecuacin:
Q

0,37 3,28 H

1,567 W

0,026

(1.29)

siendo:
Q: caudal instantneo (m3 s).
W: ancho del estrechamiento, (m).
H: calado del agua en el punto de observacin fijo, (m).

Fig. 1.13. Esquema de un medidor Parshall.


La medicin manual de la altura (H), arroja como resultado un valor
instantneo del caudal. Existe en el mercado un conjunto de instrumentos
para realizar la medida de forma continua y as obtener el caudal medio
diario, punta, mensual, etc., de forma precisa. En ocasiones adems es de
inters acoplar el canal a un indicador con registro grfico y totalizador.

30

Indicadores de la Contaminacin

Fig. 1.14. Canal Parshall.

1.9.2. Vertedores
Los vertedores consisten bsicamente en una obstruccin en la que se estanca el lquido y vierte por encima de ella (Figura 1.15).

Fig. 1.15. Esquema de un vertedor.


31

Menndez Gutirrez, C. y J. Prez Olmo

En el uso de los vertedores se emplean trminos que requieren ser defiQLGRVWDOHVFRPRFUHVWDDOWXUDGHODFUHVWDFRQWUDFFLyQGHODVXSHUIL


FLH\FDUJDGHOYHUWHGHUR6HGHQRPLQDFUHVWDDOERUGHSRUHOTXHIOX\H
HOOtTXLGRDOWXUDGHODFUHVWDDODGLVWDQFLDGHVGHHOIRQGRGHOFDQDODOD
FUHVWDFRQWUDFFLyQGHODVXSHUILFLHDODIRUPDFXUYDTXHGHVFULEHHOOLTXL
GRVREUHODFUHVWD\FDUJDGHOYHUWHGRU (H) a la diferencia entre la altura
de la superficie alcanzada por el liquido y la altura de la cresta medida aguas
DUULEDGHOYHUWHGRUDQWHVGHODFRQWUDFFLyQ
Una vez medido el valor de la carga del vertedor, H, se puede obtener el
caudal de agua residual que pasa en cada instante.
Hay dos elementos que deben ser tomados en consideracin cuando
se desea o necesita instalar un vertedor para la medicin de caudales.
Estos son:
partiendo del hecho de que la contraccin de la vena lquida comienza a
una distancia aproximada de 2H aguas arriba del vertedor, la medida
debe efectuarse a una distancia mayor que esta,
la altura mnima de la cresta debe ser de 2,5 H para permitir la contraccin completa de la lmina.
Los vertedores ms utilizados son:
Rectangular.
Triangular.
Trapezoidal.
Vertedor rectangular
Estos pueden ser sin contraccin lateral o con ella.
En aquellos casos en los que no exista contraccin lateral, la cresta ocupa
todo el ancho del canal. El caudal viene dado por la ecuacin,
Q 1,84 L H 1,5

(1.30)

siendo:
Q: caudal, m3 s.
L: longitud de la cresta, m.
H: carga sobre la cresta, m.
Las dimensiones que debe cumplir el vertedor se detallan en la figura 1.16.

32

Indicadores de la Contaminacin

Fig. 1.16. Relacin de las dimensiones de un vertedero rectangular sin


contraccin lateral.
En los vertedores rectangulares con contraccin lateral, la cresta no ocupa
toda la anchura del canal. Las dimensiones de las contracciones laterales se
especifican en la figura 1.17.
La ecuacin que permite calcular el caudal en este tipo de vertedor es,
Q 1,84 L  0 ,1 n H H 1,5

(1.31)

donde n es el nmero de contracciones laterales.

Fig. 1.17. Relacin de las dimensiones en un vertedor rectangular con


contraccin lateral.
33

Menndez Gutirrez, C. y J. Prez Olmo

Vertedor triangular
Este tipo de vertedor es uno de los ms utilizados debido a que, para un
mismo caudal, se consigue mayor altura que en los rectangulares. Este efecto
se traduce en una mayor precisin en la lectura (figura 1.18).
El ngulo del vrtice de este vertedor puede tener diferentes valores. Los
ms usuales son 60 y 90. La expresin que da el caudal en este tipo de vertedor es:
Para 90

1,38 H 2,5

(1.32)

donde:
Q: gasto, m3 s.
H: altura del lquido con relacin al vrtice, m.
Para 60

0,79 H 2 ,5

(1.33)

Fig. 1.18. Relacin de las dimensiones en un vertedor triangular.

Fig. 1.19. Relacin de las dimensiones en un vertedor trapezoidal.


34

Indicadores de la Contaminacin

Vertedor trapezoidal
Este vertedor tiene una forma similar al rectangular (figura 1.19). La expresin que se emplea para calcular el caudal es:
Q 1,859 L H 1,5

(1.34)

L: la longitud de la cresta del vertedero, m


Medidores de caudal en lnea
Los mtodos usualmente utilizados en la determinacin de caudales en
tuberas a presin corresponden a los llamados mtodos dinmicos, que suponen una aplicacin del teorema de Bernouilli entre dos puntos de una tuEHUtD/RVGLVSRVLWLYRVPiVHPSOHDGRVVRQORVWXERV9HQWXUL

Fig. 1.20. Tubo Venturi.

1.10. DISMINUCIN DEL VOLUMEN Y FORTALEZA


DE LOS RESIDUALES
Con frecuencia, haciendo sencillas modificaciones en la industria, se
pueden lograr reducciones apreciables en el volumen y fortaleza de las aguas
residuales, obteniendo como consecuencia de ello disminuciones en los costos de inversin y operacin de las instalaciones de tratamiento, entre estas
modificaciones puede sealarse:
Recirculacin: el agua que est relativamente poco contaminada puede ser
recirculada con un mnimo de tratamiento o sin tratamiento alguno.
Clasificacin de las diversas corrientes y segregacin de las mismas: el
agua de enfriamiento y otras poco contaminadas pueden segregarse
35

Menndez Gutirrez, C. y J. Prez Olmo

con anterioridad al tratamiento. Las aguas que contengan sustancias


txicas no deben llegar nunca a las unidades de tratamiento biolgico.
Sustitucin de aditivos qumicos: hay casos en los que se facilita el
tratamiento sustituyendo algunas materias primas cuyo residuos sean
resistentes por otros ms fcilmente tratables.
Recuperacin de subproductos: en ocasiones con instalaciones de bajo
costo se logra obtener un subproducto dando lugar a una produccin
marginal, o a la recuperacin de alguna materia prima del propio proceso productivo.
La disminucin en las variaciones del caudal y de la fortaleza de las
aguas residuales deben ser prioritarios, lo que puede ser logrado, entre otros
medios, mediante el empleo de tanques compensadores.

Notas bibliogrficas
1 CHUDOBA, J., C. MENNDEZ Y J. PREZ O.: Fundamentos tericos de algunos
procesos para la purificacin de aguas residuales, Ed. ISPJAE, Ciudad de La Habana, 1986.
2 THOMAS, M.A.: *UDSKLFDO GHWHUPLQDWLRQ RI %2' FXUYH FRQVWDQWVWater
and Sewage Works, 1950.
3 MOORE, E.W., H.A. THOMAS, AND W.B. SNOW.:6LPSOLILHGPHWKRGIRUWKHDQDO\VLV
RI%2'GDWDSewage and Ind. Wastes, vol. 22, no. 10, 1950.
4 RHAME, G. A.: 7ZR3RLQW0HWKRGIRUHVWLPDWLRQRIILUVWVWDJH%2'Sewage
and Ind. Wastes, vol. 28, no. 9, 1950.
5 Standard Method for the Examination of Water and Wastewater, Ed.
APWA-AWWA, 1995.
6 MOORE, W.A.:'HWHUPLQDWLRQRIR[\JHQFRQVXPHGYDOXHRIRUJDQLFZDVWHV
Anal. Chemistry, 1951.
7 ARGAMAN, Y. AND A. BRENNER:([SHULPHQWDOHYDOXDWLRQRIPRGHOLQJDOWHU
natives and process parameters for the single-sludge nitrogen removal
V\VWHP IAWPRC Specialised Seminar, Copenhagen, Denmark, August, 1985.
8 MENNDEZ, C.: Reporte Indito, ISPJAE, Ciudad de La Habana, 1989.
9 ECKENFELDER, W.W.: Water Quality Engineering for Practicing Engineers,
CBI, Pub. 1980.
10 LOEHRS, H.C.:9DULDWLRQRIZDVWHZDWHUSDUDPHWHUVPub. Works, vol. 99,
no. 81, 1968.
11 RODRGUEZ, C.: &DUDFWHUL]DFLyQGHODVDJXDVUHVLGXDOHVGH%HMXFDO Ingeniera Hidrulica, vol. 19, no. 4, 1998.
36

Indicadores de la Contaminacin

12 HUNTER, J.V. AND H. HEUKELEKIAN: 7KH FRPSRVLWLRQ RI GRPHVWLF VHZDJH


IUDFWLRQV-RXUWater Poll. Control Fed., vol. 37, no. 8, 1965.
13 WALTER, L.:&RPSRVLWLRQRIVHZDJHDQGVHZDJHHIIOXHQWVSDUWHV\
Water Sewage Works, vol. 109, no. 11 y 12, 1961.
14 OMS, REPORTE: Evaluacin rpida de fuentes de contaminacin de aire,
agua y suelo, Mxico, 1988.
15 Programa integral de desarrollo de los derivados de la caa de azcar.
Sub-programa alcohol y levaduras, Cuba, 1985.

37

Pretratamiento de Aguas Residuales

CAPTULO 2

PRETRATAMIENTO DE AGUAS RESIDUALES

El tratamiento biolgico de las aguas residuales, fundamentalmente las


de origen industrial, en ocasiones requiere de algn tipo de pretratamiento
cuyo objetivo puede ser la remocin de contaminantes, como ocurre durante la sedimentacin primaria, o el acondicionamiento del agua residual
mediante la compensacin y (o) neutralizacin, para facilitar la depuracin
biolgica.

2.1. COMPENSACIN
Los residuales lquidos industriales se caracterizan por la diversidad
de corrientes con diferencias en su composicin y concentracin. Aun una
misma corriente puede presentar variaciones horarias en sus propiedades. Cuando
los residuales manifiestan mucha variacin en su composicin se recomienda el
empleo de tanques compensadores.
La experiencia ha demostrado que los procesos de tratamiento se realizan
mejor si las fluctuaciones extremas en la carga del sistema pueden ser evitadas
o en ltima instancia ser atenuadas mediante la compensacin.
En los tanques compensadores los residuales se retienen durante un periodo
en el que se llega a obtener un efluente relativamente estable.
La neutralizacin de corrientes cidas o bsicas, la estabilizacin de la DBO
y la separacin de metales pesados, son algunos de los objetivos de la compensacin.
Aguas residuales con alta acidez o alcalinidad que no son compensadas
requieren de la neutralizacin mediante la adicin de reactivos qumicos.
La mezcla en el tanque de compensacin generalmente se promueve con
aire. Un mtodo comn es el empleo de aereadores superficiales, con un consumo de potencia de 4.10   W m. Cuando se utiliza aire mediante
difusores, el consumo aproximado de este puede tener un valor cercano a los
4 m3 m d.1
39

Menndez Gutirrez, C. y J. Prez Olmo

Para el diseo de los tanques de compensacin pueden emplearse criterios


empricos y calcular su volumen considerando el tiempo requerido para acumular el agua residual de un ciclo de produccin o por turnos de trabajo. Siempre
que se pueda, el criterio de diseo debe tener como base el nivel de varianza
aceptable que tiene el parmetro que se desea normalizar para las etapas subsiguientes del tratamiento.
Los tanques compensadores pueden disearse para operar a volumen constante o volumen variable, en funcin de que interese amortiguar variaciones de
concentracin o flujo respectivamente.

2.1.2. Dinmica de los sistemas de compensacin


La compensacin se logra vertiendo el residual en un tanque o cisterna de
forma tal que se mezcle con el que le ha precedido, con lo cual se amortiguan las
variaciones bruscas.

Fig. 2.1. Efecto de un compensador sobre variaciones de flujo y / o


composicin.
A fin de ilustrar la dinmica de un compensador considere el caso ms
sencillo de un tanque perfectamente agitado de volumen constante, al cual
llega continuamente una corriente de agua residual de flujo Q y concentracin del componente de inters CE, siendo la salida de igual flujo pero de
concentracin CS.

Fig. 2.2. Compensador de volumen constante.


40

Pretratamiento de Aguas Residuales

Aplicando un balance diferencial para el componente de inters:

Q CS  V

Q CE

dCS
dt

(2.1)

por lo que integrando:


CS

C0

dCS
C E  CS

Q
dt
Vt 0

CE  CS
CE  C0

(2.2)

por lo tanto:

ln

donde:

V
Q

entonces:

CS

t
CE  (C E  C0 ) exp 

(2.3)

En la tabla 2.1 se muestra un ejemplo numrico donde se ilustra el efecto


compensador del sistema mostrado, suponiendo por simplicidad, que el flujo de
agua residual es constante y la composicin variable.
Tabla 2.1. Efecto compensador del sistema

Horario
1
2
3
4
5
6
7
8
9
10
11
12

DQO, mgL-1
150
175
180
200
250
300
320
300
250
180
150
140
41

Menndez Gutirrez, C. y J. Prez Olmo

Se ha supuesto que el tiempo de residencia en el compensador es de 10


KRUDV 

Fig. 2.3. Ilustracin del efecto de un compensador de volumen constante.


En la figura 2.3 se muestran las curvas de variacin de DQO de entrada al
compensador, las del efluente compensado y la composicin promedio. De la
misma puede observarse el efecto compensador del sistema, ya que la variacin
que experimenta la composicin de salida es menor que la de entrada.
Es de destacar que el compensador no modifica la composicin promedio
del efluente con respecto al caudal de entrada, sino su varianza.
Al incrementarse el tiempo de residencia del compensador aumenta su efecto
amortiguador, por lo que para un tiempo de residencia infinito desaparecen las
fluctuaciones de la composicin de salida igualndose a la promedio.
El mtodo que se siga para calcular las dimensiones del tanque compensador
depender de las condiciones en que este operar.

2.1.3. Compensacin para flujo de residual constante


y de composicin variable
Bajo condiciones de flujo constante la relacin entre la varianza del efluente
de un tanque completamente mezclado y la del afluente, cuando las variaciones
de composicin son aleatorias, viene expresada por la relacin:2
S 2e
S 2i

7
2 .

42

(2.4)

Pretratamiento de Aguas Residuales

donde:
S2e: varianza de la concentracin del efluente.
S2i: varianza de la concentracin del afluente.
'T: tiempo de recoleccin de la muestra compuesta.
T : tiempo de retencin (V / Q).
De acuerdo con lo anterior puede calcularse el tiempo de retencin si se
conocen la varianza del afluente y la permisible o deseada para el efluente.
La varianza en el afluente, S2i se determina segn:
S 2i

1
n 1

Xi  X

(2.5)

i 1

donde:
Xi: valor individual de la concentracin de cada muestra tomada.
X : valor promedio de todas las mediciones.
n: nmero de muestras.
Se define el valor tpico normal Z de la propiedad controlada como:

Xm  X
Se

(2.6)

donde:
Xm: valor mximo que puede tomar la propiedad en el efluente del tanque de
compensacin.
Se: desviacin tpica del efluente.
Procedimiento para el diseo
El procedimiento de diseo consiste en hallar el tiempo de retencin requerido en el tanque compensador para obtener las condiciones de salida deseadas.
El tiempo de retencin y el caudal de agua a tratar, definirn el volumen de
tanque necesario.
1. Determinar el valor promedio de la concentracin del afluente X .
2. Calcular su desviacin tpica.
3. Fijar el valor mximo permisible de concentracin que se desea a la salida
(Xm).
4. Determinar la desviacin tpica del efluente tomando un valor de Z con un
por ciento de confiabilidad, por ejemplo, 90 %.
5. Hallar el valor de T .

43

Menndez Gutirrez, C. y J. Prez Olmo

6. Con el valor de T y el flujo que se desea procesar se estima el volumen del


compensador:

V
Q

Ejemplo 2.1

Se estudi un agua residual cuyo caudal es de 1 500 m 3d. Se tomaron


cuatro muestras diarias integradas durante 6 horas cada una en 10 das diferentes. Los resultados se muestran en la tabla 2.2:
Tabla 2.2. Resultados del ejemplo 2.1
HORA
Da
1
2
3
4
5
6
7
8
9
10

6:00 a.m.
850
1430
1550
1390
410
500
420
960
395
600

12:00 m 6:00 p.m. 12:00 a.m.


Valores de DBO (mgL-1)
1240
600
1130
324
1290
741
421
750
422
1324
1050
560
990
684
1220
270
820
530
450
750
1205
1230
680
890
1190
300
500
1304
1020
922

Debido al tratamiento que se emplear se requiere disminuir la variacin que


presenta la DBO se pretende disear un tanque para compensar tales efectos.
Determine el volumen del compensador si se conoce que el valor mximo
permisible de la DBO a la salida es de 1000 mgL.

X
n

832,8

Si 367,438
Xm 1000
De la tabla de distribucin normal (Anexo 2), para 90 % de confianza, el
valor de Z 1,30

Xm  X
Se

Se

44

1000  832 ,8
167 ,2
1,30

Pretratamiento de Aguas Residuales

S 2e
S 2i

7
2 .

Se2 = 27955,84; Si2 = 135810,68

El tiempo de retencin en el tanque compensador ser,

6 135810 ,68
14 ,5 h
2 27955 ,84

T 0,6 d, por lo tanto, el volumen es,


V 1500 0,6 900 m3

2.1.4. Compensacin cuando hay variacin simultnea de flujo


y composicin3
Realizando un balance de masa en el tanque compensador:
Q C1 T  V C0

Q C2 T  V C

(2.7)

donde:
C1: concentracin que entra al tanque durante el intervalo de muestreo T.
T: intervalo de muestreo, por ejemplo, 1 h.
Q: flujo promedio en el intervalo de muestreo.
C: concentracin en el tanque al inicio del intervalo de muestreo.
V: volumen del tanque.
C2: concentracin que sale del tanque al concluir el intervalo de muestreo.
En este caso se est asumiendo que la concentracin del efluente es constante durante un intervalo de muestreo. Esto es cierto si el tiempo del intervalo
es espaciado apropiadamente.

C2

C1T  C 0V/Q
T  V/Q

(2.8)

Procedimiento para el diseo


1. Se asume un volumen del tanque compensador, y este define el tiempo de
retencin.
2. Tomando como primer valor de C0 el valor promedio de las mediciones, se
calculan los valores de la concentracin compensada. Para los restantes
intervalos se toma la concentracin final del intervalo anterior.
3. Si los valores de concentracin as obtenidos no llegan a satisfacer los requerimientos, se toma otro valor de tiempo de retencin y repite el proceso.
45

Menndez Gutirrez, C. y J. Prez Olmo

Ejemplo 2.2
Tabla 2.3. Resultados
Q
(m3h-1)

Intervalo
de
tiempo
1
2
3
4
5
6
7
8
Promedio

Concentracin
de DBO
(mgL-1)
250
70
60
170
330
50
60
400
173,75

363
45
230
270
363
450
290
260
283,87

La tabla 2.3 fue obtenida de una planta industrial durante un ciclo de produccin de 8 horas. Cada intervalo de tiempo representa un periodo de muestreo de
1 hora. Determine el factor pico del efluente de un tanque compensador cuando
se utiliza un tiempo de retencin de: a) 8 horas, b) 2 horas y comprelos con el
factor pico cuando no se utiliza compensacin.

Factor Pico

Valor mximo
Valor promedio

400
2 ,3
173,75
a) Asumiendo un tiempo de retencin igual a 8 h:

Al inicio: FP

V = 283,87 8 = 2270,96 m3
C1  C0
C2
1

V
Q

V
Q

Despus del primer intervalo,


C1 = 250 mgL

Q = 363 m3h

46

C0 = 173,75 mgL

Pretratamiento de Aguas Residuales

2270 ,96
363
2270 ,96
1
363

250  173,75
C2

184 ,39 mgL1

Despus del segundo intervalo:


C1 = 70 mgL
Q = 45 m3h
2270 ,96
45
2270 ,96
1
45

70  184 ,39
C2

C0 = 184,79 mgL

182 ,17 mg L1

Al proceder de manera similar para los restantes intervalos se obtienen los


resultados que se muestran en la tabla 2.4:
Tabla 2.4. Resultados
Intervalo

C1

Primero
Segundo
Tercero
Cuarto
Quinto
Sexto
Sptimo
Octavo
Promedio

250
70
60
170
330
50
60
400
173,75

363
45
230
270
363
450
290
260
283,37

Concentracin compensada (C2)


Para T 2 h
Para T 8 h
173,75
203,49
182,17
193,70
170,93
155,15
170,83
160,00
192,77
226,30
169,16
148,35
156,68
109,28
156,80
200,60

Factor Pico para un tiempo de retencin de 8 h en el tanque compensador:


FP

192 ,70
1,1
173 ,75

b) Asumiendo un tiempo de retencin igual a 2 h:


V = 283,87 2 = 567,74 m3
Despus del primer intervalo,
C1 = 250 mgL

Q = 363 m3h
47

C0 = 173,75 mgL

Menndez Gutirrez, C. y J. Prez Olmo

567 ,74
363
567 ,74
1
363

250  173 ,75


C2

203,49 mgL1

Despus del segundo intervalo:


C1 = 70 mgL
Q = 45 m3h
567 ,74
45
567 ,74
1
45

70  184 ,39
C2

C0 = 203,49 mgL

193 ,70 mgL1

Procediendo de manera similar para los restantes intervalos se obtienen los


resultados que se muestran en la tabla anterior para: T = 2 h.
Factor Pico para un tiempo de retencin de 2 h en el tanque compensador:
FP

226 ,30
1,3
173,75

2.1.5. Compensacin cuando hay variacin simultnea de flujo


y composicin, pero el volumen en el compensador es
variable
El volumen requerido para la homogeneizacin o compensacin se determina mediante un diagrama de los caudales a tratar, en el cual se representa el
volumen de afluente acumulado a lo largo del da. El caudal medio diario, tambin representado en el mismo diagrama, es la pendiente de la lnea recta trazada desde el origen hasta el punto final del diagrama.
Para determinar el volumen necesario, se traza una recta paralela a la que
define el caudal medio diario, tangente a la curva de caudales acumulados. El
volumen requerido es igual a la distancia vertical existente entre el punto de
tangencia y la lnea recta que representa el caudal medio, tal como muestra la
figura 2.4.
Si una parte de la curva de caudales acumulados est situada por encima de
la lnea que representa el caudal medio, el diagrama acumulado debe limitarse
con dos lneas paralelas a la del caudal medio y tangente a las dos curvas del
diagrama. El volumen requerido en este caso es igual a la distancia vertical
existente entre las dos tangentes. Figura 2.5.

48

Pretratamiento de Aguas Residuales

Horario
Fig. 2.4. Determinacin del volumen del compensador.

Volumen
acumulado

La interpretacin fsica de los diagramas representados es la siguiente:


En el punto inferior de tangencia el tanque de compensacin est vaco. A
partir de este punto el tanque empieza a elevar su nivel por lo que la pendiente
del diagrama de la curva de volumen del afluente es mayor que la del caudal
medio diario y contina llenndose hasta que lo hace al final del horario. Para el
segundo modelo de flujo, el tanque est totalmente lleno en el punto de tangencia superior.

Horario
Fig. 2.5. Determinacin del volumen del compensador.
49

Menndez Gutirrez, C. y J. Prez Olmo

Procedimiento para el diseo


1. Se desarrolla una curva de caudales acumulados de agua expresada en metros cbicos.
V
Vf

Q T
Vo  Vi  VS

V1 Vo 1
V2

Vo 1  V1 2 Vn

V01  V1 2  .............  V( n1)  n

2. Se traza la lnea de caudal medio diario.


3. Se determina el volumen del tanque requerido.
4. Para estimar el efecto de homogeneizacin:
a) Se calcula el volumen de lquido existente en el tanque al final de cada
periodo de tiempo mediante la expresin:

Vf

Vo  Vi  Vs

donde:
Vf: volumen en el tanque al final del periodo de tiempo considerado.
V0: volumen en el tanque al final del periodo de tiempo anterior.
Vi: volumen aportado durante el tiempo considerado.
VS: volumen de caudal saliente durante el tiempo considerado.
b) Se calcula la concentracin media que sale del tanque:

Xf

Vi X i  V0 X 0
Vi  V0

donde:
Xf: concentracin media en el caudal que sale del tanque durante el tiempo
considerado mg / L.
Vi: volumen aportado durante el tiempo considerado, m3.
V0: concentracin del agua residual contenida en el tanque al final del periodo anterior.
5. La magnitud de la carga horaria se calcula utilizando la expresin:
Carga

horaria

X i Qi

El efecto de la homogeneizacin puede mostrarse numricamente a partir


de las relaciones siguientes:
punta / media; mnima / media y punta / mnima
50

Pretratamiento de Aguas Residuales

Estas relaciones se comparan para el residual antes y despus de homogeneizar. Para el residual homogeneizado estas relaciones estn ms prximas al
valor de 1,0.
En la prctica el volumen del tanque de compensacin debe ser algo superior al determinado tericamente (10 al 20 %) para tener en cuenta los dos
factores que se destacan a continuacin:
La operacin continua de los equipos de aereacin y mezclado no permitirn un vaciado total.
Debe contemplarse un volumen adicional para hacer frente a los imprevistos que puedan producirse para cambios no esperados del caudal diario.
Ejemplo 2.3
En las columnas 1 y 2 de la tabla 2.5 que se reporta ms abajo, estn recogidos los datos de flujo y concentracin de un agua residual industrial que ser
sometida a tratamiento. Para ello se necesita que el valor del flujo se mantenga
lo ms constante posible. A tales efectos se desea conocer el volumen que ha de
tener el tanque compensador requerido.
Determine adems el volumen de agua que contendr el tanque cada hora.
La industria opera durante 12 horas al da, y los datos que se ofrecen son los
promedios de cada intervalo medido.
Para estimar el volumen del tanque compensador se traza la curva de 6V vs
intervalo de tiempo. Considerando que el primer intervalo en el que se tomaron
las muestras termina a las 8 am, el valor de 6V es el del intervalo de 7 a 8 a.m.:
6V 10.
Para el intervalo siguiente: 6V 10  15 25
De la misma manera se procede con los siguientes intervalos. Los valores
de 6V aparecen en la columna 3 de la tabla 2.5.
Se traza la curva correspondiente y se unen mediante una recta los dos
extremos de la curva. La pendiente de esta recta es numricamente igual al
flujo promedio, y es al mismo tiempo el flujo de salida que debe tener el
compensador.
En la figura 2.6 se aprecia que la curva presenta un valor mnimo con respecto a la recta a las 10:00 a.m. (horario 3 en el grfico), y un valor mximo a
las 2:00 p.m. (horario 7 en el grfico). Estos son las horas que se corresponden,
al menos tericamente con los momentos en los que el tanque est vaco y lleno
respectivamente. Mientras tanto, el efluente se mantiene constante y con un
valor igual al flujo promedio de la entrada.

45,83 m3h 1
51

Menndez Gutirrez, C. y J. Prez Olmo

Tabla 2.5. Datos para el tratamiento de un agua residual industrial

Intervalo
de
muestreo
7-8 a.m.
8-9
9-10
10-11
11-12
12-1 p.m.
1-2
2-3
3-4
4-5
5-6
6-7

Columna 1 Columna 2 Columna 3


DQO
Flujo
6V
3 1
mgL1
mh
10
300
10
15
275
25
35
200
60
75
170
135
170
90
305
140
100
445
30
210
475
25
250
500
20
250
520
15
300
535
10
340
545
5
360
550

Columna 4
Volumen
contenido, m3

29,17
153,34
247,51
231,68
210,85
185,02
154,19
118,36
77,53

Fig. 2.6. Clculo del volumen del tanque compensador.


El volumen del tanque compensador se calcula midiendo la diferencia de
volumen entre los valores del mximo y el mnimo de la curva, con respecto a la
recta trazada.
V = 231 m3
52

Pretratamiento de Aguas Residuales

Para estimar el volumen contenido en el tanque en cada intervalo de tiempo,


se comienza a calcular a partir de cualquiera de los dos momentos en los que a
priori se conoce el volumen contenido, esto es 10:00 a.m. 2:00 p.m.
Tomando como valor inicial el correspondiente a las 10:00 a.m.,
A las 10:00 a.m., V 0
En el intervalo de 10:00 a 11:00 entran al compensador 75 m 3, y salen
45,83 m3, por lo tanto, el volumen a las 11:00 a.m. ser,
V11 = V 10 + V 10-11 - Vpromedio
V11 = 0 + 75 - 45,83 = 29,17
El volumen contenido a las 12:00 m se estima:
V12 = V 11 + V 11-12 - Vpromedio
V12 = 29,17 + 170 - 45,83 = 153,34
De manera similar se procede con los intervalos siguientes. Los valores as
obtenidos se reportan en la columna 4 de la tabla 2.5.
Como se aprecia en la tabla 2.5, a las 7:00 p.m., hora en que cesa la actividad laboral, el tanque contiene 77,53 m3. Si este volumen se mantiene en el
tanque hasta que se inicia el periodo laboral al da siguiente, se tendr que:
El volumen en el tanque de 7:00 a.m. a 8:00 a.m. ser: 41,7
8:00 a.m. a 9:00 a.m.:
10,87
9:00 a.m. a 10:00 a.m.:
0,04
Este ltimo valor corrobora la estimacin inicial que se hizo al considerar
que a las 10:00 a.m. el tanque estar vaco.

2.2. SEDIMENTACIN
La sedimentacin es un proceso fsico de uso muy difundido en el tratamiento de las aguas r esiduales. Puede utilizarse como p arte de un
tratamiento primario, o como una etapa del tratamiento secundario. El objetivo de la sedimentacin primaria es reducir la concentracin de slidos suspendidos y la carga orgnica de un agua residual para facilitar los tratamientos
posteriores. La secundaria constituye un proceso importante para la clarificacin del efluente de las unidades de tratamiento biolgico y para el
espesamiento de los lodos orgnicos que se obtienen.
La sedimentacin es efectiva para la remocin de slidos suspendidos en la
medida en que la fuerza de gravedad que acta sobre la partcula que se desea
53

Menndez Gutirrez, C. y J. Prez Olmo

remover es mayor que la resultante de las fuerzas de sustentacin y la viscosa


que actan sobre ella.

2.2.1. Tipos de sedimentacin


Atendiendo a la naturaleza de los slidos suspendidos y a su concentracin,
la sedimentacin puede clasificarse en:

Discreta,
floculenta,
retardada o por zonas y
por compresin.

2.22. Sedimentacin discreta


En este tipo de sedimentacin, caracterstica de las suspensiones de arena,
cenizas y de carbn mineral entre otros, las partculas conservan su individualidad y no cambian de tamao, forma ni densidad, durante el proceso de sedimentacin. La velocidad de sedimentacin solo depende de las propiedades del fluido
y de las partculas que se desean remover, y es constante durante todo el proceso de sedimentacin.
La fuerza resultante que acta sobre una partcula que se mueve a travs de
un lquido en reposo puede calcularse:
(2.9)
Fr = Fe)s)d
donde:
Fr: Fuerza resultante.
Fe: Fuerza externa que propicia que la partcula se mueva hacia abajo.
Fs: Fuerza de sustentacin hidrulica.
Fd: Fuerza de arrastre.
La fuerza externa, Fe, puede estimarse segn:
(2.10)
Fe = m g
donde:
Fe: Fuerza externa que acta sobre la partcula, N.
m: Masa de la partcula, kg.
g: Aceleracin de la gravedad, m s.
La fuerza de sustentacin hidrulica puede ser calculada segn:

Fs

mg
s
54

(2.11)

Pretratamiento de Aguas Residuales

Fig. 2.7. Esquema de sedimentadores: clarificadores, a) Rectangular b) y


c) Circular.
55

Menndez Gutirrez, C. y J. Prez Olmo

U : Densidad del fluido, kg m.


U s: Densidad de la partcula, kg m.
Por otra parte, la fuerza de arrastre es funcin de la forma, tamao y rugosidad de la partcula, as como de su velocidad de sedimentacin y de la densidad y viscosidad del fluido:
CD A U 2
2

Fd

(2.12)

donde:
CD: Coeficiente de arrastre (adimensional). Es funcin del nmero de
Reynolds para un tipo de partcula.
A: rea proyectada sobre un plano perpendicular a la direccin del movimiento, m2.
U: Velocidad de la partcula, m s.
De las ecuaciones (2.10), (2.11) y (2.12) puede obtenerse.
m

dU
dt

mg

s  C D A U 2

s
2

(2.13)

En la ecuacin 2.13 se ha expresado la fuerza resultante en funcin de la

dU
dt
La aceleracin de la partcula en su movimiento de descenso a travs del
seno del lquido puede obtenerse de la ecuacin 2.13:

masa de la partcula y de su aceleracin,

dU
dt

s  C D A U 2

s
2m

(2.14)

Durante el descenso, se llega a una velocidad a partir de la cual la aceleracin de la cada se hace igual a cero, y por tanto, la velocidad de sedimentacin
es constante e igual a:


m
2 g s


s
A CD

(2.15)

para partculas esfricas,


A

d2
4

56

(2.16)

Pretratamiento de Aguas Residuales

Teniendo en cuenta adems que:

masa
volumen

(2.17)

y que el volumen de partculas esfricas viene dado por:

1
d3
6

(2.18)

La ecuacin 2.15, para partculas esfricas se transforma en:

s 
U g
 d
CD

(2.19)

El valor U es la velocidad de sedimentacin m s.


O sea, que la velocidad de sedimentacin de una partcula esfrica aislada
es funcin de su dimetro y densidad, as como de la viscosidad del lquido claro
en el que se encuentra suspendida, ya que, como fue mencionado, el coeficiente
de arrastre depende del nmero de Reynolds. Para emplear la ecuacin (2.19)
se requiere conocer el valor del coeficiente de arrastre CD.

Fig. 2.8. Variacin del coeficiente de arrastre con el NRe.


El rgimen de flujo alrededor de la partcula puede ser laminar o turbulento.
En la regin laminar prevalecen las fuerzas viscosas, mientras que en regiones
turbulentas predominan las fuerzas inerciales. Por tanto, la relacin entre el
coeficiente CD y el nmero de Reynolds depende del rgimen de flujo que prevalezca, segn la figura 2.8.
57

Menndez Gutirrez, C. y J. Prez Olmo

Para partculas esfricas:


CD

24

N Re

3
N Re

 0 ,44

(2.20)

Cuando NRe SUHGRPLQDHOSULPHUWpUPLQRGHODHFXDFLyQ\HVWD


queda:

CD

24
N Re

(2.21)

Para NRe entre 1 000 y 250 000 el valor de CD es relativamente constante e


independiente del nmero de Reynolds (figura 2.6).
(2.22)
CD = 0,4
Para NRe intermedios se cumple:

CD

18 ,5
N Re0,6

(2.23)

El procedimiento normal de diseo de sedimentaciones para partculas discretas consiste en seleccionar una velocidad de sedimentacin U de forma que
todas las partculas con velocidad superior o igual a U son removidas. La velocidad de diseo puede calcularse mediante la ecuacin (2.19) o ser obtenida de
pruebas experimentales. Las pruebas de sedimentacin discontinua (a templa)
dan origen a curvas similares a la de la figura 2.9. En tales pruebas la interfase
slido-lquido crece desde el fondo hacia arriba en la medida que mejora la
clarificacin de la suspensin. En el tiempo t, puede suponerse que todas las
partculas que tengan mayor tamao que la caracterizada por una velocidad de

Ho  H
sedimentacin de t  t habrn sedimentado.
o

Fig. 2.9. Variacin de la interfase slido-lquido en la sedimentacin a templa.


58

Pretratamiento de Aguas Residuales

Si la turbidez del lquido sobrenadante o la suspensin contienen cantidades


despreciables de partculas ms pequeas que la caracterizada, el dato obtenido
en la prueba puede ser utilizado en el diseo del sedimentador.
rea del sedimentador
El rea del sedimentador puede calcularse segn:

Q 2
m
U

(2.24)

donde:
A: rea, m2.
Q: Flujo de sobrenadante, m3 s.
U: Velocidad de sedimentacin seleccionada, m s.
Por otro lado, la velocidad de ascenso del lquido claro, tambin llamada
carga superficial es:

Q 3  2 1
m m d
A

(2.25)

De las ecuaciones 2.24 y 2.25 se concluye que en la sedimentacin de partculas discretas el diseo se basa en que la carga superficial V debe ser, tericamente, igual a la velocidad de sedimentacin U seleccionada.
En la discusin anterior se tom en consideracin una velocidad de sedimentacin lmite, y la separacin de la suspensin de aquellas partculas, que
con velocidades iguales o superiores a ellas, caen a travs del lquido iniciando
su descenso a partir de la superficie. Sin embargo, al iniciarse la sedimentacin,
considerando que las partculas estn uniformemente distribuidas en todo el volumen, hay una cierta fraccin de partculas que aunque tienen la velocidad de
sedimentacin menor que U0 se encuentran a una altura tal, que alcanzan el
fondo al mismo tiempo o en un tiempo menor que las partculas seleccionadas
para el criterio de diseo.

Fig. 2.10. Sedimentacin ideal de partculas discretas.


59

Menndez Gutirrez, C. y J. Prez Olmo

Hazen y Camp desarrollaron el concepto de tanque ideal para definir las


relaciones aplicables al diseo de sedimentadores.

Fig. 2.11. Relaciones de sedimentacin de partculas discretas en una


columna tpica.
Suponiendo que las partculas de los distintos tamaos estn distribuidas
uniformemente por toda la profundidad a la entrada del tanque de sedimentacin, mediante un anlisis de la trayectoria de la partcula en la figura 2.10,
puede verse que aquellas con velocidad de sedimentacin Up menor a Uo, se
eliminarn en la proporcin:
Xr

Up
Uo

(2.26)

La eficiencia de la sedimentacin, considerando toda la gama de velocidades existentes en una suspensin, puede ser determinada mediante el uso de una
columna de sedimentacin en una prueba a templa.
Procedimiento de diseo
El procedimiento consiste en tomar muestras a distintas alturas a intervalos
regulares de tiempo, y determinar en cada caso la concentracin de slidos
suspendidos de la extraccin.
Para un caudal de clarificacin dado, Q, solo las partculas con velocidad U
mayor a Uo sern totalmente eliminadas. Las restantes partculas sedimentan
Up
. Por tanto, la fraccin total de partculas eliminadas
en la proporcin
Uo
60

Pretratamiento de Aguas Residuales

cuando se selecciona como criterio de diseo la velocidad de sedimentacin


U o ser:
Xr

1  X r  U p
U
0

dX

(2.27)

donde:
Xr): fraccin de partculas con velocidad ! Uo.
Uo: velocidad seleccionada como lmite.
Xr

Up

U
0

fraccin de partculas con velocidad menor que Uo y que son


dX : removidas.

Desde el punto de vista prctico, los sedimentadores estn sujetos a efectos


de turbulencia, cortocircuitos de las corrientes y gradientes de velocidades. Por
tanto, nunca debe disearse considerando que la carga superficial es igual a la
velocidad de sedimentacin seleccionada. Se recomienda que la carga superficial sea menor a la velocidad de sedimentacin en un factor que pueda variar
entre 1,25 y 1,75 y el tiempo de retencin incrementarse de 1,5 a dos veces.
Ejemplo 2.4
Al someter a prueba de sedimentacin una suspensin con baja concentracin de slidos se obtuvieron los datos de la tabla 2.6.
Tabla 2.6. Datos de sedimentacin obtenidos de una suspensin con baja
concentracin de slidos
Tiempo requerido para
sedimentar 1,5 m (min)
Fraccin de partculas
que no sedimentaron en
el tiempo indicado

70

120 230 460 630 880

0,46 0,40 0,28 0,12 0,06 0,02

Determine el porcentaje de slidos que se eliminarn por sedimentacin si


se utiliza una carga superficial de 17,28 m3 m d en el sedimentador.
Por criterio se seleccionar una velocidad de sedimentacin 1,5 veces mayor que la carga superficial:
Uo = 0,3 10 m s = 0,03 cms
De la tabla que se brinda a continuacin se puede calcular en cada caso la
velocidad de sedimentacin experimental.
61

Menndez Gutirrez, C. y J. Prez Olmo

Tabla 2.7. Porcentaje de partculas y su velocidad de sedimentacin

0,0357 0,0208 0,0109 0,0054 0,0040 0,0026


V
(cm s-1)
f
46
40
28
12
6
2
V: velocidad de sedimentacin (cm s).
f: porcentaje de partculas con velocidad de sedimentacin menor o igual a
la indicada.
Con los datos de la tabla anterior se grafica el porcentaje de partculas contra la velocidad de sedimentacin (figura 2.12).

Fig. 2.12. Porcentaje de partculas vs velocidad de sedimentacin para


el ejemplo 2.4.

1  X r  1
Uo

Xr

dX

Uo = 0,03 cms
De la figura, Xr = 45
Xr

dX

A1  A2  A15

100  45 

1
0,4735
0 ,03

X = 70,78 %
62

Pretratamiento de Aguas Residuales

2.2.3. Sedimentacin de partculas floculentas


Los slidos suspendidos que generalmente se encuentran presentes en las
aguas residuales tanto de origen domstico como industrial son ejemplos tpicos
de partculas floculentas. Tambin presentan caractersticas floculentas los
efluentes de los tratamientos biolgicos que poseen relativamente una baja concentracin de slidos suspendidos.
Cuando la floculacin ocurre, la velocidad de sedimentacin de las partculas se incrementa en la medida que van sedimentando, como consecuencia de la
asociacin con otras partculas individuales o conglomerados de partculas. De
tal forma, la velocidad de sedimentacin del flculo que se forma es mayor que
la de las partculas individuales que lo constituyen, producindose una trayectoria curvilnea como se ilustra en la figura 2.13.

Fig. 2.13. Sedimentacin ideal de partculas floculentas.


Cuando est presente la floculacin, en el diseo del sedimentador debe
tenerse presente no solo la carga superficial, sino adems el tiempo de retencin, ya que a medida que este ltimo se incrementa, aumenta la formacin de
flculos y esto incide en la velocidad de sedimentacin.
Teniendo en cuenta la influencia del tiempo de retencin en la formacin de
los flculos, se dificulta un anlisis matemtico de la situacin, y por tanto, se
hace necesario acudir a las pruebas de laboratorio para la estimacin de los
parmetros que se requieren en el diseo de los sedimentadores.
Procedimiento de diseo
Las pruebas de sedimentacin se efectan mediante un procedimiento similar al empleado en los experimentos de sedimentacin con partculas discretas.
63

Menndez Gutirrez, C. y J. Prez Olmo

En este caso las curvas de profundidad contra tiempo a las distintas concentraciones tienen la forma que se muestra en la figura 2.14.

Fig. 2.14. Relaciones de sedimentacin de partculas floculentas. Curvas


de isoconcentracin.
Las curvas representan el porcentaje de remocin a cada profundidad y
tiempo, e indican la mxima trayectoria de la senda de la sedimentacin para
cada concentracin en especfico, por ejemplo, 45 % de las partculas en suspensin sedimentan a una velocidad promedio superior a 1,8 / 24 = 0,075 m /
min. Adicionalmente, hay otras fracciones de partculas que aunque tienen
velocidades inferiores a esta tambin sedimentarn.
La fraccin de partculas que aun teniendo una velocidad de sedimentacin
Up menor a la fijada U0 sedimenta, viene dada segn la ecuacin (2.27).
Xr

Up
U0

(2.27)

Como se trabaja a un tiempo dado, esta relacin de velocidades se convierte


en una relacin de profundidades,
Xr

hp
h0

(2.28)

Considerando la figura 2.12 y suponiendo que como promedio 87,5 % de las

h1
100  75
sedimentan a una velocidad igual a
partculas
h0 , 67,5 %

2
64

Pretratamiento de Aguas Residuales

h2
75  60
tiene una velocidad de sedimentacin igual a

h0 , y as sucesiva 2
mente, el porcentaje de partculas que sedimenta en veinticuatro minutos en un
sedimentador de 1,8 m de profundidad ser:

R %

h1 R1  R2 h 2 R2  R3 h3 R3  R4


h0
h0
h0
2
2
2

donde:
R1 = 100 %
R2 = 75 %

(2.29)

R3 = 60 %
R4 = 45 %

El procedimiento puede repetirse para distintos tiempos y obtener una relacin entre la velocidad de sedimentacin y el porcentaje de remocin.
Una vez que se llega al porcentaje de remocin de SS deseado, se estima la
carga superficial teniendo en cuenta que esta debe fijarse entre 1,25 y 1,75
veces menor que la velocidad de sedimentacin. As mismo, el tiempo de diseo
debe ser entre 1,5 y 2,0 veces mayor que el trabajado. De esta manera se
calcula el rea del sedimentador y el tiempo de retencin.
Ejemplo 2.5
Una suspensin floculenta que contiene 450 mgL de slidos suspendidos
llena una columna de 3,0 m con salidas laterales cada 50 cm. A distintos tiempos
se toman muestras y se determina la concentracin de slidos suspendidos. Los
resultados obtenidos se muestran en la tabla que se adjunta. Determine el porcentaje de remocin total de slidos suspendidos que se obtendr utilizando un
sedimentador de 2,75 m de profundidad trabajando con una carga superficial de
25 m3 m d.
Tabla 2.8. Porcentaje de remocin de SS a diferentes profundidades
Tiempo (min)
Profundidad (m)
0,5
1,0
1,5
2,0
2,5
3,0

30
270
369
405
427
436
441

60
90
120
150
Concentracin de SS (mg L-1 )
112
45
27
9
207
103
54
22
315
157
126
58
360
270
139
94
387
315
202
135
405
337
256
166

65

180
9
9
18
58
94
112

Menndez Gutirrez, C. y J. Prez Olmo

A partir de los datos anteriores se calcula, para cada tiempo y profundid ad, el po r cen taje de r emoci n de S S y se tr azan las cu r v as d e
isoconcentracin segn la figura 2.16. para mayor facilidad del trazado puede graficarse previamente el porcentaje de remocin contra tiempo para
cada profundidad (figura 2.15).

Fig. 2.15. Grfico auxiliar para el trazado de las curvas de la figura 2.16.

Fig. 2.16. Relaciones de sedimentacin. Curvas de iso-remocin. Ejemplo 2.5.


La carga superficial de diseo es 25 m3md. Aplicando el factor 1,4, la
velocidad de sedimentacin lmite seleccionada ser:
U = 1,4 25 = 23 md
= 0,024 mmin
66

Pretratamiento de Aguas Residuales

Para una profundidad de 2,75 m el tiempo correspondiente ser 111 min.


R %

0,75 190 0,5 170 0,6 150 0 ,4 130

3 2 3 2 3 2 3 2


0,5 100

78,29 %
3 2

2.2.4. Sedimentacin por zona


La sedimentacin retardada o por zona se presenta en suspensiones de concentracin intermedia. Son caractersticas de este tipo de suspensin los flculos
de aluminio de concentracin comprendida entre 500 y 1000 mgL y los efluentes
de los tanques de aeracin de los lodos activados.
En este tipo de sedimentacin las interacciones entre partculas son lo suficientemente grandes como para que cada una mantenga una posicin fija con
respecto a las restantes durante la cada.
En estos casos se aprecia una interfase generalmente muy bien definida
entre la masa de slidos que sedimentan y el lquido claro sobrenadante, cuando
se realizan pruebas de probeta.4,5
La velocidad de sedimentacin se define como la velocidad con que desciende la interfase slido-lquido. La velocidad de sedimentacin disminuye con
el incremento de la concentracin inicial aun cuando se trate de la misma suspensin (figuras 2.17 y 2.18).

Fig. 2.17. Velocidad de sedimentacin a diferentes concentraciones de


slidos en suspensin.
67

Menndez Gutirrez, C. y J. Prez Olmo

La pendiente del tramo de velocidad de sedimentacin constante en la curva


de la figura 2.17 es la velocidad de sedimentacin a la concentracin inicial.

Fig. 2.18. Influencia de la concentracin de slidos suspendidos en la


velocidad de sedimentacin.
rea del sedimentador.
Varios son los mtodos que se utilizan para determinar el rea de sedimentacin necesaria para la remocin de partculas que tienen el comportamiento
antes descrito.
El mtodo que a continuacin se describe, conocido como mtodo de la
curva de flux, es aplicable, tanto para la sedimentacin por zona como para
floculenta y la de compresin. El fundamento del mtodo es el anlisis del avance del flujo de masa de los slidos en toda la profundidad del sedimentador.
En un sedimentador continuo los slidos que entran sedimentan por la accin de dos componentes del flujo:
Descenso de los slidos debido a la sedimentacin por gravedad.
Descenso de los slidos como consecuencia de la extraccin de fondo del
sedimentador.
De acuerdo con lo anterior, el flujo msico de slidos por unidad de rea,
llamado de ahora en adelante flujo msico, ser:
(2.30)
Gt = Gs + Gb
donde:
Gt: flujo msico total, kg m h.
Gs: flujo msico debido a la gravedad.
Gb: flujo msico debido a la extraccin de fondo.
68

Pretratamiento de Aguas Residuales

El flujo msico por gravedad viene dado por:


(2.31)
Gs = U X
donde:
X: concentracin de slidos suspendidos, kg m.
U: velocidad de sedimentacin de los slidos de concentracin X mh .
Mediante experiencias en probeta, se puede obtener Gs a distintas concentraciones iniciales de la suspensin. Una curva tpica de flujo debido a la gravedad, se muestra en la figura 2.19.
En la curva de Gs contra X se observa que tanto para bajos como muy altos
valores de concentracin de slidos suspendidos, Gs es pequeo. Para concentraciones intermedias (3-5 kgm), el flujo de slidos debido a la gravedad, alcanza su valor mximo.
El flujo de slidos debido al caudal de fondo es:
(2.32)
Gb = Uu X, kg m h

Uu: Velocidad de extraccin de fondo, m h .
Si se considera el caudal de extraccin Qu en lugar de la velocidad de la
extraccin, la ecuacin 2.32 se transforma en:

Gb

Qu
X , kg m  2h 1
A

(2.33)

donde:
Qu: Caudal extrado por el fondo, m3 h.
A: rea de sedimentacin, m2.

Fig. 2.19. Flujo msico debido a la


gravedad.

69

Fig. 2.20. Flujo msico debido


a la extraccin de fondo.

Menndez Gutirrez, C. y J. Prez Olmo

Para una extraccin de fondo constante, el flujo de slidos debido a ella


es directamente proporcional a la concentracin de slidos suspendidos (figura 2.20).
Teniendo en cuenta la ecuacin 2.30, para unas condiciones dadas de extraccin de fondo, pueden sumarse grficamente los dos componentes del flujo
msico (Gs + Gb) (figura 2.21).
Para cualquier valor Q u que se seleccione, la curva de G t alcanza un
valor mnimo para cierto valor de concentracin de slidos. En la figura
2.21, la curva de flujo total, G t alcanza el valor mnimo G 1 para la concentracin de X 1.
Este mnimo limita la velocidad a la cual los slidos pueden alcanzar el
fondo del sedimentador, entonces, para asegurar que todo el slido alcance
el fondo, la masa de slido aplicada por unidad de rea en el tiempo unitario,
o sea, G aplicado, debe ser menor o igual a G 1:
(2.34)
Gap G1
Si por cualquier razn la cantidad de slidos que se alimentan al
sedimentador es mayor que el valor lmite G 1, estos se acumularn en el
fondo del sedimentador, producindose un aumento paulatino de su nivel en
el fondo, que puede incluso llevar a la salida de slidos por el sobrenadante.

Fig. 2.21. Flujo msico total.


La concentracin del caudal de extraccin de fondo se obtiene trazando una
recta paralela al eje x y que pasando por el mnimo de la curva de flujo, corte a
la recta Gb. Para la figura 2.21 esta concentracin es Xr.
70

Pretratamiento de Aguas Residuales

Una vez determinado el valor lmite del flujo de slidos G1, puede obtenerse
por un balance de masa, el rea necesaria para la sedimentacin:

Qt
X
G1 0

(2.35)

donde:
Qt: Flujo total que entra al sedimentador, m3d.
X0: Concentracin de SS que entra al sedimentador, gm.
De la figura 2.21 puede deducirse que para una suspensin dada, el rea
de sedimentacin, y por tanto, la concentracin de los slidos de fondo se
modifica al cambiar el caudal de extraccin de fondo. Debe tenerse presente que cada vez que se cambie el caudal del fondo, se obtendr un G b diferente y se requiere, por tanto, trazar de nuevo la curva de flujo msico total.
Como se aprecia, el procedimiento anterior es tedioso porque para cada
Q u se necesita construir nuevas curvas de flujo. Este procedimiento grfico
se simplifica notablemente cuando se utiliza el concepto de punto de situacin (figura 2.22).
La concentracin de slidos a la entrada, X 0, se representa por una lnea
vertical en el valor especificando X 0. La velocidad de flujo de sobrenadante,
U 0, viene dada por una lnea denominada lnea del sobrenadante, y que partiendo del origen tiene una pendiente igual a U 0. El intercepto de la lnea de
sobrenadante con la de operacin (la de X 0), recibe el nombre de punto de
situacin.

Fig. 2.22. Definicin del punto de situacin.


71

Menndez Gutirrez, C. y J. Prez Olmo

La velocidad de extraccin de fondo se representa por una lnea denominada lnea de fondo, con pendiente igual a -Uu y que pasa por el punto de situacin
y es tangente a la curva.
El intercepto de la lnea de fondo con el eje de las concentraciones da el
valor de la concentracin del caudal de fondo Xr, mientras que el intercepto con
el eje de flujo slidos refleja el flujo de slidos aplicado, Gap.
Entre las ventajas que posee este ltimo procedimiento, est el de la posibilidad de analizar en un tiempo relativamente breve distintas variantes para el
clculo del rea del sedimentador. Al mismo tiempo, brinda la facilidad de evaluar el comportamiento de un sedimentador en funcionamiento ante la variacin
de los parmetros de operacin.
Procedimiento para el clculo del rea del sedimentador
Como puede concluirse de lo anteriormente expuesto, el mtodo de diseo
se fundamenta en el conocimiento de las velocidades de sedimentacin a distintas concentraciones de slidos suspendidos. Para esto se requiere de pruebas
relativamente sencillas, a escala de laboratorio. Las pruebas consisten en seguir
en el tiempo, la trayectoria descendente de la interfase slido-lquido.
Esta interfase es fcil de distinguir cuando las suspensiones presentan caractersticas de sedimentacin por zona, o son de carcter floculento y poseen
concentraciones altas o medias. En aquellos casos en que la interfase no se
aprecia de manera definida, puede acudirse al mtodo de diseo que se basa en
las pruebas que se conocen como de tubos largos.
Descripcin del procedimiento
1. Se realizan las pruebas de sedimentacin en probeta, a temperatura constante, a distintas concentraciones de slidos suspendidos (figura 2.17). La pendiente en la regin recta de la curva es la velocidad de sedimentacin. Para
cada una de las pruebas anteriores se determina, por gravimetra, la concentracin de slidos suspendidos.
2. Para mayor facilidad en el trabajo y poder realizar interpolaciones, se recomienda trazar la curva de velocidad de sedimentacin en funcin de la concentracin (figura 2.18).
3. De la informacin que brinda la figura 2.18, pueden obtenerse datos de flujo
msico debido a la gravedad, para distintas concentraciones, Gs = U X.
4. Se traza la curva de flujo de slidos en funcin de la concentracin (figura 2.19).
5. Haciendo uso de la curva obtenida en la figura 2.19, y partiendo de la concentracin de fondo deseada, Xr se traza la lnea de fondo tangente a la
curva de flujo (figura 2.22). El intercepto con el eje Y da el flujo de slidos
lmite (Gap) .
72

Pretratamiento de Aguas Residuales

6. Haciendo uso de la ecuacin (2.35), puede calcularse el rea de sedimentacin.


A

Qt
X
Gap 0

A: rea del sedimentador, m2.


Qt: Flujo total a la entrada del sedimentador, m3 h.
X0: Concentracin de slidos suspendidos en la corriente de entrada, kgm.
Gap: Flujo de slidos aplicado, kg m h.
Velocidad de flujo del sobrenadante U0. En la figura 2.22 se traza la lnea de
operacin definiendo el punto de situacin. La pendiente de la recta que une el
punto de situacin con el origen (recta que define la lnea del sobrenadante), da
el valor de la velocidad de flujo del sobrenadante.
Ejemplo 2.6
Se determinaron las caractersticas de sedimentacin de una suspensin
que procede de un lodo activado. Las pruebas de laboratorio se realizaron para
un intervalo de concentracin entre 1000 y 10000 mgL, obtenindose la curva
de la figura 2.23.

Fig. 2.23. Clculo del rea de un sedimentador. Ejemplo 2.6.


73

Menndez Gutirrez, C. y J. Prez Olmo

Calcule la velocidad de flujo de sobrenadante mxima permisible si se desea


obtener una concentracin de slidos suspendidos en el fondo de 11 g L y la de
entrada es de 3000 mgL.
En la curva de la figura 2.23 se traza la lnea de fondo a partir de X = 11 g L,
y que sea tangente a la curva. El intercepto en el eje de ordenadas da el flujo de
slidos lmites, que ser el aplicado, Gap = 7,3 kg2 h.
El rea del sedimentador, en funcin del flujo de entrada ser:
A

Qt
3 0 ,41 Qt m 2
7,3

en que Qt est dado en m3 h


para X0 = 3 gL se traza la lnea de operacin que intercepta a la lnea de fondo
en el punto P.
La velocidad de flujo de sobrenadante es la pendiente de la lnea OP
(figura 2.23).
U0 = 1,82 m3 m h
La velocidad del flujo del fondo ser:
Uu

Gap
Xr

0 ,68 m3 m 2 h 1

2.2.5. Evaluacin de la operacin de sedimentadores


El punto de situacin define el estado de operacin del sedimentador, y
los cambios de las condiciones de trabajo se reflejan en la curva de flujo de
la manera que se ilustra en las figuras 2.24 a 2.26.
1. Una disminucin del caudal de extraccin del fondo hace rotar, en el
sentido contrario a las manecillas del reloj, la lnea de fondo (figura 2.24),
disminuyendo el flujo de slidos a aplicar, G ap , y aumentando la concentracin de slidos en la extraccin del fondo, X r.
2. Un aumento en el flujo de extraccin del sobrenadante, hace rotar, en el
sentido contrario a las manecillas del reloj, la lnea del sobrenadante y
mueve el punto de situacin hacia arriba (figura 2.25), mientras que la
pendiente de la lnea de fondo se mantiene constante.
3. Un aumento en la concentracin de los slidos que entran al sedimentador,
X0, mueve la lnea de operacin hacia la derecha y el punto de situacin se
mueve a lo largo de la lnea del sobrenadante. La lnea de fondo pasa por el
nuevo punto de situacin (figura 2.26).
74

Pretratamiento de Aguas Residuales

Fig. 2.24. Variacin del flujo de salida del fondo.

Fig. 2.25. Variacin del flujo del sobrenadante.

Fig. 2.26. Aumento de la concentracin de slidos a la entrada.


75

Menndez Gutirrez, C. y J. Prez Olmo

De las situaciones anteriores es necesario destacar un aspecto importante relativo a la lnea de fondo y su posicin en la curva de flujo.6 Cuando la
lnea de fondo:
1. Corta a la curva de flujo en un solo punto, se obtienen condiciones seguras de
trabajo.
2. Corta a la curva en un punto y es tangente a la misma en otro, prevalecen las
condiciones lmites de operacin.
3. Corta a la curva de flujo en dos puntos, existen condiciones de sobrecarga en
la operacin.
Otra informacin que puede obtenerse sobre la operacin del sedimentador al analizar las curvas anteriores es el relativo a los criterios de clarificacin y espesamiento:
1. Cuando el punto de situacin queda ubicado por debajo de la curva de flujo,
la clarificacin del lquido sobrenadante est garantizada, y son las condiciones de espesamiento las que controlan el proceso.
2. Cuando el punto de situacin est en la curva de flujo prevalecen las condiciones crticas para la clarificacin.
3. Cuando el punto de situacin est ubicado por encima de la curva de flujo,
tanto la clarificacin como el espesamiento son ineficientes.

Notas bibliogrficas
1 ECKENFELDER, W. W.: Principles of Water Quality Management, CBI, Publishing Co. EE.UU., 1980.
2 BENEFIELD, L. AND C. RANDALL: Biological Process Design for Wastewater
Treatment, prentice Hall Series in Environmental Sciences, EE. UU., 1980.
3 PATTERSON, J.W. AND J.P. MENEZ'HVLJQRIHTXDOL]DWLRQEDVLQV$PInst.
Chem. Engrs. Env. Prog., vol. 3, no. 1, 1984.
4 ROBIND:+7KH7KHRU\RIWKH'HVLJQDQG2SHUDWLRQRI6HWWLQJ7DQNV
Transaction Institute of Chemical Engineering, vol. 42, no. 158, 1964.
5 EKAMA, G. A.; Et al: Secondary Settling Tanks. Theory, Design and Operation, Sudafrica, 1984.
6 RITTMAN%$HURELF%LRORJLFDO7UHDWPHQWEnvironmental Science Technology, vol. 21, no. 2, 1987.

76

Oxidacin Biolgica. Nitrificacin

CAPTULO 3

OXIDACIN BIOLGICA NITRIFICACIN

3.1. PRINCIPIOS DE LA OXIDACIN BIOLGICA


Los procesos biolgicos, cualquiera que sea su nivel, implican un intercambio continuo de sustancia y de energa.
Los elementos ms importantes que intervienen en el intercambio son el
hidrgeno, oxgeno, carbono, nitrgeno, fsforo y azufre. Otros muchos elementos tambin participan, pero en menor grado.
La energa para estos procesos puede obtenerse de tres fuentes que a su
vez sirven de criterio para clasificar los organismos que participan en cada caso:
Radiacin solar.
Compuestos orgnicos.
Compuestos inorgnicos.
En la biosfera la fuente fundamental de energa es la radiacin solar.
Los organismos fotoauttrofos fijan una fraccin pequea de la energa solar mediante la formacin de compuestos orgnicos de alto contenido energtico (CHO) y produciendo oxgeno. El hidrgeno lo obtienen del agua, el
carbono del CO2 y el fsforo, nitrgeno y azufre de las sales disueltas. La
disponibilidad de P y N generalmente es limitada, ya que el primero no es
fcil de encontrar en forma disuelta y la forma en que el segundo es asimilado ms fcilmente es como NH4 + y NO3 . Las restricciones en la disponibilidad de estos dos elementos, N y P, constituyen usualmente un factor
limitante para la vida de los organismos auttrofos.
Los compuestos de alto contenido de energa sintetizados por los organismos auttrofos constituyen la fuente bsica de energa de los
hetertrofos para a su vez sintetizar molculas ms complejas, constituyentes de la masa celular, incluyendo las protenas. Este proceso no es
muy eficiente desde el punto de vista energtico, y solo una fraccin de la
energa disponible aparece incorporada a la biomasa producida, el resto se
pierde en forma de calor.
77

Menndez Gutirrez, C. y J. Prez Olmo

Fig. 3.1. Cadena alimentaria. Relacin presa-depredador.


Esta biomasa a su vez sirve como fuente de nutriente y energa (presa) a
otros organismos que se alimentan de ella (depredador), y esta ltima es presa
de otro depredador. Cada transformacin presa-depredador va acompaada de
intercambio y prdida de la energa, aquella que se gener en la primera accin
de los organismos fotoauttrofos. Cuando las transformaciones son tales que la
energa contenida en el compuesto orgnico va reducindose, la vida de los
organismos hetertrofos tambin se reduce y se transforma en otras formas de
energa.
En el tratamiento biolgico de aguas residuales no se permite que la cadena
de vida descrita anteriormente se desarrolle hasta sus ltimas consecuencias,
sino que llegado a un punto, la biomasa portadora de la energa es separada
diariamente por procedimientos fsicos.
Los organismos quimioauttrofos obtienen la energa de la oxidacin de los
compuestos inorgnicos. En el campo del tratamiento de las aguas residuales
cobran especial importancia dentro de este grupo las bacterias nitrificantes. Estos
organismos son estrictamente aerobios y obtienen la energa de su accin oxidante
sobre el NH4+ y el NO2.
En el tratamiento de las aguas residuales las bacterias nitrificantes constituyen un vnculo muy importante en la eliminacin del nitrgeno del agua al
convertir el N-NH3 a N-NO 3 y posteriormente, por medios adecuados,
inducir a los organismos hetertrofos a utilizar el NO 3 en su metabolismo
de material carbnico. En esta accin el NO 3 es reducido a N2 que escapa
como gas.

3.2. METABOLISMO
Los microorganismos requieren de una fuente de energa y carbono para la
sntesis de nuevas clulas as como de nutrientes inorgnicos para sus diferentes funciones.
El metabolismo es la combinacin de un conjunto de transformaciones
bioqumicas necesarias para la sntesis celular (anabolismo) y la produccin de
energa (catabolismo) que ocurren a travs de procesos enzimticos. De hecho,
78

Oxidacin Biolgica. Nitrificacin

los diferentes microorganismos pueden ser clasificados a partir de la forma en


que obtienen el carbono y la energa.
En el tratamiento biolgico de aguas residuales la energa est presente
inicialmente en los compuestos orgnicos carbonosos y nitrosos. Esta se convierte en otras formas con las consecuentes prdidas de energa. La energa
carbonosa es asimilada por los organismos hetertrofos, quienes descomponen
los compuestos proteicos en sus constituyentes carbonosos y nitrosos. El amonio
obtenido es utilizado a su vez como fuente de energa por las bacterias nitrificantes.

3.2.1. Mecanismo para la obtencin de energa


por los organismos
La reaccin global involucrada en el metabolismo de los microorganismos
hetertrofos para la obtencin de energa puede esquematizarse:
Compuestos
microorgan ismos
Nuevas
orgnicos  O 2 o CO 2  H 2O  clulas  Energa

En detalle lo que ocurre es que las sustancias orgnicas son descompuestas


por la accin de los microorganismos dando lugar a la liberacin de H+, CO2 y
electrones. Se dice, por tanto, que la sustancia se oxida.
Los electrones (e) y el H+ liberados son transferidos a otra sustancia que,
por tanto, es la aceptora de e. Las reacciones de oxidacin-reduccin contemplan la transferencia de electrones de una sustancia reducida (donador de e) a
una sustancia oxidada (consumidor de e). El sustrato o alimento es la sustancia
donante.
El metabolismo hetertrofo utiliza los compuestos orgnicos como donantes de e . En el auttrofo son los compuestos inorgnicos los donantes.
Por otro lado, el aceptor de electrones depende de que el proceso ocurra en
un medio aerobio, anxico o anaerobio. Mientras que el oxgeno es el aceptor
ltimo o terminal de la cadena oxidacin-reduccin en un medio aerobio, el
par nitrito-nitrato es el aceptor en medio anxico y el CO 2 y SO4  lo son en
el anaerobio.
La cantidad de energa liberada durante el proceso de oxidacin reduccin
depende tanto del donante como del aceptor de electrones. Cuando el donante
es carbonoso y el aceptor final es el oxgeno, la energa liberada es relativamente alta y 5 % mayor que cuando el aceptor1 es el NO2 o NO3 . La menor
cantidad de energa liberada corresponde al CO2 y SO4 como aceptores de
electrones.
Una fraccin de la energa liberada durante los procesos descritos anteriormente es utilizada para la sntesis de nuevas clulas, y el resto est destinado a
79

Menndez Gutirrez, C. y J. Prez Olmo

satisfacer los requerimientos de la supervivencia y el desarrollo de la propia


clula (figura 3.2).
Cuando la materia orgnica es completamente oxidada a CO2 y el O2 es el
aceptor de e, la masa de oxgeno consumida da la medida estequiomtrica de
los moles de e transferidos. De aqu, la capacidad de cualquier sustrato como
donante de e puede ser medida en trminos de oxgeno requerido para oxidar la
materia orgnica a CO2, este es el fundamento de la DQO.
Aun cuando no es objetivo de este texto profundizar en las reacciones
metablicas, se ha considerado oportuno esquematizar en la figura 3.3 las sucesivas etapas de oxidacin reduccin que ocurren desde que la materia orgnica
es inicialmente oxidada hasta que los electrones son transferidos hacia el ltimo
aceptor.

Fig. 3.2. Energa liberada durante la oxidacin del sustrato y su transformacin.


De acuerdo con el esquema de la figura 3.3, la energa almacenada en la
materia orgnica AH2 se libera durante la oxidacin biolgica por dehidrogenacin
del sustrato mediante la accin del NAD (dinucletido de nicotinamida y de
adenina). El FAD (flavin adenn-dinucletido) es una flavoprotena que adems
de transportar hidrgeno, es un intermediario entre las deshidrogenasas y los
citocromos para el transporte de electrones. Por su parte, los citocromos son
protenas transportadoras de electrones que se encuentran solamente en las
clulas aerobias.
Como se aprecia en el esquema, se han representado tres puntos de produccin de ATP (trifosfato de adenosina) a partir de ADP (difosfato de adenosina).
El ATP es el transportador de energa qumica ms importante de las clulas de
todas las especies vivientes. A medida que el ATP transfiere su energa a otras
80

Oxidacin Biolgica. Nitrificacin

molculas, se transforma en ADP. Este a su vez, acepta energa qumica de las


transformaciones que le preceden y se transforma nuevamente en ATP, ya sea
a expensas de organismos auttrofos o hetertrofos.

Fig. 3.3. Esquema de los procesos redox hasta la transferencia de electrones al ltimo aceptor. Metabolismo aerobio, anaerobio y anxico.
De esta manera, las reacciones con el ltimo aceptor y la energa asociada
a ellas pueden resumirse como se muestra a continuacin:
(aerobio)

AH2 + O2

CO2 + H2O + E1

(3.2)

(anxico)

AH2 + NO3

N2 + H2O + E2

(3.3)

AH2 + SO4

H2S + H2O + E3

(3.4)

AH2 + SO4

H2S + H2O + E4

(3.4)

AH2 + CO2

CH4 + H2O + E5

(3.5)

AH2 + B

BH2 + A + E6

(3.6)

(anaerobio)

81

Menndez Gutirrez, C. y J. Prez Olmo

donde:
E1 > E2 > E3 > E4 > E5 > E6
puede apreciarse adems que la reduccin del sulfato ocurre tanto en los
procesos anaerobios como en los anxicos, aunque en los segundos se reducen preferentemente los nitratos antes que los sulfatos.
El ATP y la actividad dehidrogenasa son los mejores indicadores de la biomasa
activa en los procesos de tratamiento biolgico.2

3.3. BIODEGRADABILIDAD DE LAS AGUAS RESIDUALES


La purificacin biolgica puede ser aplicada efectivamente solo en aquellos
casos en los que el agua residual contenga compuestos orgnicos biodegradables.

3.3.1. Condicin de degradacin biolgica de compuestos


orgnicos
Los compuestos orgnicos solubles presentes en aguas residuales pueden
ser clasificados en cuatro grandes grupos de acuerdo con su condicin de degradacin biolgica y toxicidad:
Compuestos biolgicamente degradables y no txicos (sacridos, aminocidos,
cidos grasos).
Compuestos biolgicamente degradables, pero txicos a altas concentraciones (fenol, formaldehdo).
Compuestos no degradables biolgicamente y no txicos (cido hmico, cidos lignosulfnicos, colorantes azo).
Compuestos no degradables biolgicamente y txicos a bajas concentraciones (pesticidas, DDT).
Un agua residual en la que todos sus constituyentes son biodegradables
no txicos o solamente txicos a altas concentraciones tiene una relacin
DBO5 /DQO en el intervalo de 0,55-0,70. Esta relacin es un buen indicador
de la posibilidad del tratamiento biolgico de un agua residual dada. Mientras ms baja es esta relacin, mayor ser la proporcin de compuestos no
degradables en el agua residual. Si la relacin es 0,2 menor, se est en
presencia de un agua residual constituida, fundamentalmente, por compuestos no degradables por medios biolgicos, en este caso, la purificacin biolgica sola no es suficiente.
Las causas por las que un compuesto puede ser no biodegradable son
diversas y no todas conocidas. Los compuestos constituidos por cadenas
82

Oxidacin Biolgica. Nitrificacin

alifticas son degradables, sin embargo, el crecimiento de estas es disperso.


Crecimiento disperso de las cadenas alifticas:
CH2 &+2)XCH3CH3

CH2 &+2)XCH3

CH2

SO3Na

CH2-(CH2)-CH3

SO3Na

Degradable

No degradable

Otra causa de no biodegradabilidad de un compuesto puede encontrarse


en la sustitucin de un anillo aromtico. Mientras que el 2,4-dinitrofenol puede ser degradable biolgicamente, la sustitucin del grupo nitro de la posicin cuatro a la seis hace al compuesto no biodegradable.
Sustituciones en el anillo aromtico
OH

OH
NO2
O2N

NO2

NO2

Degradable

No degradable

En general, los compuestos de alta masa molecular y/o de estructura compleja son no biodegradables o de difcil degradacin. En estos casos, es ms
aconsejable el empleo de mtodos fsico-qumicos de tratamiento.
En la figura 3.4 se muestran los resultados obtenidos al analizar por
cromatografa de gel un agua residual antes y despus de ser sometida a un
tratamiento biolgico.3 En la parte (a) de la figura se aprecian dos picos que
caracterizan a compuestos de alta masa molecular (1) y compuestos de baja
masa molecular (2). En (b), puede observarse que despus de un tratamiento
biolgico, la curva que caracteriza a los compuestos de baja masa molecular ha
desaparecido, pero persiste la de aquellos de mayor masa molecular, y que no
son removidos mediante este tipo de tratamiento.
Los compuestos orgnicos remanentes en un agua residual que ha sido
sometida a un buen tratamiento biolgico (DBO5 soluble  10 mgL ) son
fundamentalmente no biodegradables y fcilmente determinados con la DQO.
Estos compuestos pueden clasificarse en dos subgrupos (figura 3.5). El pri83

Menndez Gutirrez, C. y J. Prez Olmo

mer subgrupo est constituido por la fraccin no biodegradable contenida en


el residual original y que pasa a travs del tratamiento sin experimentar
ningn cambio (DQOND). Al segundo subgrupo pertenecen aquellos compuestos que son producidos por los propios microorganismos que realizan la
accin depuradora, como consecuencia de su metabolismo. Su concentracin vara entre 1 y 5 % de la DQO que es removida.

Fig. 3.4. Curva de elusin de cromatografa de gel. a) Antes del tratamiento biolgico, b) despus del tratamiento biolgico.
84

Oxidacin Biolgica. Nitrificacin

Debe ser destacado que durante el tratamiento biolgico solo se remueven


aquellos compuestos de relativa baja masa molecular.

DQOD + DQOND

(0,01-0,05) DQOD + DQOND

Fig. 3.5. Clasificacin de los compuestos de acuerdo con su biodegradabilidad.

3.3.2. Purificacin biolgica de las aguas residuales


Cuando un agua residual se pone en contacto con los microorganismos presentes en la unidad de tratamiento, los compuestos orgnicos pueden ser removidos a travs de diversos mecanismos.
Los slidos suspendidos y coloidales son removidos por adsorcin y
coagulacin. Si son degradables, son hidrolizados por las exoenzimas. Los
productos de esta hidrlisis son enzimticamente transportados al interior
de las clulas. Si son inertes, pasan a constituir la fraccin inerte de la
biomasa.
Por su parte, los compuestos que aunque solubles son de alta masa molecular
deben tambin ser previamente hidrolizados antes de que puedan transportarse
a travs de la membrana bacteriana.
Los compuestos de baja masa molecular son transportados directamente al
interior de las clulas por las enzimas correspondientes.
Los compuestos orgnicos removidos de las aguas residuales son parcialmente oxidados, y utilizados en parte en la sntesis de constituyentes de
las clulas y en nuevas clulas (figura 3.6).
Los principales productos de la oxidacin son CO2, H2O y NH3. El propsito
fundamental de esta oxidacin es obtener la energa necesaria para las reacciones de sntesis. En principio los productos de la sntesis se almacenan como
85

Menndez Gutirrez, C. y J. Prez Olmo

productos de reserva (polisacridos, lpidos). Con posterioridad son utilizados


para la constitucin de nuevas clulas.

Fig. 3.6. Mecanismo de la oxidacin biolgica.


No obstante que la produccin de biomasa generalmente no es el objetivo
del tratamiento biolgico de los residuales (aunque puede serlo en alguna situacin particular), su formacin en cierta cantidad es inevitable. En procesos
aerobios Mc Carty1 ha obtenido valores de rendimiento (Y) (biomasa producida/DBO removida), que varan entre 0,30 y 0,51 para glucosa, anilina, lactato y
acetato. Por otra parte, Servizi y Bogan4 han presentado valores de rendimiento
GHSDUDXQDYDULHGDGGHFRPSXHVWRVPLHQWUDVTXH(FNHQIHOGHU\2&RQQRU 5
muestran valores de este parmetro entre 0,37 y 0,46 para muchos compuestos
biodegradables.
Para los procesos anaerobios, como la produccin de energa es menor,
tambin es menor la cantidad de esta que se dedica a la produccin de biomasa.
En estos casos, el rendimiento apenas alcanza valores comprendidos entre 0,032
y 0,27.6

3.3.3. Sustrato exgeno y endgeno


Teniendo en cuenta que los compuestos orgnicos contenidos en el agua
residual sirven como fuente de energa a los microorganismos, son denominados
sustratos. Los sustratos se clasifican como exgenos (fuera de la clula
bacteriana) y endgenos (en el interior de la clula bacteriana).
Cuando el sustrato exgeno se agota, los microorganismos oxidan el
sustrato endgeno. En una primera etapa estos sustratos endgenos sirven
como material de reserva y cuando se agota, comienza la oxidacin de las
protenas. Si los microorganismos constituyentes de la biomasa se mantie86

Oxidacin Biolgica. Nitrificacin

nen durante un cierto tiempo sin sustrato exgeno, muchos de ellos morirn.
La masa remanente puede llegar a contener menos del 10 % en peso de
organismos vivos, y ser relativamente estable a la descomposicin posterior. Este fenmeno es utilizado en la prctica para la estabilizacin aerobia
de los lodos.
La oxidacin del sustrato exgeno puede ser descrita segn la ecuacin:

1
1
1
C x H y O z  X  Y  Z O 2 o X CO2  Y H 2O

4
2
2

(3.7)

La oxidacin del sustrato endgeno puede representarse:


C5 H 7 NO 2  5 n O 2 o 5 n CO 2  2 n H 2O  n NH 3

(3.8)

La frmula emprica C 5 H 7 NO 2 representa la composicin media de


la materia voltil celular.7 Esta biomasa contiene 12,3 % de nitrgeno y
tericamente requiere 1,42 g de oxgeno por cada gramo de biomasa
totalmente oxidado. Aunque la frmula emprica anterior, de Hoover y
Porges, es la ms difundida, hay muchas otras que tambin se utilizan. No
obstante en la prctica el equivalente de oxgeno de biomasa voltil vara en
el intervalo comprendido entre 1,3 y 1,5 gg .

3.4. CINTICA DEL CRECIMIENTO BIOLGICO


Y REMOCIN DE SUSTRATO
3.4.1. Crecimiento y multiplicacin de microorganismos
Cultivo discontinuo de poblaciones puras de microorganismos
Cuando los microorganismos se ponen en contacto con un sustrato al cual
estn bien adaptados, crecen y se multiplican.
Si se parte de presuponer que para el tiempo inicial t = 0 la concentracin de microorganismos es X0, despus de un cierto tiempo la concentracin
inicial se duplica. El tiempo de duplicacin, tg, es llamado tiempo medio de
generacin. Despus de transcurrido el tiempo de generacin t = tg, los
microorganismos se duplican nuevamente. Esto puede demostrarse mediante
el esquema siguiente:
t=0
tg
2tg
3tg
n tg

X0
2X0
4X0
8 X0

87

21 X0
22 X0
23 X0
2n X0

Menndez Gutirrez, C. y J. Prez Olmo

As para el tiempo t = ntg la concentracin de microorganismos ser:

X t 2n X 0 2
Aplicando logaritmos a la ecuacin 3.9:

tg

X0

t
ln 2  ln X 0
tg

ln X t

(3.9)

(3.10)

Escribiendo:

0,69
tg

ln 2
tg

(3.11)

Sustituyendo en la ecuacin 3.10 se obtiene,


ln X t

W  ln X 0

(3.12)

La ecuacin 3.12 tambin puede escribirse:

Xt

X0 e t

(3.13)

Conociendo el valor de P se puede, mediante la ecuacin 3.13, calcular el


valor de la concentracin de microorganismos a cualquier tiempo t.
Diferenciando con respecto al tiempo la ecuacin 3.13, se obtiene la ecuacin cintica general de crecimiento.

dX
dt

1 dX
X dt

(3.14)

El trmino P es denominado velocidad especfica de crecimiento. A partir de


la ecuacin 3.14 puede concluirse que P solo depende del tiempo de generacin,
tg, de un microorganismo dado. Sin embargo, esta conclusin es vlida
solamente para altas concentraciones de sustrato.
Para bajas concentraciones de sustrato, P se hace dependiente de la misma
de acuerdo con la bien conocida ecuacin de Jacques Monod:

mx

S
KS  S

donde:
S: concentracin de sustrato limitante (mgL).
Pmx: Velocidad especfica de crecimiento mxima h.
KS: Constante de saturacin (mgL).
KS: VS = S para P = P mx
88

(3.15)

Oxidacin Biolgica. Nitrificacin

Fig. 3.7. Representacin grfica de la ecuacin de Monod.


Combinando las ecuaciones 3.14 y 3.15, se obtiene que:

dX
dt

mx

S
X
KS  S

(3.16)

Aqu se aprecia que la velocidad de crecimiento de los microorganismos es


funcin de su concentracin y de la concentracin de sustrato.
Cuando el sustrato se encuentra en condiciones lo suficientemente altas
como para que su variacin no afecte la velocidad de crecimiento, la ecuacin
3.16 se transforma:

dX
dt

mx X

(3.17)

O sea, que la velocidad de crecimiento en un instante dado es solo funcin


de la concentracin de microorganismos multiplicado por la velocidad especfica mxima de crecimiento.
Todo el desarrollo anterior fue logrado inicialmente para cultivos puros y
sustratos de un solo componente. Sin embargo, su uso se ha extendido para el
caso de cultivos mixtos y sustratos multicomponentes, que es la situacin que
prevalece durante el tratamiento biolgico de aguas residuales.
Cultivo de poblaciones mixtas de microorganismos
La dinmica poblacional de los cultivos es un proceso complejo y no
totalmente conocida an.
Al aplicar la ecuacin de Monod a cultivos mixtos, debe tenerse en cuenta
que las constantes de crecimiento P mx y KS representan valores promedio de
las constantes de crecimiento de las especies individuales y de su proporcin en
el cultivo.
Los cambios en la poblacin se reflejan en cambios en las constantes de
crecimiento.
89

Menndez Gutirrez, C. y J. Prez Olmo

Fig. 3.8. Relacin entre Pmax y KS para varios sustratos. 1. Glucosa


2. Sorbitol. 3. Sacarosa. 4. Histidina. 5. Galactosa. 6. Lactosa.
En la figura 3.8 se muestra que mientras mayor es el valor de KS mayor es
P mx. Sin embargo, hay compuestos para los que no se ha hallado correlacin.
Tal es el caso, por ejemplo, de los cidos actico y propinico y la fenilalanina.
Por otro lado los cidos glutmico y crico muestran relaciones inversas.

Fig. 3.9. Relacin entre P y la concentracin de sustrato para dos


microorganismos.
A modo de ejemplo de lo anterior puede citarse un caso extremo de un
cultivo formado por dos especies de microorganismos denominados por A y B
(figura 3.9), cuando se utiliza la lactosa como sustrato. La especie A tiene un
valor de P mx = 0,2 h y KS = 1 mgL. Para la especie B estos valores son
respectivamente, 0,5 h y 50 mgL.

90

Oxidacin Biolgica. Nitrificacin

En la figura 3.9 puede apreciarse adems que para bajas concentraciones


de sustrato la velocidad de crecimiento de la especie A es mayor que la de B.
Consecuentemente, la proporcin de A en el cultivo ser mayor que la
de B. Para concentraciones de sustratos cercanas a 35 mgL  (punto de
interseccin de ambas curvas), ambas especies alcanzan la misma velocidad
de crecimiento y su proporcin en el cultivo mixto es la misma. A altas
concentraciones la especie B alcanza mayor velocidad de crecimiento, y
consecuentemente se encontrar en mayor proporcin.
El significado prctico de esta conclusin terica es que queda demostrada una de las formas en las que se puede inhibir o evitar el crecimiento
de algunos microorganismos indeseables en la unidad de tratamiento, por
ejemplo, los filamentosos en los procesos de lodo activado y lagunas
aereadas.

3.4.2. Cintica de la remocin de los compuestos orgnicos


de las aguas residuales mediante cultivos mixtos
Durante aos se han desarrollado diferentes modelos matemticos para
explicar el mecanismo de remocin de la DBO en los procesos de tratamientos biolgicos. 8, 9
Todos estos modelos coinciden en que la velocidad de remocin de DBO
por unidad de biomasa (' DBO/SSV.t) es constante cuando se tienen valores altos de la DBO. Esta velocidad especfica de remocin de la DBO se
mantiene constante hasta que se alcanza un valor de concentracin de DBO,
a partir del cual la velocidad de remocin se hace dependiente de la concentracin y decrece con ella.
Hay otros casos en los que ha quedado demostrado que la remocin de
sustrato (DBO DQO) sigue una cintica de orden cero, o sea, es independiente de la concentracin de DBO DQO en todo el intervalo de concentracin. 10 Uno u otro comportamiento se debe a que el contaminante est
constituido por una mezcla de sustancias o por una sola sustancia.
Para el estudio cintico de la remocin de la DBO es conveniente clasificar los sustratos como:
Sustratos de un solo componente que pueden ser transportados directamente al interior de la clula.
Sustratos multicomponentes, constituidos por una mezcla de sustratos simples.

91

Menndez Gutirrez, C. y J. Prez Olmo

Sustratos de un solo componente


Como ya ha sido expresado, la remocin de sustrato de una solucin, siempre va acompaada de crecimiento de biomasa, existiendo una relacin matemtica que vincula ambos fenmenos:

dX
dt

dS
dt

(3.18)

donde:
Y: Constante de rendimiento o de produccin de biomasa (gg).
A partir de la ecuacin que describe el crecimiento de biomasa en funcin
de la concentracin de sustrato,

dX
dt

S
X
KS  S

y de la ecuacin 3.18, puede obtenerse:




dS
dt

S
KS  S

(3.19)

en tratamiento de residuales es frecuente encontrar que KS  S, y la ecuacin


3.19 queda:

dS
dt

m
X
Y

(3.20)

dS
dt

KX

(3.21)

o tambin:

donde:

m 1
d
Y

De acuerdo con la ecuacin 3.21 los sustratos de un solo componente son


removidos siguiendo una cintica de orden cero con respecto a la concentracin
de sustrato (figura 3.10).

92

Oxidacin Biolgica. Nitrificacin

Fig. 3.10. Variacin de la concentracin de DBO con respecto al tiempo,


para sustratos de un solo componente.
Sustratos multicomponentes
La casi totalidad de las aguas residuales estn constituidas por sustratos
multicomponentes, o sea, una mezcla de compuestos que integralmente se reportan como DBO DQO.
Cada uno de los compuestos que conforman el residual est presente en
concentraciones distintas, y se remueven de acuerdo con una cintica de orden
cero aunque a distintas velocidades, como se ilustra en la figura 3.11(a), donde
cada recta representa la variacin, con respecto al tiempo, de la DQO de cada
uno de los constituyentes individuales.
En la figura 3.11(b) se aprecia que para un tiempo cualquiera la velocidad de
remocin del sustrato multicomponente y la concentracin total del sustrato (expresada como DQO) es la suma de las velocidades de remocin de los sustratos
simples y de la concentracin de los sustratos respectivamente. En la medida
que un sustrato simple se agota, la velocidad de remocin disminuye instantneamente, dando origen a un punto de ruptura en la curva de remocin.
Si el nmero de componentes es suficientemente alto, como es el caso de la
prctica del tratamiento de las aguas residuales, el punto de ruptura puede ser
sustituido por una disminucin suave. Esta curva puede ser descrita por una
ecuacin diferencial general que representa la cintica de una reaccin qumica
simple:

dS
dt

kn X 0 S n

donde:

dS
: Velocidad de remocin (mgLd).
dt
kn: Constante de velocidad de remocin (d).
93

(3.22)

Menndez Gutirrez, C. y J. Prez Olmo

S: Concentracin de DBO DQO (mgL).


X0: Concentracin inicial de biomasa (mgL).
n: Orden de la reaccin.
Este tipo de ecuacin ha sido aplicada durante aos para los procesos
microbiolgicos. Sin embargo, el punto dbil en la aplicacin de esta ecuacin,
es que presupone un incremento en la velocidad de remocin ante cualquier
incremento en la concentracin de sustrato.
Experimentalmente ha quedado demostrado que si se incrementa la concentracin de un sustrato multicomponente, y se mantiene constante la proporcin de cada componente en el sustrato, no se apreciar incremento en la
velocidad de remocin.
De la figura 3.11 se aprecia que la velocidad instantnea de remocin de un
sustrato multicomponente est relacionada con el nmero de sustratos simples
remanentes.

Fig. 3.11. Ejemplo terico de remocin de sustratos multicomponentes.


94

Oxidacin Biolgica. Nitrificacin

Al considerar la velocidad de remocin inicial, que es constante para un


cultivo y mezclas de sustratos especficos, puede expresarse la velocidad en
funcin de nmero de componentes remanentes en relacin con la situacin
original.
As:

m
f
M

(3.23)

donde:
M: Nmero de componentes inicialmente presentes.
m: Nmero de componentes individuales remanentes.
r: Velocidad de remocin relativa.
En general, lo que ocurre es que no se conoce el nmero de componentes
originalmente presentes, ni la ley que describe su remocin de la mezcla.
Suponiendo un nmero grande de componentes, puede sustituirse el nmero
de componentes por la concentracin.
La funcin f expresa adems, en alguna forma, el decrecimiento de la velocidad de remocin causada por el nmero reducido de componentes y, por tanto,
la disminucin de la concentracin total de sustrato con el tiempo. Para reflejar
esto, un exponente similar al de orden de reaccin puede utilizarse.
La ecuacin cintica se puede expresar:
dS

dt

S
kn X 0
S0

kn

X0 n
S
S0 n

(3.24)

donde:
S0: Concentracin inicial de DBO DQO.
S: Concentracin final de DBO.
X0: Concentracin de biomasa.
kn: Velocidad especfica de remocin de DBO.
n: Orden de la reaccin.
La ecuacin 3.24 tambin puede expresarse,
S
k
S0

6
X

V
Q

95

(3.25)

Menndez Gutirrez, C. y J. Prez Olmo

El modelo de velocidad de reaccin representado por la ecuacin 3.24 fue


inicialmente descrito por Grau y col.11 Aunque bastante general, el modelo cintico
de Grau no es ms que otro de los muy diversos modelos que pretenden explicar
la cintica de los procesos biolgicos de tratamiento de aguas residuales. Entre
muchos otros modelos descritos pueden ser citados el de Eckenfelder 12 y el de
Lawrence y McCarty. 13
La ecuacin que responde al modelo cintico de Eckenfelder ha tenido un
uso muy difundido a travs de los aos. Originalmente formulada para una cintica
de primer orden con respecto a la DBO, ha sido posteriormente utilizada para
rdenes de reaccin superior al primero,

dS
dt

K S n X

(3.26)

donde:
n: Orden de reaccin.

Fig. 3.12. Diferentes valores de la constante cintica K que se obtiene al


aplicar el modelo de primer orden de Eckenfelder a un agua residual de
DBO variable.
Otra forma de expresar la ecuacin 3.26 es,

6
X

K S n

(3.26 a)

La mayor limitacin del modelo de Eckenfelder radica en que para aguas


residuales cuya DBO es variable, se obtiene distintos valores de K en dependencia de los valores de la DBO inicial (S0). Figura 3.12. Esta dificultad es
salvada en el modelo cintico de Grau.
Por otra parte, el modelo de Lawrence y McCarty es aplicable para cintica
de orden intermedio entre cero y primer orden,
96

Oxidacin Biolgica. Nitrificacin

dS
dt

km

6
X

S
X
KS  S

km

(3.27)

S
KS  S

(3.27 a)

Cualquiera que sea el modelo cintico que se desee emplear, debe comprobarse en qu medida se cumple en el caso particular bajo estudio.

3.4.3. Clculo de la constante de velocidad


Para estimar la constante de velocidad de remocin de DBO deben realizarse corridas experimentales a diferentes tiempos de retencin y conocer para
cada caso, la concentracin de biomasa, medida como slidos suspendidos voltiles, as como la DBO inicial y final del proceso.
En el ejemplo 3.1 se ilustra el procedimiento de clculo.
Ejemplo 3.1
Para determinar la constante del modelo cintico de Grau se realizaron
corridas experimentales en un reactor de 10 L con alimentacin continua. Los
resultados obtenidos aparecen resumidos en la tabla 3.1.
Determine el valor de la constante asumiendo que la reaccin de degradacin es de primer orden con respecto a la DBO.
Tabla 3.1. Datos experimentales
Corrida
Experimental
No.
1
2
3
4
5
6

S0

Ld-1
13
15
30
30
40
60

mgL-1
900
950
1200
1200
1500
1500

De acuerdo al modelo de Grau:

6
X

k
97

S
S0

63
133
420
300
750
900

7247
4126
3151
4959
2500
2582

Menndez Gutirrez, C. y J. Prez Olmo

graficando

6
X

vs

S
S0 , puede obtenerse el valor de k de la pendiente de la

recta correspondiente.
Los datos de la tabla 3.1a se arreglan segn:
Tabla 3.1.a. Transformacin de la tabla 3.1
Corrida 1
0,15
6
X
0,07
S
S0

2
0,30

3
4
5
6
0,75 0,55 1,20 1,40

0,14

0,35 0,25 0,50 0,60

Se obtiene la recta de la figura 3.13 cuya pendiente es el valor de la constante k (k = 2,3 d):

Fig. 3.13. Clculo de la constante de remocin de DBO para el modelo


de Grau.

3.5. NITRIFICACIN-DESNITRIFICACIN
Es muy til relacionar las concentraciones de nitrgeno con las de DBO. La
tabla 3.2 da algunos valores en dependencia del origen del agua residual. El
valor 0,2 puede considerarse representativo de este conjunto de datos. A falta
de disponer de anlisis ms precisos, puede ser adoptada esta relacin como
buena para el clculo de instalaciones de tratamiento. Generalmente la incorporacin de aguas residuales industriales a las municipales, favorecen la disminucin de esa relacin.
98

Oxidacin Biolgica. Nitrificacin

Tabla 3.2. Relacin N/DBO5 para diferentes tipos de aguas residuales

Procedencia
Municipal
Lecheras
Cerveceras
Papeleras
Centrales azucareros
Tintoreras

N/DBO5
0,17-0,20
0,05-0,04
0,25
0,1-0.02
0,1-0,2
0,1-0,05

Formas del nitrgeno y su evolucin


En las aguas residuales de origen domstico, la forma ms comn en que se
presenta el nitrgeno es como nitrgeno orgnico (N-org) y nitrgeno amoniacal
(N-NH3). En esta agua los nitritos y nitratos aparecen generalmente en concentraciones casi siempre despreciables. La presencia de nitritos y nitratos en las
aguas residuales sin tratar casi siempre es debida a contaminacin de origen
industrial. Cuando la concentracin de N-NH 3 es elevada, se produce una accin inhibidora superior sobre un grupo de las bacterias nitrificantes, las
nitrobacterias, que sobre otro, las nitrosomonas.
(O QLWUyJHQR WRWDO .MHOGDKO 117. UHSUHVHQWD HO WRWDO GH QLWUyJHQR HQ
forma orgnica y amoniacal y no incluye nitritos ni nitratos.
El nitrgeno orgnico puede ser inerte o degradable, con un orden de magnitud del 12 % (10 % en suspensin y 2 % soluble) y 88 % respectivamente. Las
dos ms importantes fuentes de contaminacin nitrogenada son los vertidos de
protenas y urea.
La fraccin orgnica degradable se transforma, por accin enzimtica, en
forma amoniacal, mediante el proceso conocido como amonificacin. La cintica
de esta reaccin es muy rpida para la urea, pero inferior para las protenas.
Aun as, se puede considerar que al final del tratamiento no subsiste ms que la
fraccin orgnica inerte.
De lo anterior puede concluirse que, con el tratamiento por lodos activados, no es necesario conocer la proporcin entre las formas orgnicas y
amoniacales. En cambio, en los sistemas de tratamiento del tipo cultivos
fijos (lechos bacterianos, discos biolgicos, filtros, etc.), en los cuales el
tiempo de retencin en el reactor es muy corto, puede no completarse la
amonificacin y si puede cobrar importancia conocer la proporcin en que
se encuentran el N-org y el N-NH 3.
El nitrgeno amoniacal procede de la amonificacin del nitrgeno orgnico
que puede efectuarse en la propia red de colectores y que afecta a prcticamente
la totalidad de la urea y de los dems compuestos orgnicos fcilmente
99

Menndez Gutirrez, C. y J. Prez Olmo

degradabIes. Una parte de este nitrgeno se incorpora a la masa bacteriana en


la constitucin de las clulas a travs del proceso de asimilacin.
Cuando las condiciones ambientales son favorables, en particular el tiempo
de retencin de la biomasa y la temperatura, el nitrgeno llega a oxidarse a la
forma de nitrato en el proceso biolgico aerobio identificado como nitrificacin.
En un reactor anxico algunas bacterias hetertrofas son capaces de consumir oxgeno de los nitratos, los cuales se reducen liberando nitrgeno libre.
Este proceso se denomina denitrificacin.
Todos estos procesos se representan en el diagrama de bloques de la
Figura 3.14.

Fig. 3.14. Ciclo del nitrgeno en el tratamiento.

3.5.1. Nitrificacin
En el estudio de la nitrificacin es importante diferenciar entre las funciones
de oxidacin del carbono y del nitrgeno propiamente dicho. Ambos procesos
pueden llevarse a cabo en una sola etapa o en etapas separadas, tanto en sistemas de biomasa en suspensin como en lecho fijo.
En casi todos los procesos biolgicos estn presentes las bacterias
nitrificantes, sin embargo para que la nitrificacin tenga lugar es necesario que
100

Oxidacin Biolgica. Nitrificacin

existan, entre otras cosas, condiciones adecuadas para el crecimiento de las


bacterias nitrificantes. En otro caso, prevalecer el crecimiento de las bacterias
hetertrofas. La nitrificacin se propicia disminuyendo la carga orgnica de los
procesos biolgicos.
El N-NH3 es transformado en nitrito y nitrato por bacterias autotrficas
constituyendo el oxgeno requerido para esta oxidacin, una demanda adicional
a la que posee la materia orgnica que es normalmente estabilizada durante los
tratamientos biolgicos.
La primera etapa de la nitrificacin tiene su origen en la accin de las bacterias Nitrosomonas, siendo las Nitrobacter las responsables de la transformacin posterior del N-NO3. Ambas reacciones de oxidacin se representan a
continuacin:
Nitrosomonas

NH 4  3/2 O 2 o 2H   H 2O  NO 2
Nitrobacte r

NO 2  1/2 O 2 
o NO 3

(3.28)
(3.29)

A partir de la estequiometra de estas ecuaciones puede deducirse que se


requieren 4,6 mg de oxgeno para la oxidacin completa de cada mg de N-NH3.
Si se acepta que el efluente de un tratamiento secundario convencional posee aproximadamente una concentracin de 25 mgL de N-NH3, esto tiene
como consecuencia una demanda de oxgeno al cuerpo receptor de 114 mgL 
por concepto de la ulterior oxidacin a NO3. Si esta demanda se compara con
la que normalmente tienen estos efluentes (demanda carboncea), aproximadamente
50 mgL, puede inferirse la importancia que posee el proceso de nitrificacin,
su estudio y control.
La oxidacin del nitrgeno amoniacal no tiene lugar de forma aislada, sino
que en realidad va acompaada de la sntesis de nuevas clulas adems de
consumo de oxgeno. La sntesis de nuevas bacterias nitrificantes se expresa:14

+
(Nitrosomonas)
15CO2 + 13 NH4+ 

o 10 NO2 + 3 C5H7NO2 + 23 H + 4 H2O (3.30)

+
(Nitrobacter)
5 CO2 + NH4+ + 10 NO2 + 2 H2O 
o 10 NO3 + C5H7NO2 + H (3.31)

El rendimiento celular de estas reacciones est en el intervalo de 0,04 a


0,29 mg SSV (mg N-NH3) para las bacterias Nitrosomonas y de 0,02 a
0,084 mg SSV (mg N-NO2) para Nitrobacter.15
Suponiendo para estas dos reacciones respectivamente rendimientos de 0,15
mg SSV(mgN-NH3) y 0,02 mg SSV(mgN-NO2) puede plantearse:

55 NH4+ + 76 O2 + 109 HCO3 


o C5H7NO2 + 54 NO2 + 57 H2O +
+ 104 H2CO3
(3.32)

101

Menndez Gutirrez, C. y J. Prez Olmo

400 NO2 + NH4+ + 4 H2CO3 + HCO3 + 195 CO2


C H NO +

o 5 7 2
+ 3 H2O + 400 NO3
(3.33)
De las ecuaciones 3.32 y 3.33 puede obtenerse:
NH 4   1,83 O 2  1,98 HCO3  o 0,021C 5H 7 NO 2  1,04 H 2 O 
 0,98 NO3  1,88 H 2CO3

(3.34)

Dos conclusiones importantes se derivan del anlisis de la ecuacin 3.34. La


primera de ellas es que el N-NH3 requerido para sntesis es despreciable, 2 %,
con respecto al que se utiliza por las bacterias nitrificantes para la oxidacin. La
segunda conclusin es que la nitrificacin consume alcalinidad del medio,
requirindose 7,15 mgL como CaCO3 por cada mgL de N-NH3 oxidado,
valor que se ajusta al reportado por la literatura.16
Cintica de la nitrificacin
Para los sistemas de biomasa en suspensin los factores que se apuntan
como incidentes en la nitrificacin son:

Concentracin de N - NH3 y N- NO2


Relacin DBO5/NKT
Concentracin de oxgeno disuelto
Temperatura
pH

Se ha demostrado que existe una relacin directa entre la capacidad de


nitrificacin en un determinado proceso y la relacin DBO 5/NKT. Para valores
de este cociente entre 1 y 3, el porcentaje de bacterias nitrificantes en la biomasa
oscila entre 21 y 8,3 % respectivamente.
En los sistemas de lodo activado de tipo convencional, para los que la relacin emprica DBO5/NKT es mayor de 3, el porcentaje de bacterias nitrificantes
es mucho menor de 8,3. En los sistemas de lecho fijo, tanto en los filtros
percoladores como en los biodiscos, tambin puede presentarse la nitrificacin.
Temperatura
La temperatura ejerce una influencia importante sobre la velocidad de
nitrificacin, aumentando esta ltima con el incremento de la temperatura del
medio. Esta influencia se refleja en las constantes asociadas a la cintica de la
nitrificacin.
El intervalo de temperatura en el que ocurre preferentemente la nitrificacin
es el comprendido entre 4 y 45 qC, siendo 35 qC la temperatura ptima para las
EDFWHULDV1LWURVRPRQDV\qC para Nitrobacter.
102

Oxidacin Biolgica. Nitrificacin

En la tabla 3.3 se reportan valores de velocidad especfica de crecimiento


de Nitrosomonas para tres temperaturas diferentes.
Tabla 3.3. Velocidad mxima especfica de crecimiento para Nitrosomonas

Temperatura, qC P n,m
10
0,3
20
0,65
30
1,2
Para propsitos de diseo, una expresin aceptable del efecto de la temperatura sobre la velocidad mxima especfica de crecimiento para las
nitrosomonas es:
nm

0,65 e 0 ,098 ( t  20 )

(3.35)

Oxgeno disuelto
Generalmente la influencia de la concentracin de oxgeno en la velocidad de nitrificacin ha sido objeto de controversias. No obstante, es aceptado, por evidencia experimental, que bajo determinadas condiciones se puede
obtener nitrificacin completa aun a concentraciones de oxgeno tan bajas
como 0,5 mgL . Un estudio relativamente reciente para analizar la influencia de diversos factores en la nitrificacin ha sido reportado por Stenstrom y
Song. 17 En este estudio se concluye que:
Tanto para sistemas de biomasa en suspensin como para medio fijo, la
concentracin de oxgeno que limita la nitrificacin est en el intervalo
comprendido entre 0,5 y 2,5 mgL.
La concentracin lmite de oxgeno para la nitrificacin en condiciones
estacionarias es funcin del tiempo de retencin de la biomasa y de la
resistencia difusional al transporte de masa.
En la medida en que la concentracin de oxgeno es menor, se requiere
mayor tiempo de retencin de la biomasa para garantizar una completa
nitrificacin.
Bajo condiciones transcientes de choques de carga orgnica, o de aumento de la resistencia a la difusin o competencia entre bacterias
hetertrofas y nitrificantes, la concentracin lmite de oxgeno puede ser
significativamente mayor que bajo otras condiciones.

103

Menndez Gutirrez, C. y J. Prez Olmo

Efectos del pH y alcalinidad


El pH tambin puede influir notablemente sobre la velocidad de nitrificacin.
Como ya se ha expresado, la nitrificacin lleva implcita la destruccin
de una cantidad importante de alcalinidad del medio.
Puede demostrarse que en los reactores biolgicos, para el intervalo de
pH comprendido entre los valores de 5 y 8, el pH de equilibrio esta determinado por la alcalinidad y el CO 2 presente en el sistema. En aquellos casos de
reactores abiertos, en los que ocurre desprendimiento de CO 2, pueden mantenerse altos valores de pH a bajos niveles de alcalinidad en el medio.
En sistemas cerrados, donde normalmente el despojamiento de CO 2 se
dificulta, la alcalinidad del agua residual debe ser hasta 10 veces el valor
de la concentracin del nitrgeno que se oxida para que pueda mantenerse
el pH.
Para propsitos de diseo, es suficiente tomar en consideracin que la
velocidad de nitrificacin disminuye considerablemente en la medida que el
pH cae por debajo del intervalo de neutralidad del medio. Es mejor mantener el pH entre los valores de 6,5 y 8,0.
La influencia del pH sobre la velocidad mxima de nitrificacin puede cuantificarse mediante la expresin,

nm

nm >1  0,833 7 ,2  pH

(3.36)

donde:
P nm: velocidad especfica de nitrificacin a pH = 7,2.
Efecto de la relacin DBO5/NKT
Generalmente en todos los sistemas de nitrificacin hay suficiente materia orgnica en el reactor como para propiciar el crecimiento de las bacterias hetertrofas. Cuando se incrementa el tiempo de retencin medio celular
(edad del lodo) para incrementar la concentracin de biomasa (SSV), con el
objetivo de obtener mayor remocin de DBO, se corre el peligro potencial
de que la velocidad de crecimiento de las bacterias nitrificantes sea mayor
que el de las hetertrofas. En caso contrario, y se reduzca mucho el TRMC,
se lavaran del sistema. Por tanto, la condicin de trabajo ha de ser tal que el
tiempo de retencin de los slidos sea mayor que un valor limite que se
establezca.
Opuestamente a lo que sucede en los sistemas de biomasa en suspensin, en aquellos en los que la biomasa esta adherida a un medio, sistemas de
biopelcula, el factor que controla es el proceso de transporte de masa.

104

Oxidacin Biolgica. Nitrificacin

Las condiciones que existen en el entorno cercano de los microorganismos


en la biopelcula, no son las mismas que para los sistemas de biomasa en
suspensin. En la biopelcula la concentracin de sustrato vara con la profundidad a lo largo del espesor, y es significativamente menor que en el seno
del lquido. Siendo esto vlido tanto para la concentracin de oxgeno como
para la de nitrgeno amoniacal, la velocidad de nitrificacin en biopelculas
es menor que la que podra esperarse tomando como referencia la concentracin de nitrgeno en el lquido.
El transporte de masa o resistencia difusional incide sobre la nitrificacin de
la manera que se describe:
No es vlida la suposicin de que la etapa limitante de la nitrificacin
sea la oxidacin del nitrgeno amoniacal a nitrito.
Aumenta el tiempo de retencin que se requiere para los slidos.
Cambia la concentracin lmite de oxgeno para que ocurra la nitrificacin.
Investigaciones realizadas por Downing, Painter y Knowles1 8 concluyen que
la cintica de la nitrificacin puede ser formulada en trminos de la relacin de
Monod.

dX n
dt
y

nm

dNa
dt

(3.37)

Na
Kn  Na

(3.38)

Yn

donde:
Xn: Concentracin de microorganismos nitrificantes (mgL).
Na: Concentracin de N-NH3 (mgL).
Yn: Masa de organismos nitrificantes obtenidos por masa de N-NH3 utilizado.
P n: Velocidad especfica de crecimiento de los organismos nitrificantes a la
concentracin Na (mg Xn (mg Xn) d).
Kn: Coeficiente de saturacin para nitrosomonas (mgL N-NH3).
Est demostrado que la energa que liberada durante el proceso de conversin de N-NH3 a N-NO2 es aproximadamente tres veces mayor a la correspondiente a la transformacin del N-NO2 a N-NO3.19 O sea, se requiere producir
el triple de N-NO3 que N-NO2 para liberar una misma cantidad de energa.
Segn Adams y Eckenfelder20 esta es la posible explicacin al hecho de que
generalmente el N-NO2 se halla en bajas concentraciones en los efluentes de
los tratamientos biolgicos.
105

Menndez Gutirrez, C. y J. Prez Olmo

Teniendo en cuenta que las Nitrobacter tienen una velocidad especfica de


crecimiento mayor que el de las Nitrosomonas, la modelacin de la nitrificacin
puede considerarse como un proceso de una sola etapa, la limitante, que es la
conversin del N-NH3 a N-NO2.21
Esta es la razn por la cual los diferentes trminos definidos para las
ecuaciones 3.37 y 3.38 tienen como base el N-NH3.
La velocidad de crecimiento de las bacterias nitrificantes puede expresarse
como:

dX n
(3.39)
n X n
dt
Combinando las ecuaciones 3.37, 3.38 y 3.39 para explicar la velocidad de
crecimiento en funcin del consumo de N-NH3 se obtiene:


1
Na
X
nm
Kn  Na n
Yn

dNa
dt

(3.40)

O sea, que la velocidad de nitrificacin es de primer orden con respecto a la


concentracin de Nitrosomas, y de un orden intermedio con respecto a la concentracin de N-NH3 .
La ecuacin 3.40 puede escribirse de una forma equivalente si se define la
velocidad especfica mxima de consumo de sustrato, Km

Km

nm
Yn

(3.41)

Entonces la velocidad especfica de consumo de sustrato puede expresarse:

Km

Na
Kn  Na

(3.42)

y la velocidad de consumo de sustrato, o sea, de consumo de N-NH3 para el


proceso de nitrificacin ser:

dN a
dt

Km

Na
X
Kn  Na n

(3.43)

Se conoce que el valor de Kn a 20 oC es menor de 1 mgL.22


En la tabla 3.4 se presentan tres valores reportados en la literatura de velocidades especficas de crecimiento y coeficientes de saturacin para
Nitrosomonas, a 20 oC.
En aquellos casos en los que la concentracin de N-NH3 tenga un valor
cercano a los 25 mgL o mayor, puede considerarse, desde el punto de vista
106

Oxidacin Biolgica. Nitrificacin

prctico, que la velocidad de nitrificacin es de orden cero con respecto a la


concentracin de N-NH3. Para Na !! Kn, la ecuacin 3.41 se transforma en:

dN a
(3.44)
Km X n
dt
o sea, la velocidad de nitrificacin solo es funcin de la concentracin de las
bacterias nitrificantes, su velocidad especfica de crecimiento mxima y del rendimiento celular.
Tabla 3.4.Velocidad especfica mxima de crecimiento y coeficientes
de saturacin para Nitrosomonas a 20 oC

P nm
d-1
1,32
0,84
1,62

Kn
Referencia
-1
mgL N-NH3
3,6
22
1,0
23
0,6
24

Con el objetivo de llegar a conocer las condiciones a partir de las cuales


ocurre la nitrificacin puede hacerse un balance de biomasa nitrificante (figura 3.15).

Fig. 3.15. Esquema para el balance en la etapa de nitrificacin.

; n
W

Na
n m
X V  bn X n V  X n Qw
Kn  Na n

donde:
V: Volumen del reactor.

; n : Velocidad de crecimiento de las bacterias nitrificantes.


W
P nm: Velocidad especfica de crecimiento.
107

(3.4)

Menndez Gutirrez, C. y J. Prez Olmo

Xn: Concentracin de las bacterias nitrificantes.


Na: Concentracin de N-NH3.
bn: Constante de respiracin endgena.
QW: Flujo de extraccin de lodo.
Transformando la ecuacin 3.45 se obtiene:

; n
Na
Q
nm
X n  bn X n  X n w
W
K n  Na
V
Considerando que:
V Xn
x
Qw X n
y que en condiciones de estado estacionario se cumple que
Xn
W

(3.46)

(3.47)

entonces, de la ecuacin 3.46 se tiene que,

Na

1
K n bn 
x

n m  bn 
x

(3.48)

De la ecuacin 3.48 puede afirmarse que: la concentracin de nitrgeno


amoniacal en el reactor, y por tanto, en el efluente, es funcin de la edad del
lodo e independiente tanto de la concentracin de amonio en el afluente Nai
como del coeficiente de rendimiento Yn .
El desarrollo anterior demuestra que la nitrificacin no siempre est presente, sino que se requiere un valor de edad de lodo mnima para que ocurra.
Suponiendo valores tpicos21 se obtiene la figura 3.16.
En la figura 3.16 se aprecia que para altas edades de lodo el valor del
nitrgeno amoniacal es pequeo, pues ha ocurrido la trasformacin de la
forma amoniacal a N-NO2 y N-NO3 . Para edades de lodo, cercanas a 3,5
d, la concentracin de Na se incrementa bruscamente. Si T x  3,5 d, el valor
de N-NH3 aumenta hasta que se obtiene:
Na = Nai , que es la condicin lmite.

108

Oxidacin Biolgica. Nitrificacin

Fig. 3.16 Influencia de la edad del lodo en la nitrificacin.


Desde el punto de vista cuantitativo el valor de edad de lodo por debajo del
cual no existe nitrificacin se obtiene haciendo Na = Nai en la ecuacin 3.48.
Teniendo en cuenta, que Na rara vez es menor de 20 mgL y que Kn | 1,0 mgL,
se cumple que:

Ko
1
N ai
Ko
Suponiendo que N | 0 , lo que es vlido desde el punto de vista prctico, y
ai
resolviendo la ecuacin 3.48 para xm , se tiene que:
x, m

1
n, m  bn

(3.49)

De aqu puede concluirse que aunque a los efectos pr cticos,


generalmente se considera que para que ocurra nitrificacin se requiere una
edad de lodo superior a tres das, esto no es absoluto, ya que la edad de lodo
mnima depende de la velocidad especfica de crecimiento mxima de las
Nitrosomonas y de su coeficiente de respiracin endgena. Tambin a los
efectos prcticos, el valor del coeficiente bn puede asumirse despreciable en
la ecuacin 3.49.
Esto explica el fenmeno de nitrificacin que se ha reportado a valores de
edades de lodo de dos das en el tratamiento de residuales de pulpa y papel.22
109

Menndez Gutirrez, C. y J. Prez Olmo

Ejemplo 3.2
Calcular la velocidad mxima especfica de crecimiento de las bacterias
nitrificantes bajo condiciones no limitantes de pH y oxgeno disuelto. Considere
una temperatura de 30 RC.
Para propsitos de diseo puede utilizarse le ecuacin 3.35:
n,m

0 ,65 e 0 ,098 t  20

n,m

0 ,65 e 0 ,098 30  20

mn,m = 1,73 d
Ejemplo 3.3
Determine el tiempo de retencin celular de diseo (edad del lodo de diseo)
para que ocurra nitrificacin.
En estado estacionario, la cantidad de slidos que salen del sistema es numricamente igual a la que se produce. Por tanto, la relacin entre la edad de
lodo mnima para que ocurra nitrificacin y P n ser,
1
x,m

n,m  bn

n,neto

Asumiendo que bn = 0,
1
x,m

n,m

Para disear puede considerarse que la edad del lodo que debe tomarse
debe ser al menos 3,5 veces la edad del lodo mnima,

x,d
x,d

3,5 x,m
2d

Ejemplo 3.4
Considerando que la alcalinidad del medio en los ejemplos anteriores es
25 mgL como CaCO3, estime la que debe ser aadida para satisfacer la demanda exigida por la nitrificacin.
Cada mg de N-NH3 al oxidarse a NO3, consume 7,1 mg de alcalinidad.
Para determinar el N-NH3 que se oxida,

110

Oxidacin Biolgica. Nitrificacin

n
n

1
x ,d

0,5

n,m

Na
Kn  N a

suponiendo que Kn

0,5 1,73

Na
1  Na

1,0

Na = 0,41 mgL

La alcalinidad a consumirse durante la nitrificacin es,


= 24,59 mgL
alcalinidad consumida 24,59 7,1 175 mgL
alcalinidad a adicionar  25 mgL

3.5.2. Desnitrificacin
La denitrificacin biolgica implica la reduccin microbiolgica del nitrato a nitrito y finalmente del nitrito a nitrgeno gaseoso. El nitrato y nitrito
constituyen la fuente de oxgeno para este proceso. De tal forma, la
desnitrificacin normalmente ocurre en ausencia de oxgeno molecular, pero
con la presencia de nitrato. Estos procesos son denominados anxicos. De
modo que la desnitrificacin transforma el nitrgeno de una forma potencialmente objetable, nitrato, a otra biolgicamente inerte, nitrgeno gaseoso.
La desnitrificacin presenta cinco campos de inters, sin considerar los
riesgos que para la salud puede implicar un efluente con alto contenido de
nitrato:
Consigue recuperar una parte de la alcalinidad que fue consumida
durante el proceso de nitrificacin. Esto es:
3,5 g como CaCO3 por cada g N-NO3 reducido.
Disminuye considerablemente los riesgos potenciales comnmente asociados a la desnitrificacin incontrolada que puede desarrollare en la
sedimentacin secundaria.
Recupera una parte del oxgeno consumido por la nitrificacin:
2,6 g de oxgeno por cada g de N-NO3 reducido.
Un reactor anxico previo a la etapa de aeracin mejora la sedimentabilidad
de los fangos, disminuyendo el ndice volumtrico de lodos.
Contrariamente al pequeo nmero de especies nitrificantes, las de las
desnitrificantes es mucho mayor. De ah que resulten ms aptas para su acli111

Menndez Gutirrez, C. y J. Prez Olmo

matacin y, por consiguiente, tengan una mayor y mejor resistencia a las


agresiones del medio ambiente.
Muchos microorganismos presentes en los sistemas de lodos activados municipales pueden llevar a cabo procesos de desnitrificacin, aunque estos no se
hayan diseado especficamente para ello. Su presencia se debe en parte a que
son organismos facultativos. O sea, que pueden utilizar indistintamente el oxgeno o el nitrato como ltimo aceptor de electrones. Por otro lado las bacterias
desnitrificantes pueden tambin proliferar en sistemas aerobios, debido a su
habilidad para emplear el oxgeno molecular, y oxidar eficientemente la materia
orgnica.
La capacidad de ubicuidad de las bacterias desnitrificantes minimiza la necesidad de crear un ambiente con condiciones especiales para su subsistencia,
tal como ocurre con las nitrificantes.

3.5.2.1. Metabolismo de la desnitrificacin


En la desnitrificacin el nitrato y nitrito actan como aceptores de electrones en la cadena respiratoria, tal como lo hace el oxgeno en un medio aerobio.
El transporte a travs de esta cadena es el mecanismo fundamental mediante el
cul las clulas liberan energa.
En el sistema generalmente existe un mecanismo de autocontrol, de manera
tal que si hay oxgeno presente en el medio donde est teniendo lugar el proceso,
el oxgeno se utilizar preferentemente sobre el nitrato. En ausencia de oxgeno, el
nitrato ser la especie que se favorecer como aceptor final de electrones sobre
cualquier otro compuesto que pueda ser reducido, como por ejemplo el sulfato.
El mecanismo de control de las bacterias desnitrificadoras ocurre en dos
niveles:
Sntesis de las enzimas requeridas para la desnitrificacin
Actividad enzimtica propiamente dicho
Es conocido que son necesarias entre 2 y 3 horas para la sntesis enzimtica
en cultivos puros cuando el hbitat de las bacterias es modificado de un medio
aerobio para uno anxico. Sin embargo, estudios han demostrado que las enzimas
desnitrificantes pueden estar presentes aun cuando el medio no sea anxico,
sugiriendo que el periodo de sntesis de las enzimas puede ser innecesario con
anterioridad al inicio de la desnitrificacin.23
Por otro lado el oxgeno inhibe la actividad de las enzimas desnitrificadoras.
Para cultivos puros se ha reportado que a valores de la concentracin de oxgeno de 0,2 mgL ya se aprecia inhibicin.24 En lodos activados los valores
inhibitorios reportados estn comprendidos entre 0,3 y 1,5 mgL .25
112

Oxidacin Biolgica. Nitrificacin

Generalmente la masa de agua que se desea desnitrificar necesita tener


suficiente materia orgnica que pueda ser utilizada como fuente de energa para
la masa biolgica encargada de transformar el nitrato en nitrgeno gaseoso.
Cuando se carece de este contenido de materia orgnica, hay que utilizar una
fuente externa tal como el metanol, etanol o cido actico, entre otros compuestos.

3.5.2.2. Cintica de la desnitrificacin


La cintica de la desnitrificacin puede ser expresada de modo similar a la
cintica de la nitrificacin o de la oxidacin de los compuestos orgnicos.
La ecuacin de Monod es un medio til para explicar los conceptos generales del proceso de desnitrificacin. Esta expresin frecuentemente es empleada
en aquellos casos en los que este proceso ocurre en reactores independientes y
se emplea el metanol como fuente de materia orgnica.
Cuando se concibe un solo reactor para que tengan lugar todos los procesos,
se dificulta el empleo de Monod debido a la heterogeneidad tanto del sustrato
como de la biomasa.
Como ya se expres, existen casos en los que la velocidad de crecimiento
de las bacterias desnitrificantes puede representarse mediante la ecuacin de
Monod:

Dm

D
KD  D

(3.50)

donde:
P D: Velocidad especfica de crecimiento de las bacterias desnitrificantes h.
D: Concentracin N-NO3 (mgL).
P Dm: Velocidad especfica de crecimiento mxima h.
KD: Constante de saturacin (mgL).
Si KD  D, entonces la ecuacin anterior se transforma en una expresin
de orden cero con respecto a la concentracin de nitrato,
mD = mDm
(3.51)
Por el contrario, si KD !! D, se estar en presencia de una reaccin de
primer orden,
D

D m
KD

(3.52)

Como la velocidad de crecimiento de las bacterias desnitrificantes es similar


al de las hetertrofas, mucho mayor que el de las nitrificantes, el tiempo de
113

Menndez Gutirrez, C. y J. Prez Olmo

retencin medio celular para prevenir el lavado de dichas bacterias del reactor
es mucho menor que el de estas ltimas.
La velocidad de remocin de nitrato puede relacionarse con la velocidad
especfica de crecimiento,

D
YD

UD

U Dm

D
KD  D

(3.53)

UD: Velocidad especfica de remocin g N-NO3(gSSVd).


YD: Coeficiente de rendimiento SSV (N-NO3 remov.).
UDm: Velocidad especfica mxima de remocin de nitrato
g N-NO3 (g SSV).
La influencia de la concentracin de materia orgnica sobre la desnitrificacin
puede expresarse mediante una relacin similar,

UD

U Dm

S
KS  S

(3.54)

donde:
S: Concentracin de sustrato orgnico mgL.
Como ambos, tanto la concentracin de nitrato como de sustrato de
origen orgnico influyen sobre la velocidad de desnitrificacin, una mejor
aproximacin se obtiene cuando la velocidad de remocin de nitrato se
expresa,

UD

U Dm

S
D
KS  S K D  D

(3.55)

Existen evidencias experimentales de que los valores de KD son relativamente


bajos (0,1-0,2 mg N-NO3 L).26 A partir de este hecho, y analizando la ecuacin (3.52), puede concluirse que concentraciones de N-NO3  tan bajas como
1-2 mgL no tienen efecto sobre la velocidad de desnitrificacin. Por lo tanto, la
concentracin de nitrato no influye en la velocidad de ese proceso, a menos que
la concentracin sea mucho ms baja que la del intervalo analizado. Esto ltimo,
aunque cierto, quizs carezca de inters prctico por el valor tan bajo de
concentraciones de que se trata.
Por otro lado, el valor de KS depender del tipo de compuesto orgnico que
se emplee como dador de electrones. Valores de KS para el metanol han sido
reportados entre 0,1 y 72 mgL.27
Cuando se emplea agua residual municipal como sustrato orgnico, expresado como DBO, los valores de KS son mayores que los reportados para el
114

Oxidacin Biolgica. Nitrificacin

metanol, por esta razn, en estos casos, la velocidad de desnitrificacin puede


considerarse que depende de la concentracin de sustrato.28
La influencia del oxgeno en la velocidad de desnitrificacin puede tambin
ser reflejada en la ecuacin (3.55) de la velocidad,

UD

U Dm

S
D
KO
K S  S K D  D K O  SO

(3.56)

donde el subndice (O) representa los parmetros para el oxgeno.


Por otro lado, como la desnitrificacin lleva implcita el consumo de materia
orgnica, su cintica puede ser expresada solo en funcin de la velocidad de
consumo de DBO.
Considerando que la ecuacin que describe la cintica de remocin de DBO
es similar a la 3.19,

qS

qSm

S
KS  S

(3.57)

donde KS tiene el mismo valor que la usada en la ecuacin 3.54.


Entonces la velocidad de remocin de nitrato puede relacionarse con la
velocidad de consumo de DBO mediante una expresin similar a la que se
emplea para relacionar el consumo de oxgeno a la utilizacin de sustrato
orgnico:
UD

1  1,42 Yob

qS
1,42

b
2,86 2,86 d

(3.58)

Yob: Rendimiento observado en funcin de la DQO removida.


El factor 1,42 es el equivalente de oxgeno de la biomasa en trminos de
DQO, mientras que el 2,86 representa la relacin g de O2 (g N_NO3).
En la ecuacin (3.58) el primer trmino describe la fraccin de DQO y
nitrato que son empleados para la sntesis de biomasa. El segundo trmino representa la fraccin de nitrato utilizado en la respiracin endgena.
Al igual que ocurre con la nitrificacin, el proceso de desnitrificacin es
funcin de la temperatura y la concentracin de oxgeno disuelto en el medio.
As la velocidad de desnitrificacin puede expresarse,29
8D = UD 1,09 (t   OD)
(3.59)

8D: Velocidad total de desnitrificacin, kg N-NO3 (kgSSVd) .
UD: Velocidad especfica de desnitrificacin a 20 C.
t: Temperatura del agua residual, C.
OD: Concentracin de oxgeno disuelto en el agua residual, mgL.
115

Menndez Gutirrez, C. y J. Prez Olmo

Teniendo en cuenta lo que representa en cuanto a costo la utilizacin de una


fuente externa de carbono para satisfacer los requerimientos energticos de la
biomasa, en los procesos de desnitrificacin suelen llevarse a cabo de manera
conjunta ambos procesos, nitrificacin-desnitrificacin, para aprovechar la fuente
de carbono existente en el agua residual.
Segn se ha reportado, con este tipo de proceso secuencial se logran velocidades de desnitrificacin de hasta 0,2 kg N-NO3 (kg SSVd).
Ejemplo 3.5
Se producen diariamente 5000 m3 de un agua residual nitrificada. El efluente
del tanque de nitrificacin contiene 30 mgL de NNO3, 2 mgL de oxgeno
disuelto y 0,5 mgL de NNO2.
Asumiendo que el efluente tiene un contenido mximo de NNO3 de
3 mgLy que la DBO5 no excede los 10 mgL, calcule el volumen del reactor
requerido para la desnitrificacin.

UD

U Dm

S
D
KS  S KD  D

suponga que:
KS 25 mgL para DBO5
KD 0,1 mgL para N
Concentracin de N-NO3 en el efluente = 3,0 - 0,5 = 2,5 mgL1

UD

UD

1
Dm

0,6

2,5
10
0,1  2,5 25  10

 kdD , considerando k

dD

0 ,165 d -1

0,07 d

El tiempo de retencin celular lmite se calcula,

0,6

1
Dm

 0 ,07 ;

qDm = 10,52 d1

Aceptando un factor de seguridad de 2, el tiempo de retencin celular de


diseo ser,
T D, diseo = 21 d1
Mediante un balance de nitrgeno para el proceso de desnitrificacin, se
obtiene una ecuacin similar a la 3.42. A partir de ella puede obtenerse,

116

Oxidacin Biolgica. Nitrificacin

Q D0  D
VX

1  k dD D,diseo
D,diseo Yn

Si X = 2 750 mgL1 y Yn = 0,8,


V = 340 m3

Notas bibliogrficas
1 MC CARTY, P. L.7KHUPRG\QDPLFVRIELRORJLFDOV\QWKHVLVDQGJURZWK$GY
in Water Poll. Res., vol. 2, Pergamon Press, EE. UU. 1964.
2 JORGENSEN, P.E., T. ERIKSEN, AND B. K. JENSEN: (VWLPDWLRQRIYLDEOHELRPDVVLQ
wastewater and activated sludge by determination of ATP, Oxygen Utili]DWLRQUDWHDQG)'$+\GURO\VLVWater Res., vol. 26, no. 11, 1992.
3 PREZ, J. Y C. MENNDEZ: Anlisis cromatogrfico de los residuales de pulpa y
papel antes y despus de un tratamiento biolgico, indito, ISPJAE, Ciudad de La Habana, 1986.
4 SERVIZI, J.A. AND R. H. BOGAN: )UHH HQHUJ\ DV D SDUDPHWHU LQ ELRORJLFDO
WUHDWPHQWProceeding, ASCE, vol. 89, sa, vol. 3, no. 17, 1963.
5 ECKENFELDER, W.W. AND'-2&ONNOR: Biological Waste Treatment, Pergamon Press, Oxford, 1961.
6 BENEFIELD, L. AND C. RANDALL: Biological Process Design for Wastewater
Treatment, Prentice Hall Series in Environmental Sciences, EE. UU., 1980.
7 HOOVER, S. R. AND N. PORGES:$VVLPLODWLRQVRIGDLU\ZDVWHVE\DFWLYDWHG
VOXGJH3DUW,,Sewage and Industrial Wastes, vol. 24, no. 3, 1952.
8 MCCABE, B.J. AND W. W. ECKENFELDER:%2'UHPRYDODQGVOXGJHJURZWKLQ
WKH DFWLYDWHG VOXGJH SURFHVV -RXU Water Poll. Control Fed., vol. 33,
no. 3, 1961.
9 WILSON, I. S.:&RQFHQWUDWLRQHIIHFWVLQWKHELRORJLFDOR[LGDWLRQRIWUDGHZDV
WHV Proceding 1st International Conference Water Pollution Research, Pergamon Press, London, 1962.
10 WOHRMAN, K.: Biological Treatment of Sewage and Industrial Wastes,
vol. I, Ed. Mc Cabe, EE. UU., 1956.
11 GRAU, P., M. DOHANYOS, AND J. CHUDOBA:.LQHWLFVRIPXOWLFRPSRQHQWVXEV
WUDWHUHPRYDOE\DFWLYDWHGVOXGJHWater Res., vol. 9, no. 6, 1975.
12 ECKENFELDER, W.W.: Industrial Water Pollution Control, McGraw Hill,
USA 1966.
13 LAWRENCE, A.W. AND P.L. MCCARTY8QLILHGEDVLVIRUELRORJLFDOWUHDWPHQW
GHVLJQDQGRSHUDWLRQ3URF$P6RF&LYLO(QJUV-RXUSan. Eng. Div.
96 (SA3), 1970.
117

Menndez Gutirrez, C. y J. Prez Olmo

14 VAN HAANDEL, A. C. AND G. R. MARAIS:1LWULILFDWLRQDQGGHQLWULILFDWLRQLQWKH


DFWLYDWHGVOXGJH SURFHVVResearch Report no. 42 Departament of Civil Engineering University of Cape Town, 1981.
15 PODUSKA, R. AND J. ANDREWS:'\QDPLFVRIQLWULILFDWLRQLQWKHDFWLYDWHGVOXG
JHSURFHVV-RXUWater Poll. Control Fed., vol. 47, no. 11, 1975.
16 SHERRARD, J. H.:'HVWUXFWLRQVRIDONDOLQLW\LQDHURELFELRORJLFDOZDVWHZDWHU
treatment, Jour. Water Poll. Control Fed., vol. 48, no. 7, 1976.
17 STENSTROM Y S.S. SONG.:(IIHFWRIR[\JHQWUDQVSRUWOLPLWDWLRQRQQLWULILFDWLRQ
LQWKHDFWLYDWHGVOXGJHSURFHVV-RXUWater Poll. Control Fed., vol. 63,
no. 2, 1991.
18 DOMING, A., H. PAINTER, AND G. KNOWLES:1LWULILFDWLRQVLQWKHDFWLYDWHGVOXG
JHSURFHVVJournal of the Institute of Sewage Purification, vol. 4, no.
130, 1964.
19 MC. CARTY, P. L.: Stoichiometry of Biological Reactions Progress in Water
Technology, Pergamon Press, London, 1975.
20 ADAMS, C. E. AND W. W. ECKENFELDER:1LWULILFDWLRQVGHVLJQDSSURDFKRI
KLJK VWUHQJWK DPPRQLD ZDVWHZDWHUV -RXU Water Poll. Control Fed.,
vol. 49, no. 3, 1977.
21 HENZE , M., C. P. GRADY ET AL: IAWPRC Task Group on Mathematical
Modelling for Design and Operation of Biological Wastewater Treatment, Final Report, 1985.
22 HALL, E. AND K. L. MURPHY:(VWLPDWLRQRIQLWULI\LQJELRPDVVDQGNLQHWLFVLQ
ZDVWHZDWHUWater Res., vol. 14, no. 2, 1990.
23 ARGAMAN, Y. AND A. BRENNE: ([SHULPHQWDOHYDOXDWLRQRIPRGHOOLQJDOWHUQDWL
ves and process parameters for the single-sludge nitrogen removal SysWHP3UHVHQWHGLQWKHIAWPRC Specialized Seminar, Copenhagen, Denmark, August, 1985.
24 MENNDEZ, C.: Wastewater Treatment from the Pulp and Paper Industry,
Tesis de Doctorado, Instituto de Tecnologa Qumica, Praga, 1986.
25 SIMPKIN, T.J. AND W.C. BOYLE:7KHODFNRIUHSUHVLRQE\R[\JHQRIWKHGHQL
WULI\LQJHQ]\PHVLQDFWLYDWHGVOXGJHWater Res., vol. 22, no. 2, 1988.
26 FOCHT, D.D. AND A.C. CHANG:1LWULILFDWLRQDQGGHQLWULILFDWLRQSURFHVVHVUH
ODWHGZDVWHZDWHUWUHDWPHQWAdv. Appl. Microbiol, vol. 19, no. 1, 1975.
27 BURDICK, C.R., D.R. REFLING AND H.D. STENSEL:$GYDQFHGELRORJLFDOWUHDW
PHQW WR DFKLHYH QXWULHQW UHPRYDO -RXU Water Poll. Control Fed.,
vol. 54, no. 9, 1982.
28 CHRISTENSEN, M.H. AND P. HARRAMOES:%LRORJLFDOGHQLWULILFDWLRQRIVHZDJH
DOLWHUDWXUHUHYLHZWater Tech., vol. 8, no. 2, 1977.
29 KUGELMAN, I. J. ET AL.$HURELFGHQLWULILFDWLRQLQDFWLYDWHGVOXGJHProc. of
the 1991 Env. Eng. Speciality Conf. Reno, NV. 1991.
118

Lodo Activado

CAPTULO 4

LODO ACTIVADO

4.1. GENERALIDADES
Bajo la denominacin del lodo activado se incluye un conjunto de procedimientos de tratamiento en los que se destaca, como elemento comn, el ntimo
contacto del agua residual con una masa biolgica preformada y en suspensin,
en un tanque aereado. Esta biomasa esta constituida por un cultivo mixto de
microorganismos que forma, junto con otras sustancias orgnicas e inorgnicas
un conglomerado floculento. Mediante este proceso los compuestos orgnicos
contenidos en las aguas residuales son utilizados como sustratos.
La primera informacin que se tiene del empleo del lodo activado para el
tratamiento de aguas residuales data de 1914 en Manchester, Inglaterra.
A diferencia de los cultivos puros en los que las bacterias se mueven libremente,
en el lodo activado estas se encuentran en forma de zooglea, de manera que forma
con facilidad flculos de buenas propiedades de sedimentacin.
Entre los grupos y especies de bacterias que con ms frecuencia se encuentran presentes en los lodos activados, pueden citarse:
Pseudomonas, flavobacterias chromobacterias, azotobacterias, micrococos,
bacilos (b. subtilis, b. megaterium), alcalgenes, y nocardias. En menor proporcin se encuentran los hongos, levaduras y mohos.
Entre las bacterias nitrificantes pueden estar presentes las nitrosomonas y
las nitrobacterias. Es comn que se reporte en lodo activado la presencia de
protozoos tales como vorticelas, opercularias y epistlis entre otros. Otros
microorganismos superiores presentes pueden ser los rotferos.
En los lodos activados se aprecia con mucha claridad el vnculo presapredador en la relacin que existe entre bacterias y protozoos. El papel de las
bacterias es descomponer, por oxidacin, la materia orgnica. En el proceso del
lodo activado las bacterias son los microorganismos ms importantes debido a
que son ellas las responsables de la descomposicin de la materia orgnica contenida en el agua residual a tratar. Los protozoos se alimentan de las bacterias
que se mueven libremente, y as contribuyen a la clarificacin del efluente del
119

Menndez Gutirrez, C. y J. Prez Olmo

proceso del lodo activado, que de otra forma tendra una concentracin ms
elevada de slidos voltiles en suspensin.
Por otra parte, los hongos filamentosos constituyen microorganismos indeseables porque su presencia puede traer como consecuencia dificultades con la
sedimentacin de la biomasa. Estos microorganismos pertenecen a los grupos:
Sphaerotilus, leptomitus, laucothrix, thiothrix, geotrchum, y otros.
La composicin del lodo activado, en lo cuantitativo, depende de la composicin del sustrato que le sirve de alimentacin y de los parmetros tecnolgicos
del proceso: tiempo de retencin hidrulico, tiempo de retencin medio celular,
(edad del lodo) y la remocin especfica de sustrato (carga de lodo).
De acuerdo con su facilidad de sedimentacin, los lodos pueden clasificarse en:

Floculentos.
No floculentos.
Abultados.

Un breve anlisis de las caractersticas fundamentales de cada uno de estos


tipos del lodo arroja los siguientes resultados:

4.1.1. Lodos floculentos


En condiciones ptimas del proceso, el lodo activado forma buenos flculos
de fcil sedimentacin. La formacin de buenos flculos no es necesariamente
una condicin para una alta eficiencia en la remocin de los compuestos orgnicos solubles presentes en un agua residual. Sin embargo, cuando se desea tener
un lodo denso para su recirculacin, y un efluente claro, se requiere que el lodo
activado forme buenos flculos.
El mecanismo de la biofloculacin no es totalmente conocido. La teora ms
razonable supone que el flculo se forma debido al efecto de polmeros
extracelulares de la poblacin bacteriana.1
Desde el punto de vista prctico es importante tener presente los parmetros
tecnolgicos ya mencionados, bajo los cuales se forma un lodo con buenos flculos
y con buena sedimentacin.

4.1.2. Lodos no floculentos


En ocasiones hay condiciones de trabajo bajo las cuales los microorganismos
que constituyen el lodo no forman aglomeraciones, y por tanto, no se logran
flculos sedimentables. Las bacterias permanecen dispersas y mantienen su
individualidad.
120

Lodo Activado

El efluente de estos sistemas se presenta turbio y con relativamente alta


DQO y DBO5 debido al contenido de sustancias que permanecen en suspensin
aun despus de la sedimentacin. No obstante, el contenido de sustancias orgnicas en disolucin puede ser bajo.
En la prctica del tratamiento de aguas residuales, no es usual apreciar
que un lodo pueda ser no floculento en su totalidad. Normalmente, parte del
cultivo mismo forma flculos de fcil separacin por sedimentacin y parte
permanece en forma dispersa, y cuya separacin por sedimentacin simple
se hace ms difcil.
Las diferentes causas por las que un lodo puede ser no floculento pueden
encontrarse en los parmetros tecnolgicos del proceso, y en las caractersticas
de las aguas residuales que son tratadas.
Es comnmente conocido que cuando existe una baja edad del lodo (menor de tres das) o alta carga, el lodo que se forma es propenso a presentar
caractersticas de un cultivo disperso no floculento.2 Como consecuencia de
esta dispersin se produce un aumento en el ndice volumtrico de lodo, en la
medida en que la edad del lodo disminuye a partir de determinado valor.3, 4
(Figuras 4.1 y 4.2).

Fig. 4.1. Influencia de la edad del lodo sobre la dispersin de los slidos.

Fig. 4.2. Relacin entre el ndice volumtrico del lodo (IVL) y la velocidad de sedimentacin.
121

Menndez Gutirrez, C. y J. Prez Olmo

La prdida de la propiedad de formar flculos de un lodo puede deberse a


causas temporales, de forma que cuando cesa la causa el lodo recupera la capacidad perdida. Este trastorno temporal puede deberse entre otros, a cambios
bruscos en el pH, alcalinidad o temperatura del agua. La de-floculacin de carcter permanente responde a otras causas, como la falta de algn nutriente o la
presencia de sustancias txicas.

4.1.3. Lodos abultados


El abultamiento de los lodos trae como consecuencia malas propiedades de
sedimentacin. Desde un punto de vista eminentemente prctico, las caractersticas de sedimentacin y espesamiento de los lodos activados se expresan en
trminos del ndice volumtrico del lodo (IVL). En la medida que el valor del IVL
sea mayor, las caractersticas de sedimentabilidad de los lodos sern menores.
De acuerdo con el ndice volumtrico del lodo estos pueden clasificarse en:

Lodos normales
Lodos ligeros
Lodos abultados

IVL  100 mLg


IVL   P/J
IVL ! 200 mLg

Aunque la clasificacin de los lodos de acuerdo con el ndice volumtrico se


corresponde con el comportamiento que usualmente presentan estos, debe tenerse tambin presente que el valor de este indicador es funcin de la concentracin de los lodos.5
En la figura 4.3 puede apreciarse que esta dependencia es ms significativa
a medida que el ndice volumtrico del lodo es mayor.

Fig. 4.3. Efecto de la concentracin del lodo sobre el IVL.


122

Lodo Activado

El abultamiento puede ser filamentoso o no filamentoso. Las causas de


este ltimo no son muy bien conocidas. Por otro lado desde el punto de vista
prctico este tipo de abultamiento no origina grandes problemas.
La causa del abultamiento debido a los organismos filamentosos puede
estar tanto en las caractersticas de las aguas residuales como en los
parmetros tecnolgicos.
La existencia de alta carga es generalmente propicia para el crecimiento
de bacterias formadoras de flculo, tal como existe en los reactores conocidos como Selectores, desarrollados inicialmente por Chudoba. 6
La carga en los selectores puede ser tal alta como 12-20 kg DBO kg
SSVLM d . 7 No obstante, Kappeler y su mujer han reportado buenos resultados del selector con cargas inferiores a esas. 8

4.1.3.1. Caractersticas de las aguas residuales


El abultamiento de los lodos se observa con ms frecuencia en el tratamiento de aguas residuales que contiene una alta concentracin de sacridos.
El desbalance nutricional puede traer como consecuencia el crecimiento fuera
de control de algunos microorganismos filamentosos. Esto se ha apreciado,
por ejemplo, en residuales con dficit de fsforo.9

4.1.3.2. Parmetros tecnolgicos


La presencia de microorganismos filamentosos es ms frecuente en los
sistemas de baja carga del lodo que en los de alta carga.
Por otra lodo, en reactores de flujo de pistn (poca mezcla longitudinal y
alto gradiente de concentracin de sustrato), el crecimiento de microorganismos filamentosos es ms adverso que en reactores completamente mezclados. Este fenmeno ya fue discutido con anterioridad.

4.2. PARMETROS BSICOS


Hay una serie de parmetros del proceso del lodo activado, que por su
importancia pueden ser denominados bsicos, que deben quedar claramente
definidos antes de trabajar con las expresiones matemticas de este tratamiento. Para la definicin de los parmetros bsicos se toma como referencia el diagrama de la figura 4.4.

123

Menndez Gutirrez, C. y J. Prez Olmo

Simbologa
1. criba
2. desarenador
3. sedimentador primario
4. tanque de aereacin (TA)
5. sedimentador secundario
6. recirculacin de lodo
7. afluente al TA
8. lodo a digestin

So:
S2:
X:
Xr:

DBO DQO afluente


DBO DQO efluente
concentracin de lodo en el TA
concentracin de lodo en la
recirculacin
X2: concentracin de lodo en el
efluente
Qw: flujo de exceso de lodo
Qr: flujo de recirculacin

Fig. 4.4. Esquema de tratamiento.

4.2.1. Tiempo de retencin hidrulico


El tiempo de retencin hidrulico viene dado por la relacin entre el volumen
efectivo del reactor y el flujo o caudal de entrada a l.
Cuando no hay recirculacin:

V
Q0

(h d)

(4.1)

Cuando existe recirculacin Qr :

V
Q0  Qr

124

(h d)

(4.2)

Lodo Activado

4.2.2. Relacin de recirculacin


R

Qr
Q0

(4.3)

El tiempo de retencin hidrulico puede ser expresado en funcin de la relacin de recirculacin:

V
Q0 1  R

(4.4)

4.2.3. Carga volumtrica


Se denomina carga volumtrica a la masa de sustrato que se aplica, expresado como DBO o DQO, por unidad de volumen del tanque de aereacin, en la
unidad de tiempo.
La carga volumtrica, tambin llamada en ocasiones carga orgnica, no es
ms que el flujo msico de DBO DQO por unidad de volumen efectivo del
reactor.

Bv

Q0 S0
V

S0

kg m

 3 1

(4.5)
(4.6)

Las ecuaciones 4.5 4.6 se utilizarn dependiendo de si hay o no recirculacin.

4.2.4. Razn de carga volumtrica


La razn de carga volumtrica es un parmetro que puede asociarse, en un
sentido fsico, con la velocidad msica de remocin del sustrato por unidad de
volumen efectivo del tanque de aereacin.

Bv

Q0 S0  S2
kg m 3d 1
V

(4.7)

o sea,

Bv

S0  S2
kg m 3d 1

125

(4.8)

Menndez Gutirrez, C. y J. Prez Olmo

4.2.5. Carga de lodo


Se conoce bajo la denominacin de carga del lodo al flujo msico de sustrato,
ya sea DBO o DQO que se aplica al tanque de aeracin, por unidad de slido
orgnico en el lodo.

Q0 S0
kg kg 1d 1
VX

Bx

(4.9)

Combinando las ecuaciones 4.9 y 4.5, se obtiene la ecuacin 4.10, que es


otra forma de expresar la carga del lodo:

Bv
X

Bx

(4.10)

Cuando se utiliza la recirculacin, la carga del lodo puede calcularse segn:

Q0  Qr Sm

Bx

kg kg

VX

1 1

(4.11)

4.2.6. Razn de carga de lodo


Este parmetro tambin se conoce como la relacin F/M (alimento/microorganismo), y en esencia no es otra cosa que la remocin especfica de la DBO o
DQO.
Q0 S0  S 2

Bx

VX

kg kg

1 1

(4.12)

La ecuacin 4.12 puede expresarse de forma ms sencilla:

%x

S0  S2
X

kg kg

1 1

(4.13)

o tambin, combinando las ecuaciones 4.13 y 4.7:

%x

Bv
X

kg kg

1 1

(4.14)

4.2.7. Edad del lodo


El tiempo de retencin medio celular (TRMC) o edad del lodo es hoy da el
parmetro ms importante de los utilizados para el control de los procesos del
lodo activado.
126

Lodo Activado

La edad del lodo puede calcularse a partir de la relacin que existe entre la
masa del lodo que est presente en el sistema y la masa del lodo que se extrae
diariamente.

VX  G
X w Qw  X 2 Q0  Qw

(4.15)

donde:
G: Masa del lodo en el sedimentador secundario y en las tuberas. Generalmente no se comete mucho error si se supone que G = 0
Si los lodos del proceso forman buenos flculos y poseen buenas propiedades de sedimentacin, puede suponerse que la cantidad del lodo o biomasa
que se pierde en el sobrenadante del sedimentador secundario es despreciable cuando se compara con la que se extrae por la corriente Qw:
X 2 Q0  Qw X w Qw

(4.16)

Cuando se est en presencia de lodos no floculentos, la prdida de biomasa


en el sobrenadante del sedimentador secundario puede ser apreciable, pudiendo
llegar hasta 40 y 60 % del lodo total que se extrae, y no puede obviarse la
prdida del lodo por este concepto.
En el caso en que las condiciones de los flculos son tales que se cumple la
expresin 4.16, se tiene que:

VX
Qw X w

(4.17)

Por otro lado, si la extraccin del lodo se realiza directamente del tanque de
aeracin:
X

Xw

la ecuacin 4.17 se transforma:

V
Qw

(4.18)

Un aspecto importante a tener en cuenta es que en aquellos casos en los


que no exista recirculacin, se cumple que el tiempo de retencin medio celular
es igual al tiempo de retencin hidrulico.
Tx = T
cuando R = 0
(4.19)

127

Menndez Gutirrez, C. y J. Prez Olmo

4.3. PRINCIPALES MODIFICACIONES TECNOLGICAS


El proceso del lodo activado puede definirse como aquel en el que la
biomasa floculenta est circulando continuamente en ntimo contacto con la
materia orgnica que se desea eliminar o remover, en presencia de oxgeno.
Usualmente el oxgeno es suministrado a partir de las burbujas de aire que
se incorporan a la mezcla lodo-lquido (licor mezclado) en forma turbulenta,
mediante el empleo de aereadores mecnicos o de aire comprimido. El proceso implica una etapa de separacin slido-lquido. Parte del slido que se
obtiene en esta segunda etapa o sedimentacin, es recirculado y mezclado
con el agua residual que se alimenta a la etapa de aereacin. En el tanque de
aereacin ocurre:
1. La adsorcin y coagulacin rpida de los compuestos orgnicos solubles
y coloidales por la accin de la biomasa.
2. Una progresiva oxidacin de los compuestos adsorbidos y otros que son
continuamente removidos de la solucin.
3. La sntesis de nueva biomasa, en la que se emplea como sustrato la materia orgnica contenida en el agua residual.
4. La oxidacin y dispersin de las partculas del lodo cuando la aeracin se
extiende por periodos prolongados.
Los diferentes cambios que ocurren en los procesos del lodo activado se
muestran en la figura 4.5. As por ejemplo, para un albaal cualquiera en el
que la mayor fraccin de la DBO est en forma suspendida y coloidal, el
tiempo de retencin necesario para la remocin rpida de la DBO por
adsorcin de los coloides y floculacin de los slidos en suspensin se representa por t1 . Como se observa en la figura 4.5, para el tiempo t2 ya la materia orgnica es eliminada por oxidacin y sntesis. Esto ocurre entrada ya la
zona de respiracin endgena. Si se desea que ocurra nitrificacin, el tiempo
de retencin debe ser lo suficientemente alto, t 3 , como para que las bacterias nitrificantes puedan crecer y existir en nmero lo suficientemente grande como para ejercer su accin oxidante sobre el NH 3 en un grado tal que
sea detectable.
A modo de resumen puede decirse que los mecanismos que ocurren
durante el proceso del lodo activado pueden enumerarse como sigue:
1. Remocin inicial de slidos suspendidos y coloidales por aglomeracin
fsica y floculacin, as como por adsorcin sobre los flculos biolgicos.
Estos compuestos orgnicos de estructura generalmente algo compleja
son hidrolizados y descompuestos por la accin biolgica durante la
aereacin.
128

Lodo Activado

2. Una remocin, relativamente lenta, de los compuestos orgnicos solubles. Esta accin, que es debida a la actividad de microorganismos, da
lugar a la formacin de productos finales oxidados, CO 2 y H2 O y a la
sntesis de nuevos microorganismos, constituyentes de la biomasa.
3. Oxidacin del amonaco a nitrato por los organismos nitrificantes.
4. Oxidacin de la biomasa obteniendo como producto final CO2 , NH 3 ,
H 2 O y fsforo. Adicionalmente se obtiene un residuo no biodegradable,
constituido tanto por sustancias que con estas caractersticas forman
parte de la biomasa, como por una fraccin contenida en el afluente, y
que pasa a travs del tratamiento sin alteracin, segn lo discutido en el
epgrafe 3.3.1.

Fig. 4.5. a) Remocin de DBO. b) Biomasa. c) Oxgeno utilizado.


d) NH3. E: NO3.
En los ltimos sesenta aos se han desarrollado numerosas modificaciones
al proceso del lodo activado. En la tabla 4.1 se reportan algunas de las caractersticas ms importantes de estas modificaciones.

129

Menndez Gutirrez, C. y J. Prez Olmo

Tabla 4.1. Diferentes variantes del proceso del lodo activado (los parmetros
de carga estn expresados como DBO5 y slidos suspendidos
voltiles, SSV)

Variantes

Bv
(kgm-3d-1)
Sistemas de baja carga
0,1-0,3
Sistemas de carga
0,5-1,5
media
Estabilizacin por
contacto
Sistemas de alta carga
! 1,5
Lodo activado en dos
etapas:
Primera etapa
10
Segunda etapa
0,5-1,0

Bx
(kgkg-1d-1)
0,05-0,10
0,2-0,5

Tx
(d)
! 25
3-15

T
(h)
24-72
4-12

! 0,5

3

1-2

1-2
0,2-0,5

1
3-15

1
4-6

4.3.1. Sistemas de baja carga


Estos sistemas son diseados para tratar pequeos volmenes de aguas
residuales de origen domstico o industrial, siempre que no sobrepasen los
7000 m3 d .
Los sistemas de baja carga requieren una edad del lodo alta, veinticinco
das o ms, para promover la oxidacin de la biomasa que se produce durante la sntesis. Tericamente la pequea cantidad del lodo que se produce en
estos sistemas est constituida fundamentalmente por sustancias no
biodegradables. Debido a que los sistemas de baja carga por lo comn no se
disean con sedimentacin primaria, en el tanque de aereacin existir una
mezcla del lodo primario y biomasa.
Como el proceso de estabilizacin aerobia de los lodos es lento, el tiempo de retencin hidrulico debe ser tambin mucho mayor que el requerido
para la remocin de los constituyentes orgnicos solubles y coloidales del
agua residual.
En la prctica estos sistemas, conocidos comnmente como de aeracin
extendida, se disean como unidades compactas (figuras 4.6 y 4.7) o como
canales de oxidacin (figura 4.8).

130

Lodo Activado

Fig. 4.6. Esquema de unidad compacta del lodo activado. Aereacin


extendida. 1: afluente. 2: efluente, 3: seccin de aereacin. 4: sedimentacin y recirculacin interna de lodo.

Fig. 4.7. Esquema de unidad compacta. Aereacin extendida.

Fig. 4.8. Canal de oxidacin. Variante de aereacin extendida.

4.3.2. Sistemas de carga media


Estos son los sistemas del lodo activado ms frecuentemente empleados
y pueden disearse con rgimen de flujo de pistn o de mezcla completa.

131

Menndez Gutirrez, C. y J. Prez Olmo

Los sistemas de flujo a pistn tienen un diseo de tanque de aereacin llamado corredor. Sus dimensiones varan dentro de los siguientes lmites:
Ancho:
Profundidad:
Longitud:

(4-12) m
(4-6) m
(20-120) m

Los sistemas de mezcla completa tienen la ventaja de amortiguar los efectos de las variaciones de las concentraciones de entrada al tanque de aereacin
y disminuir, dentro de ciertos lmites, las posibles consecuencias de la presencia
de sustancias txicas.
Entre los sistemas de carga media se destacan el convencional, la aereacin
escalonada, la dosificacin gradual y la estabilizacin por contacto.
Sistema convencional
El proceso que ocurre en un sistema convencional tiene cuatro etapas:
1. Sedimentacin primaria para la remocin de las sustancias orgnicas y slidos inorgnicos sedimentables cuando estn presentes en el agua residual.
2. Aeracin de la mezcla de agua residual y biomasa activa (licor mezclado).
3. Separacin del lodo por sedimentacin.
4. Recirculacin nuevamente a la etapa de aeracin de una fraccin del lodo
separado en la tercera etapa (figura 4.9).

Fig. 4.9. Variante del esquema de un lodo activado convencional.


Aeracin escalonada
Cuando se emplean reactores de flujo de pistn en el tratamiento de aguas
residuales, la demanda de oxgeno es variable a lo largo de toda su longitud.
Si el suministro del oxgeno es uniforme, puede quedar en defecto a la entrada del tanque de aeracin y en exceso a la salida (figura 4.10.a).
Con el objetivo de suministrar en cada momento el oxgeno requerido para
satisfacer la demanda, surgi la variante de la aereacin escalonada (figura
4.10.b).
132

Lodo Activado

Fig. 4.10. Relacin demanda-suministro de oxgeno. a) Flujo de pistn.


b) Aereacin escalonada. c) Dosificacin gradual. d) Mezcla completa.

Fig. 4.11. Diferentes variantes del lodo activado. a) Sistema convencional. b) Dosificacin gradual. c) Aereacin escalonada.
d) Estabilizacin por contacto.
1. Sedimentador primario. 2. Tanque de aereacin.
3. Sedimentador secundario. 4. Entrada de aire.
133

Menndez Gutirrez, C. y J. Prez Olmo

Dosificacin gradual
Esta variante es conocida como sistema Gould. Para uniformar la carga
aplicada y la velocidad de consumo de oxgeno a lo largo del tanque de
aereacin, se dosifica la alimentacin, introducindola por varios puntos a lo
largo de la longitud del tanque (figura 4.11).
El tanque de aereacin se divide por medio de deflectores en tres o ms
cmaras conectadas entre s en serie.
El lodo que se recircula se introduce en la primera de las cmaras.
Este proceso se opera de acuerdo con un rgimen hidrulico de flujo a
pistn y el sistema de aereacin es a base de difusores. De esta manera se
logra una eficiencia de remocin entre 85 y 95 %. 10
Estabilizacin por contacto
Cuando el agua residual tiene una alta concentracin de contaminantes
en forma suspendida no sedimentable y en forma coloidal, puede obtenerse
una buena eficiencia en la remocin por adsorcin en el lodo bien aereado y
previamente estabilizado.
Esta variante del lodo activado se basa en el postulado de que la eliminacin de la DBO tiene lugar en dos etapas. La primera es la fase de adsorcin
ya mencionada, llamada fase de contacto y la segunda, fase de oxidacin,
en la que se lleva a efecto la asimilacin metablica de la materia orgnica
(figura 4.11.d).

4.3.3. Sistemas de alta carga


Estos sistemas se disean raras veces debido a que con una eficiencia
de remocin entre 60 y 75 %, producen un efluente de caractersticas
comnmente poco deseables debido a que no estn adecuadamente
estabilizados, y que generalmente, el lodo no presenta buenas propiedades
de floculacin.
Comnmente los sistemas de alta carga se utilizan en dos etapas. La
primera de las etapas para este tipo de sistema opera con elevada carga
(Bv = 10 kg m d , Bx NJNJ d ) y la segunda de las etapas tiene
caractersticas de cargas similares a la de los sistemas convencionales
(figura 4.12).
Los sistemas de alta carga son utilizados fundamentalmente en el tratamiento aerobio de residuales industriales de alta DBO.

134

Lodo Activado

Fig. 4.12. Proceso del lodo activado en dos etapas.

4.4. APLICACIN DE LOS MODELOS CINTICOS


Los modelos cinticos que describen la remocin de los compuestos orgnicos pueden ser utilizados con buena efectividad en la prediccin de la eficiencia
de los procesos del lodo activado.

4.4.1. Sistemas de flujo de pistn


Para sistemas discontinuos (a templa) o continuos a flujo de pistn, la ecuacin 3.24 puede expresarse:
S

S0

dS
Sn

kn

X0
S0

dt

(4.20)

donde:
X0: Concentracin inicial de biomasa (mgL).
S0: DBO inicial (mgL).
S: DBO en el instante t (mgL).
n: Orden de la reaccin.
kn: Constante de velocidad de remocin.
Si la reaccin es de orden cero (n = 0), la ecuacin 4.20 integra:
S0  S

k0 X 0 t

(4.21)

Por tanto, la fraccin removida viene dada por:

S0  S
S0

k0

X0
t
S0

(4.22)

Para primer orden (n = 1), de la ecuacin 4.20 se obtiene:


S
S0

X
exp  k1 0 t
S0

135

(4.23)

Menndez Gutirrez, C. y J. Prez Olmo

S0  S
S0

X
1  exp k1 0 t
S0

(4.24)

Para segundo orden (n = 2):


S
S0

S0  S
S0

1
1  k2

X0
t
S0

X0
t
S0
X
1  k2 0 t
S0

(4.25)

k2

(4.26)

Las unidades de la constante de velocidad de remocin en cada caso son:


k0: gg h.
k1: Lg h.
k2: L g h.

4.4.2. Sistemas de mezcla completa


En los sistemas completamente mezclados, la concentracin de salida es
prcticamente igual a la concentracin del sustrato soluble remanente en el tanque de aereacin.
Para obtener la expresin que permita predecir la eficiencia de remocin en
sistemas completamente mezclados se requiere de un balance de sustrato alrededor del sistema (figura 4.13):

Fig. 4.13. Variante de esquema de un reactor de mezcla completa.


136

Lodo Activado

dS

dt
en estado estacionario

dS
dt

S Q S
Q0 S0
 kn X  0
V
V
S0

(4.27)

0 , y la ecuacin 4.27, queda:


S0  S
X
 kn n S n

S0

(4.28)

donde:

V
Q0

X: Concentracin de SSVLM (mgL).


Resolviendo la ecuacin 4.28 para:
n = 0; n = 1; n = 2;
se obtiene:

S0  S
S0

n=0

n=1

n=2

S0  S
S0

S0  S
S0

k0

S0

(4.29)

S0
X
1  k1

S0
k1

S0
X
2 k2

S0

1  4 k2

(4.30)

1
(4.31)

Cuando se compara las ecuaciones desarrolladas para sistemas de flujo de


pistn y completamente mezclado, puede apreciarse que para igualdad de condiciones (S0, kn, X), solamente para n = 0 la prediccin de la eficiencia de remocin arroja resultados similares (ecuacin 4.21 y 4.29). Para n ! 0, las eficiencias
de remocin que se predicen para sistemas de flujo de pistn siempre son mayores que para sistemas completamente mezclados.
137

Menndez Gutirrez, C. y J. Prez Olmo

Sin embargo, esta conclusin terica rara vez se confirma en la prctica.


La razn ms probable de que en la realidad la eficiencia de los sistemas
completamente mezclados no sea menor que en los sistemas de flujo de pistn
est en el mecanismo de remocin de sustrato multicomponente.10
Durante toda la remocin, la DBO sigue una curva (cintica de orden
distinta a cero). Esta curva es la resultante de la suma de remociones lineales (orden 0) de los componentes individuales. Esta es posiblemente la razn
por la que reactores del lodo activado completamente mezclados tienen la
misma eficiencia de purificacin que los reactores de flujo de pistn en igualdad de condiciones.
Matemticamente esto significa que las constantes de remocin de las
ecuaciones (4.30) y (4.31) deben ser mayores que la de las ecuaciones (4.24)
y (4.26), respectivamente, lo que ha sido demostrado experimentalmente. 11
Si S0 y S de las ecuaciones anteriores se expresan en unidades de DQO,
usualmente se sustrae aquella parte de la demanda qumica de oxgeno, Sn ,
que pertenece a los compuestos no biodegradables, (S0 Sn ; 66n ).
Para aguas residuales municipales los valores de Sn varan en el intervalo
comprendido entre 2 y 40 gm. En residuales industriales este resultado puede
ser mayor. En la prctica los valores de DQO y DBO se emplean indistintamente obviando la mayora de las veces la fraccin no biodegradable contenida en el
agua residual.

4.5. FACTORES QUE INCIDEN EN LA EFICIENCIA


DE PURIFICACIN
Los factores que inciden en la eficiencia de purificacin pueden ser clasificados en dos grupos:
1. Factores o parmetros bsicos.
2. Factores o parmetros derivados.
Factores o parmetros bsicos
Entre los factores o parmetros bsicos ms importantes pueden distinguirse:
a)
b)
c)
d)

Tiempo de retencin.
Concentracin de biomasa.
Temperatura.
Concentracin de contaminantes.

138

Lodo Activado

Factores o parmetros derivados


a) Carga volumtrica.
b) Edad del lodo.

4.5.1. Tiempo de retencin


El tiempo de retencin es un factor determinante en la eficiencia de purificacin del proceso del lodo activado. Tal como se refleja en la tabla 4.1, de
acuerdo con la variante del lodo activado que se utilice, as ser el tiempo de
retencin que se requiera.
El tiempo de retencin es un parmetro importante por cuanto define, entre
otros factores, el volumen del tanque de aereacin.
Para todo sistema existe un tiempo de retencin denominado crtico (Tc) por
debajo del cual la eficiencia del proceso disminuye abruptamente (figura 4.14).

Fig. 4.14. Influencia del tiempo de retencin en la eficiencia.


Este valor crtico depende en general de:
a) Carcter y concentracin de los compuestos orgnicos en el agua.
b) Concentracin de SSVLM en el tanque de aeracin.
c) Temperatura.
El tiempo de retencin crtico vara adems, de un residual a otro. Como
criterio, puede considerarse que para las aguas residuales municipales el qc es
cercano a las cuatro horas.

4.5.2. Concentracin y actividad del lodo activado


La concentracin de microorganismos en el tanque de aeracin tiene, al
igual que el tiempo de retencin, un efecto importante en la eficiencia de purifi139

Menndez Gutirrez, C. y J. Prez Olmo

cacin. Esto puede apreciarse en la figura 4.15. El grfico de esta figura fue
obtenido en un proceso a templa a partir del cual puede concluirse que existe
una relacin lineal entre la velocidad de remocin de DBO y la concentracin
inicial del lodo en el sistema.

Fig. 4.15. Dependencia de la velocidad de remocin de DBO con la


concentracin inicial de biomasa (lactosa como sustrato).
Para sistemas continuos, la relacin general entre la eficiencia y la concentracin de SSVLM (X) se ilustra en la figura 4.16, en la cual puede apreciarse
que existe un valor de X (Xc) por encima del cual la eficiencia prcticamente no
se incrementa. Este valor crtico depende de:
a) Tiempo de retencin en el tanque de aeracin.
b) Temperatura.
En todo sistema del lodo activado existe una concentracin crtica de X.
En condiciones dadas (T , ' S, T), cualquier cambio en la concentracin
de X solamente puede lograrse mediante un cambio en la edad del lodo.
La concentracin de biomasa en un reactor que trabaja en condiciones
de flujo continuo sin recirculacin est dado por la velocidad de crecimiento.
Concentraciones de biomasa por encima de este valor solo pueden lograrse
mediante la recirculacin de lodo, o sea, aumentando la edad del lodo. Sin
embargo, en la medida en que la edad del lodo se incrementa este envejece
y disminuye la fraccin de clulas activas en l. La relacin existente entre
la fraccin de clulas vivas y la edad del lodo se ilustra en la figura 4.17.
Al aumentar la edad del lodo se incrementa la concentracin de biomasa,
pero la fraccin activa de la misma disminuye.12 Esto puede traer como consecuencia que, a partir de un cierto valor de edad del lodo, no se obtengan beneficios por aumentos posteriores de esta debido a que la actividad total de la biomasa
por unidad de volumen no experimentar cambios apreciables. Esta es la razn
140

Lodo Activado

por la que la eficiencia de purificacin se eleva con el aumento de la concentracin del lodo, hasta un valor que se corresponda con la concentracin crtica, y
que incrementos por encima de esta concentracin no se traduzcan en aumentos en la eficiencia.

Xc

Fig. 4.16. Influencia de la concentracin de SSVLM en la eficiencia de


purificacin.

Fig. 4.17. Relacin entre la edad del lodo y la fraccin de clulas activas.
Sin embargo, si el sistema del lodo activado est sobrecargado, es decir,
est siendo operado a bajos tiempos de retencin, entonces un incremento
de la concentracin del lodo dar como resultado un aumento de la eficiencia de purificacin.
Teniendo en cuenta el efecto que tiene sobre la eficiencia de purificacin la
relacin que existe entre el tiempo de retencin (T ) y la concentracin de biomasa
(X), es que se defini la carga de lodo:

Bx

S0
(kg kgd)
X
141

Menndez Gutirrez, C. y J. Prez Olmo

No se reconocen evidencias de una posible relacin entre la carga del lodo


y la eficiencia de purificacin. De existir esta relacin se obtendra la misma
eficiencia de purificacin para distintos pares de valores de X y T (X, T ); (X/2,
2T); (2 X, T /2), etc.

4.5.3. Temperatura
La temperatura es otro de los parmetros que inciden de manera importante
en la eficiencia de purificacin.
Similarmente a lo que ocurre en las reacciones qumicas, en los procesos
biolgicos el efecto de la temperatura puede ser expresado a travs de una
constante de velocidad en la que se cumpla la ecuacin de Arrhenius:
K = A exp (-E/RT)
(4.32)
donde:
K: Constante de velocidad cuyas unidades depende del orden de la reaccin.
E: Energa de activacin.
R: Constante de los gases.
T: Temperatura absoluta.
Aplicando logaritmo a la ecuacin 4.32:

ln K

E
 ln A
RT

(4.33)

Para dos temperaturas, T1, T2 y sus correspondientes constantes, K1, K2


puede obtenerse:

ln

K1
K2

E
R

T1  T2

T1 T2

(4.34)

En la prctica del tratamiento de aguas residuales el intervalo de temperatura no es muy amplio, por tanto, el producto T1T2 es aproximadamente constante
y la ecuacin 4.34 puede escribirse:

K1
K2

T1 T2

en la que
n = exp (E/RT1T2)
142

(4.35)

Lodo Activado

La ecuacin 4.35 generalmente se utiliza de la forma:


kt

20 qC

t  20

(4.36)

El valor del coeficiente Q vara para diferentes procesos en el intervalo de 1,01


a 1,10. Wuhrman13 y Eckenfelder14 han reportado valores para Q de 1,074
y 1,085, respectivamente. Precisando ms estos valores, Wuhrman15 ha comprobado que para Bx  0,5 kgkgd se obtiene que el valor del coeficiente Q es
aproximadamente 1,0 Por otro lado, para valores de carga del lodo superiores a
0,5 (Bx ! 0,5 kgkgd) el valor de Q est comprendido entre 1,0-1,04
Los valores ms frecuentemente utilizados son 1,035 y 1,047.11
El efecto de la temperatura sobre la eficiencia de purificacin depende en
cierta medida de los valores del tiempo de retencin y la concentracin de los
slidos suspendidos voltiles del licor mezclado. Hay evidencias que apuntan
que a medida que son mayores estos dos parmetros, el efecto de la temperatura tiene menos importancia.

4.5.4. Composicin y concentracin


La eficiencia de purificacin, expresada en %, depende en cierta medida de
la concentracin de los compuestos orgnicos presentes en el agua residual.
Esto puede ilustrarse mediante un ejemplo. Si la DBO disminuye desde 1000
gm hasta 10 gm, eso significa una eficiencia de 99 %. Para obtener esa
misma eficiencia con un agua residual que tiene una concentracin en el influente
de 100 gm, se requiere que la DBO decrezca hasta 1 gm, y esto es prcticamente
imposible. Por tanto, la eficiencia decrece al disminuir la contaminacin orgnica. Este fenmeno recibe el nombre de efecto de la concentracin.
Por otro lado, del ejemplo anterior puede deducirse que la eficiencia de
remocin reportada en por ciento no brinda una informacin suficiente sobre el
comportamiento del proceso del lodo activado. Una visin clara de este proceso
solo puede obtenerse comparando los valores absolutos de DBO y DQO en el
afluente y efluente.
La eficiencia de purificacin puede afectarse adems como consecuencia de
un desbalance nutricional del agua residual a tratar. De estos los ms importantes
son el nitrgeno y el fsforo. Mientras las aguas residuales municipales contienen
estos elementos en exceso, muchos residuales industriales carecen de ellos. Para
propsitos de proyecto, la cantidad necesaria de estos elementos puede estimarse
aproximadamente utilizando la siguiente relacin emprica DBO5: N: P = 100: 5: 1.
Esta relacin de DBO5: N: P se cumple mejor para sistemas de carga media. Para sistemas de baja carga la demanda es menor ya que la demanda de
nutrientes aumenta en la medida que se incrementa la carga del sistema.
143

Menndez Gutirrez, C. y J. Prez Olmo

4.6. PRESENCIA DE COMPUESTOS ORGNICOS


EN EL EFLUENTE
Los compuestos orgnicos degradables en el efluente secundario pueden
dividirse en dos grupos:
a) slidos o compuestos suspendidos (SS).
b) slidos solubles o disueltos (SD).
La concentracin de compuestos solubles depende del comportamiento del
proceso del lodo activado. Por otro lado el contenido de slidos suspendidos en
el efluente est determinado en gran medida por la eficiencia del sedimentador
secundario, que rara vez alcanza valores mayores del 95 %.
Los sedimentadores secundarios operando en buenas condiciones
generalmente producen un efluente con un contenido de slidos suspendidos
entre 10 y 30 g m.
Estos slidos estn constituidos por un flculo fino del lodo activado. Los
flculos contienen microorganismos vivos que consumen oxgeno y que al mismo tiempo su presencia en el agua representa un aporte de DBO debido a su
naturaleza orgnica.
La DBO del efluente de un tratamiento biolgico es por tanto el resultado de
la suma de la DBO ejercida por los compuestos orgnicos disueltos y por los
slidos suspendidos:
DBO 5 total

DBO 5 disuelto  DBO 5

suspendido

(4.37)

Es posible obtener una relacin entre la concentracin de slidos suspendidos en el efluente (X2, ver figura 4.4) y la DBO ejercida por ellos:

DBO5 s

0,86 f o
X2
1  0,066 x

(4.38)

donde:
DBO5)s = DBO5 suspendida (mg L).
T x: Edad del lodo (d).
fo: Fraccin voltil de los slidos suspendidos.
X: slidos suspendidos totales (mg L).
Como ya se ha mencionado, la fraccin voltil de los slidos suspendidos en
funcin de la edad del lodo (figura 4.18).13 La expresin analtica de esta relacin emprica es:
fo

SSV
SS
144

0 ,85
x 0 ,1

(4.39)

Lodo Activado

Por otra parte, si:

0,86 f o
1  0,066 x

(4.40)

sustituyendo 4.40 y 4.38 en 4.37 se obtiene:

DBO5 T

DBO5 d  K X 2

(4.41)

Fig. 4.18. Fraccin voltil de los slidos suspendidos y la edad del lodo.
El coeficiente K vara generalmente entre los valores 0,2 y 0,7 en dependencia de la edad de lodo.
Para sistemas de baja carga, suponiendo T x = 30 d
f0 = 0,60 y la ecuacin 4.41 queda:

DBO5 T

DBO5 d  0,173 X 2

(4.42)

Para sistemas de carga media, suponiendo T x = 5 d. f0 = 0,72 y la ecuacin


4.41 queda:

DBO5 T

DBO5 d  0,465 X 2

(4.43)

Para sistemas de alta carga, suponiendo T x = 1 d, f0 = 0,85 y de la ecuacin


4.41 se obtiene:

DBO5 T

DBO5 d  0,686 X 2

(4.44)

Si en lugar de trabajar con la DBO5 el sistema se est analizando en


funcin de la DBO ltima o de la DQO, la concentracin de estas que aportan los slidos suspendidos del efluente secundario puede calcularse:14

DBOu s

1,42 f o X 2

145

(4.45)

Menndez Gutirrez, C. y J. Prez Olmo

Cuando el valor de X2 se toma en funcin de los slidos suspendidos


voltiles en lugar de los slidos suspendidos, el valor de f 0 es igual a la unidad en todos los casos.
De todo lo anterior puede concluirse que los valores totales de DBO5 y
DQO en el efluente secundario tienen una dependencia significativa de los
flculos finos que salen en el lquido claro del sedimentador secundario.
Algunos autores plantean que en los efluentes de los sistemas de carga
baja y media 50 a 75 % de la DBO 5 ) total es producida por los slidos
suspendidos. Su remocin o disminucin incide en una mejor calidad en las
caractersticas del efluente.15

4.7. PRODUCCIN DE BIOMASA Y LODO


Las aguas residuales contienen compuestos orgnicos disueltos y slidos
suspendidos, tanto de origen orgnico, como inorgnico. Aunque los slidos suspendidos se remueven en el tanque de sedimentacin primaria, esta nunca es
completa.
La eficiencia de la sedimentacin vara en un intervalo entre (50 y 70) %.
Esto significa que una fraccin de los slidos suspendidos es alimentada al tanque de aereacin.
Una parte de los slidos suspendidos que entran al tanque de aereacin es
degradable biolgicamente, y otra parte no lo es. La fraccin degradable ejerce
DBO y puede ser considerada como sustrato. Los slidos no degradables, tanto
de origen orgnico como inorgnico, son denominados slidos inertes. Estos
slidos inertes atrapados en el tanque de aeracin, forman parte del lodo activado. El lodo activado es, por tanto, una mezcla del lodo biolgico (biomasa) y
slidos suspendidos inertes.
Solamente en aquellos casos en que el agua residual no contenga slidos
suspendidos inertes, el lodo activado estar constituido en su totalidad por biomasa.
Para simplificar el tratamiento matemtico de los procesos del lodo activado, generalmente no se toman en consideracin los slidos suspendidos que
entran al tanque de aereacin y se supone que la biomasa que entra con el
residual es igual a cero.

4.7.1. Concentracin de biomasa y su produccin


Como una primera aproximacin al problema, que muchas veces no se
aleja de lo que para fines prcticos puede suponerse, se considerar que
todos los slidos a la entrada, disueltos y suspendidos, estn reflejados en la
DBO DQO.
146

Lodo Activado

Fig. 4.19. Esquema de un proceso del lodo activado para balance


de masa.
Tomando como referencia el esquema de la figura 4.19 y haciendo un balance de biomasa total:

Cambio total en Incremento


Disminucin Disminucin
el TK
= debido al + debido a la + debido a la +
crecimiento
muerte
eliminacin
del exceso
Disminucin
por lo que
(4.46)
(4.46)
sale en el
efluente
Cada uno de los trminos de la ecuacin 4.46 puede plantearse:
Cambio total en el tanque:

dX b
dt

Incremento debido al crecimiento:


Disminucin debido a la muerte:

dX b
dt

dX b
dt

Y

dS
dt

S0  S2
Q0
V

 kb X b

donde:
Y: Rendimiento o coeficiente de produccin de biomasa (kg kg).
kb: Constante de autooxidacin (kg kgd).
Teniendo en cuenta estos trminos, la ecuacin 4.46 queda:
(4.47)

V
Considerando que: Q
0

147

Menndez Gutirrez, C. y J. Prez Olmo

S0  S2

Qw X b,w  Q0  Qw X b,2
V Xb

1
x

dX b
0 , puede simplificarse la ecuacin 4.47 y
dt
despejar el trmino Xb que representa la concentracin de biomasa en el reactor
o tanque de aeracin.
y que en estado estacionario

Y x S
1  kb x

Xb

(4.48)

La ecuacin 4.48 brinda la relacin entre la concentracin de biomasa en


estado estacionario y las variables edad del lodo, tiempo de retencin y remocin de DBO una vez que son conocidos el rendimiento (Y) y la constante de
autooxidacin (kb).
En la tabla 4.2 se ofrecen algunos valores tpicos de estos dos parmetros.
Tabla 4.2. Valores tpicos de rendimiento y constante de autooxidacin en un
proceso aerbico16
Residual
Municipal
Industria
Azucarera
Papel
Residual
Lcteo

Y
(base DBO)
0,4 0,8

0,35 0,06

Y
(base DQO)
0,3 0,4
0,33

kb
-1
(d )
0,04 0,08
0,04

0,36
0,2 0,4

0,07
0,04 0,06

No obstante que en la literatura se pueden encontrar valores de Y y kb para


el tratamiento de distintos residuales, siempre que se pueda, estos deben ser
hallados a nivel de planta piloto o laboratorio mediante un procedimiento relativamente sencillo tal como se describir en el epgrafe 4.8. basta con destacar
ahora que la ecuacin 4.47 puede transformarse para obtener:

1
x

Y Bx  kb

donde:

%x

S
Xb
148

(4.49)

Lodo Activado

Al graficar el inverso de la edad del lodo en funcin de 'Bx (figura 4.20), se


obtiene:

Fig. 4.20. Clculo del rendimiento Y y la constante de auto-oxidacin kb .


La ecuacin 4.49 que ha sido desarrollada a partir de un balance de
materiales, fue obtenida hace ms de 40 aos de forma emprica por
Heukelekian. 17
La cantidad de biomasa producida puede calcularse a partir de una modificacin de la ecuacin 4.49.
; b

Y %v  k b X b

kg m

3 1

(4.50)

La produccin total de biomasa viene dada por:


PB

kg d
1

; b V

(4.51)

donde:
V: Volumen efectivo del tanque de aereacin.
La ecuacin 4.50 puede reordenarse y obtener de ella otro parmetro importante, el rendimiento verdadero o rendimiento observado, Yob:

; b
%v
El trmino

Y  kb

1
%x

(4.52)

; b
%v de la ecuacin 4.52 tiene las mismas dimensiones que Y, kg

kg y es denominado rendimiento observado:

Yob

Y

kb
%x

149

(4.53)

Menndez Gutirrez, C. y J. Prez Olmo

Otra forma til de expresar el Yob , independientemente del valor de ' S


cuyo conocimiento lleva implcito la ecuacin 4.53, puede obtenerse de la ecuacin 4.50:

Yob

Y
1  kb x

(4.54)

Todo el desarrollo hasta aqu se ha basado en la suposicin de que toda la


biomasa es degradable, sin embargo, una discusin alrededor de la ecuacin
4.54 puede ser til para justificar la necesidad de introducir el criterio de que
toda la biomasa no es biodegradable.
De la ecuacin 4.54 puede inferirse la influencia que tiene la edad del lodo
sobre Yob. Mientras menor sea la edad del lodo, la Yob se aproximar ms al valor
de Y. Esa conclusin responde a la realidad ya que mientras ms pequea sea la
edad del lodo menos acentuada ser la autooxidacin o fase endgena. De hecho, tanto la ecuacin 4-54 como las precedentes pueden utilizarse sin mucho
error siempre que T x 5 das.
El alcance o validez de la ecuacin 4.54 es ms limitado cuando la edad del
lodo es superior a los diez das.
Si se supone un caso extremo, T x o f entonces Yob o 0
Esta conclusin no responde a la realidad. En la prctica diaria, por muy alta
que sea la edad del lodo, la Yob nunca es cero. Esto se debe a que una parte de
la biomasa orgnica producida es no degradable bajo condiciones aerbicas, y
por tanto, aparece reflejada en la Yob18, 19 Consecuentemente con esto, el balance de biomasa debe contemplar tanto la fraccin degradable como la no degradable
utilizando el desarrollo propuesto por Chudoba y Tucek.20
La biomasa total sintetizada, Xb, est constituida por tres fracciones:
a) La fraccin orgnica degradable (Xb, D).
b) La fraccin orgnica no degradable (Xb, N).
c) La fraccin mineral (Xb, M).
Xb

X b , D  X b , N  X b ,M

(4.55)

De estas tres fracciones, solo Xb,D es degradable de acuerdo con la expresin:

dX b
dt

 kd X b,D

donde:
kd: Constante de velocidad de descomposicin de la fraccin degradable de
la biomasa (kg kgd).

150

Lodo Activado

Si E es la fraccin no degradable aerbicamente de biomasa orgnica y Y0,


el coeficiente de produccin de biomasa total, entonces E Y0 es el coeficiente de
produccin de la parte de la biomasa orgnica que es no degradable y (1 - E ) Y0
es el coeficiente de produccin de biomasa orgnica degradable. Sea considerado adems f 0m como la fraccin mineral de la biomasa al formarse.
Por otra parte, aceptando que ' S es la cantidad de sustrato removido en el
sistema, entonces Y0 ' S es la parte orgnica de la biomasa producida y

>f

m / 1  f 0m Y0 S es la parte inorgnica de la biomasa producida.


Un balance de masa de la biomasa degradable da origen a una ecuacin
similar a la 4.47.
Tomando en consideracin que en estado estacionario la edad del lodo de la
biomasa total T x es la misma para la parte degradable y no degradable, se podr
escribir:

1  Y0 6

X b ,D

 k d X b ,D 

(4.56)

despejando Xb,D

1  Y0 6 x
1  kd x

X b ,D

(4.57)

Mediante un balance de masa de la parte orgnica no degradable se obtiene:


Y0 6 X b , N


(4.58)

despejando Xb,N:

X b ,N

Y0 6 x

(4.59)

La concentracin de biomasa orgnica (Xb,o), se obtiene de la suma de las


ecuaciones 4.57 y 4.58:
X

b,o

Y 0 6 x 1  kd x

1 k

d x

(4.60)

De acuerdo con lo anterior, la fraccin degradable de los constituyentes


orgnicos de la biomasa, fd, es igual a
fd

X b,D
X b,o

1

1  kd x
151

(4.61)

Menndez Gutirrez, C. y J. Prez Olmo

Al hacer el balance de la parte mineral se supone que la fraccin mineral de


las nuevas clulas es f 0m. Entonces:
f 0 m Y0 6 X b . M

x
1  f 0m

(4.62)

despejando Xb,M:
f 0 m Y0 6 x

1  f 0m

X b ,M

(4.63)

La concentracin total de biomasa se obtiene sumando las ecuaciones 4.60


y 4.63:
Xb

Y0 6 x

1  kd x
f 0m


0
1 k
1

f
m
d
x

(4.64)

Teniendo en cuenta que:


X b ,o
6
x

La ecuacin 4.60 puede escribirse:


Y0 ,ob

Y0 , ob

1  kd x
Y0
1  kd x

(4.65)

para
x o 0

Y0 ,ob o Y0
obtenindose un resultado que es lgico.
Por otra parte, para aquellos casos en los que:
x o f

Y0 , ob o Y0

Este ltimo resultado de que para altas edades del lodo el valor del coeficiente Y0,ob tiende a la fraccin no degradable de la biomasa, es coherente con la
realidad prctica. De aqu puede concluirse que para edades del lodo pequeas
y moderadas, pueden utilizarse las ecuaciones 4.48 hasta la 4.54. Para altas
edades del lodo deben tenerse en cuenta las consideraciones que le siguen hasta
la ecuacin 4.65.
152

Lodo Activado

4.7.2. Influencia de los slidos suspendidos de la alimentacin


en la concentracin del lodo activado
Los slidos suspendidos primarios, X0, al igual que la biomasa, estn constituidos por tres fracciones:
a) Orgnica degradable (XO, D )
b) Orgnica no degradable (XO, N )
c) Mineral (XM )

X0

X O,D  X O,N  X M

(4.66)

Si X0 es la concentracin total de los slidos suspendidos primarios en el


agua residual, fo la fraccin orgnica y a la fraccin no degradable, se pueden
establecer las siguientes relaciones:
XO, D = (1 - D ) fo X0

(4.67)

XO, N = D fo X0

(4.68)

XM  fo) X0

(4.69)

Puede suponerse que la fraccin orgnica degradable est incluida en la


DBO5 y que de acuerdo con el producto Y 0 ' S es convertida en nueva
biomasa.
La suposicin de que Y0 tiene el mismo valor para el material orgnico, sea
soluble o insoluble, es aceptable desde el punto de vista prctico.
La concentracin en estado estacionario en el tanque de aereacin de las
dos fracciones que no experimentan degradacin, X1,N y X1,M, puede obtenerse
por un balance de masa.
El balance de masa de la fraccin no degradable da origen a una ecuacin
similar a la 4,58.
f o X 0 X 1,N


(4.70)

despejando:

X 1,N

f o X 0 x

(4.71)

Del balance de la fraccin mineral:

1  f o

X 1,M
x

153

(4.72)

Menndez Gutirrez, C. y J. Prez Olmo

X 1,M

1  fo X 0 x

(4.73)

La concentracin total del lodo primario se obtiene sumando las ecuaciones


4.71 y 4.73:
fo X 0 x

X1

1  fo


f o

(4.74)

La concentracin total del lodo activado en el tanque de aeracin en


condiciones de estado estacionario viene dado por la suma de la concentracin total de biomasa (ecuacin 4.64) y la de los slidos suspendidos primarios (ecuacin 4.74):
Xt

X b  X1

Y0 6 x 1  kd
f 0m


1  k

1  f 0m
d x

(4.75)

f o X 0 x
1  fo


fo

Tabla 4.3. Valores de constantes para biomasa y slidos suspendidos primarios para aguas residuales domsticas17

Asociados a
los SSP

Asociados a
la biomasa

Constante

Intervalo

D
fo
E
f0m
Y0
kd

0,3 0,4
0,70 0,75
0,18 0,24

Valor
recomendado
0,3
0,73
0,22
0,05
0,65
0,15 (20 oC)

Ejemplo 4.1
Se desea mantener un sistema del lodo activado operando a una edad del
lodo de tres das para las siguientes condiciones de trabajo y de acuerdo con el
esquema de la figura 4.19.
Q0 = 15000 m3 d

154

Lodo Activado

Volumen efectivo del tanque de aereacin = 3750 m3


S0 = 500 mgL (total)
Xw = 5,662 kg m S2 = 50 mgL (soluble)

Y = 0,5
kb= 0,03 d
Determine:
a) Cantidad del lodo que debe ser extrado.
b) Flujo volumtrico de lodos a extraer.
c) Flujo de recirculacin.
a) Yob

0 ,5
1  0 ,03 3

0,46

PB = Yob Q0 (S0S2).
PB = 0,46 (15 000) (0,45).
PB = 3 105 kg d.
b) X b

3 0,5 0 ,45
2 ,477 kg m 3


0,25 >1  0,03 3 @

Suponiendo X2 | 0:

3 750 2,477
Qw 5,662

Qw = 546,8 m3 d.
d) Suponiendo: X2 | 0
(Q0 + Qr) X = Xw Qw + Xw Qr
Qr

37 155  3 095,98
10 693,57 m 3 d 1
3,185

Ejemplo 4.2
En una planta que emplea lodo activado est siendo tratada un agua residual
con las siguientes caractersticas:
Slidos suspendidos totales = 0,275 kg m
Flujo de agua residual = 3 785 m3d
Volumen efectivo del tanque de aereacin = 7 570 m3
DBO5 de los slidos totales = 0,3 kg m
DBO5 de los slidos disueltos = 0,15 kg m
Toda la DBO5 es removible, por tanto ' S = 0,3

155

Menndez Gutirrez, C. y J. Prez Olmo

Determine, en estado estacionario:


a)
b)
c)
d)

Concentracin de biomasa.
Concentracin de slidos primarios.
Concentracin total del lodo activado en el tanque de aeracin.
DBO5 en el efluente de la planta si el sedimentador secundario tiene una
eficiencia de remocin de slidos suspendidos de 90 %.

Suponga las siguientes condiciones:

T x = 10 d
t = 26 oC
k

d 20

fo = 0,73 (para los slidos primarios)


D = 0,3

0 ,15 d 1

Y0 = 0,65

E= 0,22
fom = 0,05
a) kd 26 o C

0,15 1,047

26  20

0,197 d 1

0,78 0,65 0,3 10


2>1  0,197 10 @

0,256 kg m 3

X b, N

0,22 0 ,65 0 ,3 10
2

0,21 kg m 3

X b, M

0,05 0,65 0,3 10

0,05 kg m3
0,95
2

XbD

Xb = 0,256 + 0,210 + 0,050 = 0,516 kg m


b) X 1

0,73 0,275 10
0,27
0 ,3 
0,672 kg m 3
0,73
2

c) X = 0,516 + 0,672 = 1,188 kg m


d) X2 = 0,1 (0,256) = 0,0256 kg m
Para el lodo, considerando que X = 10 d: fo = 0,67 (de la figura 4.18)
DBO5 T

0 ,35 0,0256 103

156

9 mg L1

Lodo Activado

4.8. DETERMINACIN DE CONSTANTES A ESCALA


DE LABORATORIO
Una unidad del lodo activado a escala de banco es la que se utiliza comnmente para la determinacin en el laboratorio de las constantes Y y kb
entre otras. Las condiciones a las que se opera la unidad son las mismas
supuestas en las secciones precedentes para la obtencin de las ecuaciones
a partir de los balances de masa.
Elementos esenciales para la obtencin de buenos resultados son el mantener un flujo continuo de residual al reactor y un sistema completamente
mezclado en l.
Para lograr suficientes datos, la unidad debe operarse a varias edades
de lodo, generalmente en el intervalo de 3 a 20 das. La manera ms sencilla
de mantener constante la edad del lodo es la extraccin diaria de una cantidad constante de licor mezclado del propio reactor o tanque de aereacin.
Las muestras a cada edad del lodo no deben considerarse como representativas del proceso hasta que no haya transcurrido un tiempo que sea igual o
mayor a cinco veces la edad de lodo. Solo as podr considerarse que el
sistema est trabajando en estado estacionario. 21
La temperatura, pH y el oxgeno disuelto son parmetros que deben permanecer constantes o al menos ser registrados mientras duren las pruebas.
La informacin requerida para el clculo de los parmetros de operacin es:
Q: Flujo de entrada (L d ).
Qw: Flujo de extraccin del lodo (L d ).
V: Volumen del tanque de aereacin (L).
Xb : Concentracin de SSVLM (mg L ).
Xe: Concentracin de SSV en el efluente (mg L ).
S0 : DBO soluble DQO en el influente (mg L  ).
Se: DBO soluble DQO en el efluente (mg L ).
A partir de esta informacin se calculan, para cada edad del lodo el
tiempo de retencin (ecuacin 4.1) y la razn de la carga del lodo (ecuacin
4.12). Si el exceso del lodo se extrae directamente del tanque de aeracin,
el volumen diario a extraer viene dado por:

Qw

V
x

157

Menndez Gutirrez, C. y J. Prez Olmo

Los pares de valores de

1
x y %x que se obtienen para cada edad del lodo

se grafican segn la figura 4.22. La pendiente de la recta que se obtiene da el


valor del rendimiento Y y el intercepto con el eje de ordenadas el valor de la
constante de autooxidacin kb.

Fig. 4.21. Esquema de un arreglo de equipamiento para calcular las


constantes a escala de laboratorio.
Ejemplo 4.3
Se desea determinar los parmetros Y y k b de un agua residual de origen industrial utilizando un tanque de aereacin con un volumen efectivo
de 10 L. El agua residual sedimentada se aliment a un flujo de 20 L d 
para obtener un tiempo de retencin de 12 horas durante todo el experimento.
La extraccin del lodo se hizo directamente del tanque de aereacin
una vez al da, excepto para la primera de las corridas, que por ser
relativamente elevado, el volumen de extraccin, se hizo en dos partes
(tabla 4.4).

158

Lodo Activado

Tabla 4.4. Datos experimentales

S0
Se
-1
(mgL ) (mgL-1)
750
75
800
96
775
116
825
148
850
170

Qw
(L d-1)
2,4
3,0
3,9
4,8
5,4

X
(mgL-1)
2700
2347
1757
1504
1360

Xe
(mgL-1)
10
5
2,5
2,5
2,7

Como la concentracin de SSV en el efluente es baja, la edad del lodo puede


calcularse directamente a partir de la extraccin Qw.
A partir de los datos anteriores puede obtenerse la tabla 4.5.
Tabla 4.5. Datos obtenidos

Al graficar

Tx

1
Tx

' Bx

(d)
4,16
3,33
2,56
2,08
1,85

(d-1)
0,24
0,30
0,39
0,48
0,54

(kg kg-1 d-1)


0,50
0,60
0,75
0,90
1,00

1
x contra ' Bx se obtiene la recta de la figura 4.22.

Fig. 4.22. Ejemplo 4.3 (1/T X vs ' Bx).

159

Menndez Gutirrez, C. y J. Prez Olmo

Rendimiento, Y = 0,53
Constante de auto-oxidacin, kb = 0,007 d
Ejemplo 4.4
A partir de la informacin suministrada en el ejemplo anterior, determine la
constante de velocidad de reaccin:

' Bx
-1 -1
kgkg d
S
S0

1,00

0,90

0,75

0,60

0,50

0,20

0,18

0,15

0,12

0,10

S
Graficando ' Bx contra S de acuerdo con una modificacin de la ecuacin
0
4.28, se obtiene la constante k de velocidad de reaccin de la pendiente de la
recta (figura 4.23).
k = 5d

Fig. 4.23. Ejemplo 4.4 (' Bx vs S/S0).

4.9. REQUERIMIENTOS NUTRICIONALES


La formacin de biomasa y las buenas propiedades de sedimentacin de
un lodo pueden verse afectadas si el nitrgeno y fsforo no estn presentes
en cantidades suficientes. Otros muchos elementos tambin son necesarios
para el metabolismo, pero en concentraciones tales que raras veces estn
en dficit.
De acuerdo con el modelo de Lawrence y McCarty,22 el contenido de
nitrgeno y fsforo de la biomasa producida es de 12,3 % y 2,6 % respecti160

Lodo Activado

vamente. Por otra parte, a medida que disminuye la carga orgnica o aumenta la edad del lodo, existir una acumulacin de residuos celulares no
biodegradables y cuyo contenido de nitrgeno y fsforo es de 7 % y 1 %.
Por tanto el nitrgeno y el fsforo necesarios dependen de la edad del lodo.
Bajo estas condiciones, y teniendo en cuenta adems que el requerimiento
de estos nutrientes puede ser estimado a partir de la cantidad de estos dos
elementos que estn contenidos en el lodo que se extrae del proceso, se
tiene que la necesidad de ambos elementos ser:

0,78  f d
fd
PB  0,07
PB
0,78
0,78

(4.76)

0,78  f d
fd
PB  0,01
PB
0,78
0,78

(4.77)

0,123

0,026

N kg d 1

P kg d 1

donde:
PB: Produccin de biomasa (ecuacin 4.51)

fd

1 

, 0,78 (1 - )

1  k d X

X b,D

fd: fraccin degradable de los SSVLM f d


X b ,o

En las ecuaciones 4.76 y 4.77 se ha considerado que 77 % de la fraccin


degradable es biolgicamente activa.19

Notas bibliogrficas
1 PAVN, J. L., M. W. TENNEY AND W. F. ECHELBORGER:%DFWHULDOH[RFHOOXODU
SRO\PHUVDQGELRORJLFDOIORFXODWLRQ-RXUWater Poll. Control Fed., vol.
44, no. 3, 1972.
2 BISOGNI, J. J. AND W. LAWRENCE:5HODWLRQVKLSEHWZHHQELRORJLFDOVROLGVUHWHQ
WLRQWLPHDQGVHWWOLQJFKDUDFWHULVWLFVRIDFWLYDWHGVOXGJHSWater Res.,
vol. 5, no. 9, 1971.
3 CHAO, A. C. AND T. M. KEINATH:,QIOXHQFHRISURFHVVORDGLQJLQWHQVLW\RQ
VOXGJHFODULILFDWLRQVDQGWKLFNHQLQJFKDUDFWHULVWLFVWater Res., vol. 13,
no. 2, 1979.
4 LOVETT, D. A., B. V. KAVANAGH AND L. S. HERBERT:(IIHFWRIVOXGJHDJHRQ
substrate composition on the settling and dewatering characteristics of
DFWLYDWHGVOXGJHWater Res., vol. 17, no. 4, 1983.
161

Menndez Gutirrez, C. y J. Prez Olmo

5 CHUDOBA, J., V. OTTOVA AND V. MADERA:&RQWURORIDFWLYDWHGVOXGJHILOD


mentous bulking I. Effect of the hydraulic regimen of degree of mixing
LQDQDHUDWLRQWDQNWater Res., vol. 7, no. 6, 1973.
6 CHUDOBA, J., P. GRAU AND V. OTTOV:&RQWURORIDFWLYDWHGVOXGJHILODPHQ
WRXVEXONLQJ,,6HOHFWLRQRIPLFURRUJDQLVPVE\PHDQVRIDVHOHFWRU
Water Res., vol. 7, no. 9, 1973.
7 CHIESA, S. C. AND S. R. LINNE:'LVFXVVLRQRIWKHHIIHFWVRIVOXGJHDJHDQG
selector configuration on the control of filamentous bulking in the actiYDWHG VOXGJH SURFHVVV 5HVHDUFK - Water Poll. Control Fed., vol.
62, no. 6, 1992.
8 KAPPELER, J. AND W. GUJER:9HULILFDWLRQDQGDSSOLFDWLRQRIDPDWKHPDWLFDO
PRGHOIRUDHURELFEXONLQJWater Res., vol. 28, no. 2, 1994.
9 GREENBERG, A. E., G. KLEIN AND W. KAUTMAN:(IIHFWRISKRVSKRURXVRQ
WKHDFWLYDWHGVOXGJH SURFHVVSewage and Industrial Wastes, vol. 27,
no. 3, 1955.
10 CHUDOBA, J., C. MENNDEZ Y J. PREZ: Fundamentos Tericos de Algunos Procesos para la Purificacin de Aguas Residuales, Ed. ISPJAE, Ciudad de La Habana, 1986.
11 T UCEK, F. AND J. CHUDOBA:0RGHOLQJRIZDVWHZDWHUWUHDWPHQWSURFHVV
Water Research, vol. 3, no. 2, 1969.
12 WEDDLE, C. L. AND D. J ENKINS:7KHYLDELOLW\DQGDFWLYLW\RIDFWLYDWHG
VOXGJHWater Res., vol. 5, no. 8, 1971.
13 WUHRMANN, K.: Biological Treatment of Sewage and Industrial Wastes, vol. I, Ed. Mac Cabe, 1956.
14 ECKENFELDER, W. AND'2 CONNOR: Biological Waste Treatment, Pergamon Press, Oxford, 1961.
15 WUHRMANN, K.:+LJK5DWH$FWLYDWHG6OXGJH7UHDWPHQWDQG,WV5HODWLRQ
WR6WUHDP6DQLWDWLRQ-RXUWater Poll. Control Fed., vol. 26, no. 1,
1954.
16 MENNDEZ, C.: Influencia de la edad del lodo en la composicin de los
slidos biolgicos, indito. 1998.
17 HEUKELEKIAN, H., H. E. OXFORD AND R. MANGANELLI:)DFWRUVDIIHFWLQJ
WKHTXDQWLW\RIVOXGJHSURGXFWLRQLQWKHDFWLYDWHGVOXGJHSURFHVVSewage Industrial Wastes, vol. 23, no. 8, 1951.
18 SIMONS, J. M. AND E. MC KINNEY:7KHELRFKHPLVWU\RIQLWURJHQLQWKH
V\QWKHVLVRIDFWLYDWHGVOXGJHSewage and Industrial Wastes, vol. 30,
no. 7, 1958.
19 WASHINGTON, D. R. AND L. J. HETLING: 9RODWLOHVOXGJHDFFXPXODWLRQLQDFWL
YDWHGVOXGJHSODQW-RXUWater Poll. Control Fed., vol. 37, no. 4, 1965.
162

Lodo Activado

20 CHUDOBA, J. AND F. T UCEK:3URGXFWLRQGHJUDGDWLRQDQGFRPSRVLWLRQRI


DFWLYDWHGVOXGJHLQDHUDWLRQV\VWHPVZLWKRXWSULPDU\VHGLPHQWDWLRQ
Jour. Water Poll. Control Fed., vol. 57, no. 3, 1985.
21 VANDEVENNE , L. AND W. ECKENFELDER:$FRPSDULVRQRIPRGHOVIRUFRP
SOHWHO\PL[HGDFWLYDWHGVOXGJHWUHDWPHQWGHVLJQDQGRSHUDWLRQWater
Res., vol. 14, no. 7, 1980.
22 KOUNTZ, R.R. Y C. F ORNEY:0HWDEROLFHQHUJ\EDODQFHLQDWRWDOoxidation
DFWLYDWHG VOXGJH V\VWHP Sewage and Industrial Wastes, vol. 31,
no. 7, 1959.

163

Transferencia de Oxgeno

CAPTULO 5

TRANSFERENCIA DE OXGENO

5.1. INTRODUCCIN
La transferencia de oxgeno es un elemento clave en los procesos biolgicos aerobios.
Exceptuando casos particulares, la oxigenacin de un medio se realiza
mediante la aereacin aprovechando el hecho de que el aire contiene aproximadamente 21 % en volumen de oxgeno.
La transferencia se puede realizar a travs de la introduccin de aire u
oxgeno en el agua o exponiendo el lquido a la atmsfera en forma de pequeas gotas.
Cualquiera que sea el procedimiento seleccionado para producir la
aereacin, existe una serie de elementos bsicos del proceso en s que son
comunes en todos ellos.

5.2. CONSIDERACIONES FUNDAMENTALES


DE LA TRANSFERENCIA DE OXGENO
En cualquier proceso de transferencia, la velocidad de esta se puede
expresar como el producto de una fuerza impulsora y un coeficiente de transferencia. En la aereacin, la fuerza impulsora es el gradiente de la concentracin de oxgeno.
Lewis y Whitman, 1 desarrollaron el concepto de la doble pelcula que
considera pelculas estacionarias en las interfases del gas y del lquido
respectivamente, a travs de las cuales ocurre la transferencia de masa.
Figura 5.1.
De acuerdo con el concepto de la doble pelcula, la masa de oxgeno transferida a un medio lquido puede expresarse segn,
M = KL A (Cs - C) = Kg A (Pg - P)
(5.1)

165

Menndez Gutirrez, C. y J. Prez Olmo

donde:
M: masa de oxgeno transferida en la unidad de tiempo (kg h).
A: rea seccional a travs de la cual ocurre la transferencia.
KL, Kg: coeficientes de transferencia en la pelcula de lquido y gas respectivamente (L m h ).
Cs: concentracin de saturacin de oxgeno en la fase lquida.
C: concentracin de oxgeno en el lquido.
Pg, P: presin parcial de oxgeno en el aire.

Fig. 5.1. Doble pelcula segn Lewis y Whitman.


Para gases poco solubles, como el oxgeno, la pelcula controladora de la
transferencia es la de lquido. Para gases relativamente solubles, controla la
pelcula de gas.
Para el primero de los casos, al aumentar la turbulencia disminuye el
espesor de la pelcula de lquido y aumenta el coeficiente de transferencia
del gas en el lquido, KL.
Hay otros modelos tericos ms recientes que el de Lewis y Whitman,
tales como el de la velocidad fraccional de renovacin de superficie
(Danckwerts). 2 Sin embargo el ms aceptado sigue siendo el de Lewis y
Whitman.
Para aquellos procesos controlados por la pelcula de lquido, la ecuacin
5.1 puede transformarse:

1
M
V
donde: k L a K L

dC
k L a Cs  C
dt

A
, coeficiente global de transferencia de oxgeno, h.
V
166

(5.2)

Transferencia de Oxgeno

Cuando los trminos vinculados a la transferencia de oxgeno se relacionan


tal como aparece en la ecuacin 5.2, vienen referidos, por problemas de comodidad, para agua de acueducto (agua limpia), 20 0C y una presin de 101,3 kPa.

5.2.1. Concentracin de oxgeno en condiciones de saturacin


La concentracin de saturacin de oxgeno en el agua limpia depende de la
temperatura, concentracin de slidos totales disueltos y de la presin.
En la tabla 5.1 se reporta la solubilidad del oxgeno en agua a la presin
normal y diferentes concentraciones de cloruro.
Para presiones de trabajo diferentes a la normal, y considerando la profundidad del tanque de aereacin, la concentracin de saturacin se calcula:

Cs,m

Cs

Pa  0,5Ph
Po

(5.3)

Cs: concentracin de saturacin a la temperatura de trabajo y presin normal mgL.


Cs,m: concentracin de saturacin promedio a la temperatura de trabajo
y a una profundidad que es igual a la mitad de la altura del tanque de
aeracin mgL .
P0: presin normal, 101,3 kPa.
Pa: presin atmosfrica en las condiciones de trabajo kPa.
Ph: presin hidrosttica a la altura de la descarga del aire kPa.
Tabla 5.1. Solubilidad de oxgeno en agua a 101,3 kPa
Temperatura
C
16
17
18
19
20
21
22
23
24
25
26
27
28
29
30

Concentracin de cloruro en agua mgL-1


0
10,0
9,7
9,5
9,4
9,2
9,0
8,8
8,7
8,5
8,4
8,2
8,1
7,9
7,8
7,6

5
9,5
9,3
9,1
8,9
8,7
8,6
8,4
8,3
8,1
8,0
7,8
7,7
7,5
7,4
7,3

167

10
9,0
8,8
8,6
8,5
8,3
8,1
8,0
7,9
7,7
7,6
7,4
7,3
7,1
7,0
6,9

15
8,5
8,3
8,2
8,0
7,9
7,7
7,6
7,4
7,3
7,2
7,0
6,9
6,8
6,6
6,5

20
8,0
7,8
7,7
7,6
7,4
7,3
7,1
7,0
6,9
6,7
6,6
6,5
6,4
6,3
6,1

Menndez Gutirrez, C. y J. Prez Olmo

Para tener en consideracin adems los constituyentes del agua residual se


define el coeficiente E .

solubilida d en el agua residual


solubilida d en el agua limpia

Cs,w
Cs

Los valores de E suelen variar entre 0,7 y 0,8 como consecuencia de las
variaciones de la solubilidad del oxgeno debido a la presencia de sales, slidos
suspendidos y sustancias tensoactivas.3

5.2.2. Coeficiente global de transferencia de oxgeno


El coeficiente de transferencia de oxgeno depende del implemento de
aereacin y de las caractersticas fsicas y qumicas del sistema que se desea
oxigenar:4
1. Temperatura. Al aumentar la temperatura aumenta el valor del coeficiente
de transferencia,
k L a t k L a 20 o C t  20

(5.4)

El valor de T es funcin de las condiciones de trabajo. Se suele situar en el


intervalo de 1,015 y 1,040. El valor tpico para todo tipo de aereador es 1,024.
2. Turbulencia. Un incremento de la turbulencia da como resultado un aumento
en el coeficiente de transferencia.
3. Altura del lquido. El efecto que tiene la altura de la columna del lquido bajo
aereacin sobre el coeficiente global de transferencia de oxgeno depende
en cierta medida del mtodo empleado en la aereacin. Para muchos sistemas de aereacin por difusin kLa vara con la altura de acuerdo con la
relacin:
k L a1
k L a2

H1

H2

(5.5)

donde el exponente n tiene un valor cercano a 0,7 para la mayora de los


sistemas.
4. Caractersticas del medio. El valor del coeficiente de transferencia de oxgeno del agua pura puede diferir considerablemente del de un medio que contenga compuestos orgnicos u otras sustancias en disolucin.
Con el objetivo de comparar la velocidad de transferencia de oxgeno en el
agua con la de otro medio cualquiera para un implemento de aereacin particular se define el coeficiente D.
168

Transferencia de Oxgeno

k L a del medio
k L a del agua limpia

El valor de D puede no ser constante y variar durante el proceso que tiene


lugar en el bio-reactor. En la tabla 5.2 se reportan valores tpicos de D para
diferentes implementos de aereacin.
La ecuacin 5.2 queda entonces:

dC
k L a CsC 1,02t  20
dt

(5.6)

y si se toma en consideracin la profundidad del tanque:

dC
dt

k L a Cs,mC 1,02 t  20

(5.7)

Tabla 5.2. Valores tpicos de D

Implemento de aereacin
Difusor de burbuja gruesa
Difusor de burbuja fina
Eyector de aereacin
Aereador superficial
Turbinas

Valor tipo
0,85
0,50
0,75
0,90
0,85

5.2.3. Clculo del coeficiente global de transferencia


de oxgeno
Para poder aplicar la ecuacin (5.7) en cualquier sistema aerobio, se requiere conocer o determinar previamente los coeficientes kLa, D, y E.
En agua limpia la velocidad de transferencia de oxgeno puede plantearse
segn la ecuacin:

dC
dt

k L a CsC mgL h

(5.8)

Como se aprecia de la ecuacin anterior, la velocidad de la transferencia es


proporcional al dficit de oxgeno y al coeficiente global de transferencia de
oxgeno, kLa, tal como ya fue expresado.

169

Menndez Gutirrez, C. y J. Prez Olmo

El valor del coeficiente kLa depende, a su vez, de la turbulencia en la interfase


aire - lquido. Por tanto, es funcin del sistema de aereacin, la geometra del
tanque de aereacin, las caractersticas del lquido y de la temperatura.
El mtodo que se describe a continuacin para determinar el coeficiente de
transferencia de oxgeno se basa en estimar el tiempo necesario para que la
diferencia entre el oxgeno de saturacin, Cs y el oxgeno disuelto en un instante
dado, C, disminuya 90 %.
Integrando la ecuacin 5.8 entre t1 y t2 se obtiene:

2,3 log

CsC1
CsC2

kL a t2 t1

(5.9)

Suponiendo que (Cs - C2) = 0,1 (Cs - C1), la ecuacin 5.9 queda:

kLa

2,3
t90

(5.10)

t90: tiempo durante el cual la diferencia entre la concentracin de oxgeno


en condiciones de saturacin y la del oxgeno disuelto disminuye 90 %.5
Figura 5.2.

5.2.4. Procedimiento
Clculo de kLa
El procedimiento general comprende la eliminacin del oxgeno disuelto
en un volumen conocido de agua, mediante la adicin de sulfito de sodio, y la
posterior reoxigenacin hasta valores cercanos a la concentracin de saturacin.5
1. El tanque de aereacin se llena con agua limpia del acueducto.
2. Debe ser reducida la concentracin de oxgeno disuelto del tanque hasta un
valor cercano a cero. Para esto puede utilizarse una solucin de sulfito de
sodio a razn de 7,88 mg por cada mg de oxgeno disuelto del agua. Cuando
se desea acelerar la etapa de de-oxigenacin puede utilizarse una solucin
de Co2+ como catalizador a razn de 5 mgL.
3. Una vez que la concentracin de oxgeno disuelto alcanza un valor cercano a
0,5 mgL se pone en funcionamiento el implemento de aereacin.
4. A partir del momento en que la concentracin de oxgeno sea igual a
1 mgL se hacen mediciones de la concentracin a intervalos regulares
de tiempo.
5. Se tabulan los resultados y se procede segn se ilustra en el ejemplo 5.1.
170

Transferencia de Oxgeno

Fig. 5.2. Clculo del coeficiente de transferencia de oxgeno.

5.2.5. Capacidad de oxigenacin


Con el objetivo de poder comparar las posibilidades de los distintos implementos para suministrar oxgeno, es necesario introducir el trmino capacidad
de oxigenacin (CO).
La capacidad de oxigenacin de un implemento de aereacin se define como
la cantidad de oxgeno que puede ser suministrado por ese implemento, en la
unidad de volumen y de tiempo, cuando la concentracin de oxgeno disuelto es
cero. De acuerdo con la ecuacin 5.8, cuando C = 0:
CO = 0,024 kLa Cs kg md
(5.11)
donde:
kLa: coeficiente de transferencia de oxgeno, h.
Cs: concentracin de saturacin de oxgeno disuelto a la temperatura de
trabajo, mgL.
Por tanto, para determinar la capacidad de oxigenacin solo se requiere
conocer el coeficiente de transferencia de oxgeno, y la concentracin de saturacin a la temperatura y presin a la que se est trabajando.
Ejemplo 5.1
Se realizan pruebas a escala de laboratorio con un aereador superficial de
paleta a tres velocidades diferentes. A partir de los datos obtenidos, calcule el
coeficiente de transferencia de oxgeno a cada velocidad (tabla 5.3).
171

Menndez Gutirrez, C. y J. Prez Olmo

t = 25 oC

Cs = 8 mgL

Solucin:
Se grafica en papel semilogartmico (Cs - C) en funcin del tiempo t, segn
la figura 5.3.
Tabla 5.3. Concentracin de oxgeno en funcin del tiempo de aereacin para
3 velocidades diferentes de agitacin

Tiempo
(min)
0
0,5
1,0
1,5
2,0
2,5
3,0
3,5
4,0
4,5
5,0
6,0
7,0
8,0
9,0
10,0
11,0
13,0
15,0
20,0
25,0

Diferentes velocidades de rotacin del aereador


I
II
II
III
III
I
Cs-C
O2
Cs-C
O2
Cs-C
O2
mgL-1
mgL-1
mgL-1
1.00
7.00
1,00
7,00
1,00
7,00
2,00
6,00
1,55
6,45
1,40
6,60
2,90
5,10
2,02
5,98
1,75
6,25
3,70
4,30
2,55
5,45
2,05
5,95
4,35
3,65
3,00
5,00
2,40
5,60
4,85
3,15
3,40
4,60
2,70
5,30
5,35
2,65
3,80
4,20
3,00
5,00
5,80
2,20
4,20
3,80
3,25
4,75
6,15
1,85
4,50
3,50
3,50
4,50
6,40
1,60
4,80
3,20
3,75
4,25
6,70
1,30
5,05
2,95
3,95
4,05
7,15
0,85
5,55
2,45
4,40
3,60
7,40
0,60
5,95
2,05
4,75
3,25
7,60
0,40
6,25
1,75
5,10
2,90
7,70
0,30
6,55
1,45
5,40
2,60
7,80
0,20
6,80
1,20
5,65
2,35
7,90
0,10
7,00
1,00
5,90
2,10
7,95
0,05
7,30
0,70
7,50
0,50
6,60
1,40
7,80
0,20
7,20
0,80
7,40
0,60

El tiempo t90 se corresponde con el tiempo necesario para que la diferencia


(Cs - C) disminuya un ciclo logartmico, o sea:
Velocidad I t90: 6,8 min
Velocidad II t90: 13,0 min
Velocidad III t90: 21,2 min
172

Transferencia de Oxgeno

Por tanto, considerando que kL a

2,3
t90 , de la figura 5.3 se tiene:

Velocidad I kLa = 20,35 h


Velocidad II kLa = 10,60 h
Velocidad III kLa = 6,52 h

Fig. 5.3. Ejemplo 5.1 Clculo de kLa.


Ejemplo 5.2
Determine la capacidad de oxigenacin del implemento utilizado para el ejemplo 5.1 cuando se trabaja a la velocidad II.
Solucin:
CO = 0,024 kLa Cs = 0,024 x 10,60 x 8 = 2,03 kg m d

5.3. REQUERIMIENTO DE OXGENO


La determinacin de la necesidad de oxgeno en los procesos aerobios de
biomasa en suspensin constituye una etapa importante en las actividades de
diseo y evaluacin de una instalacin de tratamiento. En la medida que se
conozca la cantidad de oxgeno que es necesario para un proceso dado, se estar en mejores condiciones de seleccionar el equipamiento adecuado para suministrarlo.

173

Menndez Gutirrez, C. y J. Prez Olmo

La velocidad de consumo de oxgeno en los sistemas de biomasa en suspensin puede representarse por la siguiente ecuacin emprica:
rX

Y c % X  b

(5.12)

r
X

(5.13)

6
X

(5.14)

rX

donde:

%X

r: velocidad de consumo de oxgeno mgLd.


X: concentracin de slidos suspendidos voltiles (SSV) en el tanque de
aereacin mgL .
Y c: kg O2 consumido (kg DBO removido).
b: kg O2 consumido (kg SSV autooxidado.d).
' Bx: remocin especfica de DBO kg kgd.
' S = So - S mg L.
T = tiempo de retencin hidrulico d.
La ecuacin 5.12 puede expresarse tambin tomando como base la DQO.
Valores tpicos de Y c y b se reportan en la tabla 5.4.
Tabla 5.4. Valores tpicos de Y y b

Base
DBO5
DQO
DQO
DBO5
DQO
DQO

b
Yc
-1
(d-1)
(kg kg )
0,50
0,10
0,40
0,13
0,27
0,11
0,55 - 0,80 0,03 - 0,17
0,33 - 0,54 0,3 - 0,17
0,35
0,08

Tipo de
residual
Municipal
Municipal
Pulpa y papel
Municipal
Municipal
Tenera

Fuente
1
1
4
7
7
3

Los coeficientes Y c y b pueden ser calculados a escala de laboratorio. (Epgrafe 5.3.1).


El valor de rx depende en gran medida de la actividad del sistema. Por
ejemplo, para un proceso convencional de lodo activado, Eckenfelder 6 ha
reportado valores de 0,84 kg kg d a la entrada y de 0,2 kg kg d a la
salida del proceso. Para residuales teneros en Cuba tratados mediante lagunas
aereadas,7 se han obtenido valores de 1,43 kg kgd y 0,79 kg kgd para uno
174

Transferencia de Oxgeno

y tres das de tiempo de retencin, respectivamente. Desde Hoover y Porges 8


se acepta como intervalo para el valor de la velocidad especfica de consumo de oxgeno para la fase de respiracin endgena el comprendido entre
0,15 y 0,50 kg kgd .
En estado estacionario las condiciones de trabajo son tales que el suministro
de oxgeno debe ser igual a su requerimiento:
r = D kLa (E Cs - C)
(5.15)
El metabolismo aerobio es independiente de la concentracin de oxgeno
disuelto cuando esta es superior a un valor considerado como crtico, aproximadamente entre 0,2 y 2,0 mgL . Por debajo de este valor crtico la velocidad de las reacciones metablicas aerobias se ven limitadas. 9,10,11
El requerimiento mnimo de aire aplicado por kilogramo de DBO alimentado al tanque de aereacin es funcin de las caractersticas del sistema en
particular. Para sistemas de carga media debe ser aproximadamente de
90 kg de aire por kilogramo de DBO mientras que para aereacin extendida
puede llegar a alcanzar 120 kg por kilogramo de DBO. Estas demandas
estn calculadas sobre la base de suponer que el implemento de aereacin
es capaz de transferir al menos 1 kg de oxgeno por cada kilogramo de DBO
aplicado. En todos los casos es recomendable garantizar una concentracin
mnima de oxgeno disuelto de 2 mgL .

5.3.1. Clculo de los coeficientes Y' y b


Como ya se afirm anteriormente, los coeficientes Y c y b pueden ser calculados a partir de pruebas realizadas a escala de laboratorio. De la ecuacin 5.12,
cuando se grafica la velocidad especfica de respiracin, rx, en funcin de la
remocin especfica, ' Bx, se obtiene una recta cuya pendiente es Y c, y el intercepto con el eje de ordenadas, b. Figura 5.5.
Procedimiento
Se realizan corridas experimentales a diferentes edades de lodo, 3 como
mnimo. Para cada corrida se registran los valores promedios de los siguientes
parmetros: So, S, X, T .
Para cada corrida:
1. Se toma una muestra representativa del licor mezclado del reactor y se introduce en un recipiente en el que se pueda medir la disminucin en el tiempo de
la concentracin de oxgeno disuelto.
175

Menndez Gutirrez, C. y J. Prez Olmo

Fig. 5.4. Clculo de la velocidad de respiracin.


2. La muestra se aerea hasta saturacin en oxgeno.
3. Se detiene la aereacin y se mide la variacin de la concentracin de oxgeno
en el tiempo.
4. Se grafica la concentracin de oxgeno disuelto vs tiempo. Figura 5.4. La
pendiente obtenida en la seccin inicial de la curva es la velocidad de respiracin r.
5. Para cada corrida se divide el valor de la velocidad de consumo de oxgeno
determinado (r) por la concentracin de SSV.
6. Se prepara una tabla de rx vs ' Bx con tantos puntos experimentales como
edades de lodo se hayan prefijado.
7. Se grafica rx vs ' Bx. Figura 5.5.

Fig. 5.5. Clculo de Y y b.


176

Transferencia de Oxgeno

Ejemplo 5.3
Los datos que se dan a continuacin fueron obtenidos a escala de laboratorio trabajando a cuatro edades de lodo diferentes y midiendo en cada caso la
velocidad de respiracin r de la biomasa obtenida en el licor mezclado. Determine los coeficientes Y c y b. (Tabla 5.5)
Tabla 5.5. Datos experimentales

Tx
(d)
3
5
7
10

X
(mg L-1)
800
1 200
1 800
2 500

r
(mg L-1 h -1)
26,6
35,0
30,0
20,8

' Bx
(kg kg -1 d -1)
1,25
1,00
0,50
0,25

Solucin:
A partir de los datos de velocidad de respiracin r se calcula la velocidad de
respiracin especfica rx.
Tabla 5.5.a. Datos de velocidad de respiracin especfica
-1 -1

rx (kg kg d )
' Bx (kg kg-1d-1)

0,8
1,25

0,7
1,0

0,4
0,5

0,2
0,25

Se grafica rx contra ' Bx segn la figura 5.5. De la pendiente se obtiene el


valor de Yc y del intercepto en el eje de ordenadas el valor de b.
Yc = 0,6
b = 0,075

5.3.2. Capacidad de oxigenacin necesaria


Una vez que se conoce la capacidad de oxigenacin del implemento de
aereacin, debe compararse con la que necesita el sistema, o sea, la capacidad
de oxigenacin necesaria (CO)n.
Cuando se disea un tanque de aereacin debe calcularse la (CO)n de forma de conocer el oxgeno que se necesita para garantizar el suministro que
requieren las reacciones bioqumicas que tienen lugar y para el mantenimiento
de las condiciones aerobias en el medio.

177

Menndez Gutirrez, C. y J. Prez Olmo

CO n Y c %V

 b X

Cs,m
Cs,m  C

(5.16)

donde:
'Bv: Razn de carga volumtrica, kg m d.
X: Concentracin de SSV en el tanque de aereacin, kg m.
C: Concentracin lmite permisible de oxgeno disuelto, mgL.
La masa de oxgeno que se disuelve en el agua en la unidad de tiempo en
condiciones normales (20 oC y 101,3 kPa y cero oxgeno disuelto) es:

(CO) n

G 10 3 k L a20 C Cs, m20 CV

(5.17)

donde:
G: velocidad msica de disolucin (absorcin) de oxgeno, kg h.
kLa: h .
Cs,m: mgL.
V: volumen del agua que se est aereando, m3.
Ejemplo 5.4
Determine la capacidad de oxigenacin necesaria para satisfacer las exigencias de un proceso de lodo activado del que se dispone de la siguiente informacin:
t: 28 0 C
Yc : 0,40
b: 0,10 d 
' Bv: 0,5 kg m d X = 1,5 kg m 
OD mnimo permisible = 2 mgL
Altura del tanque de aereacin = 3 m
Presin atmosfrica = 101,8 kPa
Solucin:
La concentracin de saturacin de oxgeno a 28 0C es (Tabla 5.1):
Cs = 7,9 mgL
De acuerdo con la ecuacin 5.3:

Cs,m

Cs

Pa  0,5Ph
Po

Pa = 101,8 kPa
Po = 101,3 kPa
Ph = 0,0098 U x h
U = 990 kg m
h=3m
Ph = 29,1 kPa
178

Transferencia de Oxgeno

Cs,m

7 ,9

101,8  0,5 29,1


101,3

9 ,07mgL1

(CO)n = 0,45 kg m d

5.3.3. Otro procedimiento para determinar la necesidad


de oxgeno
En ausencia de clculos ms sofisticados, puede asumirse que para la remocin de la DBO carbnica es necesario utilizar 1,1 kg O2 por cada kg de DBO5
aplicado, para el caso de procesos convencionales. Para la aereacin extendida
el requerimiento de oxgeno puede tomarse aproximadamente como 1,8 kg
de O2 por cada kg de DBO.
Por otro lado, haciendo consideraciones tericas, la necesidad de oxgeno
puede ser determinada a partir de la DBO del agua residual y de la cantidad de
biomasa que diariamente se extrae del proceso.
Por tanto, la demanda terica de oxgeno para la remocin de las sustancias
carbonosas contenidas en el agua residual puede calcularse mediante la relacin:
kg O2d = (kgd DBOu total utilizado) - 1,42 (kgd biomasa extrada)
En la expresin anterior ha sido considerado que 1,42 es el equivalente
de oxgeno de la biomasa, asumiendo que la frmula emprica de esta es
C 5 H7 N O2 .
De acuerdo con esto,
k gO2 d

1

Q S 0 S 10 3 kg / g
f

 1,42 P

donde:
' P: es la biomasa producida (kgd).
f: es la relacin entre DBO5 y DBOu.
En el desarrollo del epgrafe 5.3, solo se consider el requerimiento de oxgeno para satisfacer las necesidades de consumo del agua residual y de la
biomasa. Sin embargo, el suministro debe ser adecuado para:

satisfacer la DBO del agua residual,


satisfacer la respiracin endgena de la biomasa,
proporcionar un adecuado rgimen de mezcla y
mantener una concentracin de oxgeno disuelto en el reactor, que no
inhiba el proceso aerobio (generalmente superior a 0,2 mgL).
179

Menndez Gutirrez, C. y J. Prez Olmo

5.4. IMPLEMENTOS DE AEREACIN


El equipamiento para producir aereacin que es ms comnmente utilizado
puede clasificarse en:
a) sistemas de aire difundido,
b) sistemas de aereacin mecnica.

5.4.1. Aire difundido


Los sistemas de aire difundido estn constituidos por difusores sumergidos en el agua, las lneas o tuberas conductoras de aire, los sopladores o
compresores, y su equipamiento auxiliar.
Bsicamente se conocen dos tipos de difusores: los que producen burbujas pequeas a partir de un medio poroso y aquellos que utilizan tuberas
horadadas o algn otro dispositivo para producir burbujas grandes o medianas. Todos estos implementos son muy diversos en dependencia de los fabricantes. Figura 5.6.
Los difusores se distribuyen a ambos lados y en toda la longitud del
tanque de aereacin. Figura 5.7. Como elemento prctico puede sealarse
que para mantener una mezcla adecuada, el ancho del tanque de aereacin
debe ser aproximadamente el doble de su profundidad. Este ancho puede
duplicarse si adems se concibe una lnea central de unidades de difusin.
Tambin puede apuntarse que la ubicacin de los difusores debe ser tal que
los puntos de salida del aire deben estar unos de otros a una distancia comprendida entre 15 y 75 cm.
Los difusores de burbuja fina se construyen de granos de slice u xido
de aluminio. Otras unidades son tubos recubiertos de diversos compuestos
como nylon, dracn y saran. El dimetro de la burbuja suele estar comprendido entre 2 y 2,5 mm. Estas unidades se disean para lograr un flujo de aire
por unidad de 3 a 28 m3 hen condiciones estndar (P = 101,3 kPa, 20 0 C).
Los difusores no porosos originan burbujas de hasta 25 mm de dimetro. Estas burbujas grandes son la causa de que estos difusores presenten
menor rendimiento que los porosos, pero por otro lado presentan las ventajas de tener menor costo, y menos necesidades de mantenimiento y de
limpieza de aire.
La aereacin mediante difusores es recomendada fundamentalmente para
profundidades de lquido en el reactor entre 2,5 y 5,0 m y valores de velocidades de consumo de oxgeno inferiores a 1 mgL  min .

180

Transferencia de Oxgeno

Fig. 5.6. Implementos para la aereacin mediante aire difundido.


a) difusor poroso para burbujas finas. b) Tubos para burbujas gruesas.

Fig. 5.7. Distribucin de los difusores. Vista de planta.

5.4.2. Aereacin mecnica


Los aereadores mecnicos pueden ser superficiales o sumergidos, y ambos
a su vez, de eje vertical o de eje horizontal. En los aereadores mecnicos el
oxgeno se toma de la atmsfera. Hay modelos de aereadores mecnicos
sumergidos, las turbinas, en los que adems se introduce aire por la parte
inferior del tanque de aireacin. Figura 5.8.

181

Menndez Gutirrez, C. y J. Prez Olmo

Entre las funciones de un aereador mecnico se pueden citar: 9,12


a) Disolver oxgeno en el tanque de aereacin.
b) Mantener los SSV en suspensin.
c) Distribuir el afluente por todo el volumen del tanque de aereacin.
d) Remover parte del CO2 que se genera.

(b)
Fig. 5.8 Aereadores mecnicos. a) Aereador superficial. b) Aereador
de turbina.
182

Transferencia de Oxgeno

5.4.2.1. Aereadores superficiales


Los aereadores de superficie, equipos para la introduccin de grandes cantidades de aire, y por tanto de oxgeno, consisten en turbinas de alta o baja
velocidad o en unidades flotantes de alta velocidad que giran en la superficie del
lquido, parcialmente sumergidos.
Cualquier superficie de agua tiende a absorber oxgeno del aire. Este proceso se acelera cuando hay turbulencia en la superficie.
Los aereadores superficiales llevan este proceso a una etapa superior al
romper mecnicamente la superficie del lquido y crear una interfase gas-lquido
en forma atomizada de pequeas gotas de agua y burbujas de aire, arrastradas
dentro del lquido. Adems el volumen total de lquido circula y se mezcla de
manera tal que el agua oxigenada se reemplaza continuamente por lquido de
otras zonas.
Los aereadores superficiales de eje vertical pueden montarse sobre una
estructura rgida o sobre flotadores. Los de eje horizontal tienen su origen
en los aereadores Kessener de cepillo y hoy se fabrican de diferentes tipos.
Figura 5.9.

Fig. 5.9. Aereadores superficial de cepillo.


La transferencia de oxgeno ocurre a travs de la accin de vrtice y por la
exposicin a la superficie de grandes volmenes de agua atomizada.
Para los implementos de eje vertical, la cantidad de oxgeno transferido al
lquido es funcin de la potencia.

183

Menndez Gutirrez, C. y J. Prez Olmo

La velocidad de transferencia depende de:


dimetro del implemento,
velocidad de rotacin y
profundidad del elemento rotatorio.
En condiciones ptimas de inmersin, la velocidad de transferencia por potencia consumida es relativamente constante dentro de un amplio intervalo de
valores de dimetro del impelente.
Kormanik13 obtuvo una correlacin entre la velocidad de transferencia de
oxgeno y la potencia por unidad de rea. As, para aereadores superficiales de
alta velocidad, puede considerarse la expresin:
Np = 1,973 P  1,0045
Np: kg O2 (kW-h)
P: kW m
Para mantener una concentracin uniforme de oxgeno disuelto se necesita disponer de niveles de potencia de 1,2 a 2 Wm . Para mantener slidos
biolgicos en suspensin es recomendable una velocidad mnima en el fondo
de 12 cm s cuando la concentracin es de 5gL de SSLM.
Cuando se emplea este tipo de aereador, la altura mnima de tanque recomendada es de 1,8 a 2,4 m y mxima entre 3,7 a 4,9 m.
Los aereadores superficiales se utilizan cuando la velocidad de consumo de
oxgeno del sistema est en el intervalo de 1,0 a 1,4 mgLmin. Para mayores
velocidades de consumo se recomienda el empleo de aereadores de turbina.

5.4.2.2. Aereadores sumergidos


Las unidades de aereacin a travs del empleo de turbinas dispersan el aire
comprimido mediante la accin cortante de un impelente rotatorio y promoviendo adems la mezcla del contenido del tanque de aereacin.
Los aereadores sumergidos de eje horizontal funcionan por el mismo principio que los superficiales excepto que la agitacin del agua se lleva a efecto con
discos o paletas acoplados al eje.
Las turbinas se distribuyen en el tanque de aereacin de manera tal que
exista una cada 85 a 220 m2.
Las potencias de los aereadores mecnicos disponibles varan de 0,75
a 100 kW.

184

Transferencia de Oxgeno

5.4.3. Capacidad de los aereadores


Los fabricantes de equipos de aereacin generalmente ofrecen la capacidad de transferencia de oxgeno de su equipamiento sobre la base de la masa de
oxgeno transferido por unidad de energa (kg de oxgeno transferido por kW h)
en condiciones normales (C.N.). Las condiciones normales o estndar estn
definidas para agua de acueducto, a 20 0 C, cero concentracin de oxgeno disuelto y la presin correspondiente al nivel del mar (P = 101,3 kPa). Esto no es
ms que la capacidad de oxigenacin del implemento.
Para las condiciones de campo o reales (C.R.) la capacidad de transferencia o rendimiento, No, reportada en (C.N.) por el fabricante, deben ser rectificadas,

N0

Cs,m  C
Cs20 oC

1,02t  20

(5.18)

N: capacidad de transferencia de oxgeno en condiciones de trabajo o reales


kg O2(kW h).
No: capacidad de transferencia de oxgeno en condiciones estndar.
kg O2(kW h).
Cs,20C: concentracin de saturacin de oxgeno disuelto a la temperatura de
20 0C mgL.
Cs,m: concentracin de saturacin de oxgeno disuelto en agua limpia a la
temperatura y presin de trabajo mgL.
C: Concentracin de oxgeno disuelto deseada mgL.
t: temperatura de trabajo oC.
Cuando se evala la capacidad de transferencia de oxgeno en sistemas de
aire difundido, es necesario valorar no solo la eficiencia de los difusores, sino
tambin la capacidad de los sopladores o compresores. Esta informacin est
disponible generalmente en los manuales de operacin y mantenimiento que
ofrecen los proveedores de los equipos.
Por tanto, para determinar la transferencia de oxgeno del sistema de aereacin
por difusin en kgd, adems de necesitar la capacidad de transferencia de los
difusores en condiciones reales, debe conocerse la capacidad de los sopladores
en condiciones estndar (m3min a 20 qC y la presin de 101,3 kPa). Con estos
dos elementos puede calcularse la masa de aire que el sistema es capaz de
entregar. Ver el ejemplo 5.8.
Cuando la capacidad de transferencia que se necesita calcular sea la de
un aereador superficial, debe determinarse la potencia de trabajo del motor, m,
y la capacidad o rendimiento, kg O 2 (kw-h) , del aereador en particular. En
el caso de no existir otra informacin, puede asumirse que la eficiencia del
185

Menndez Gutirrez, C. y J. Prez Olmo

motor es 90 % y la de los elementos mecnicos del aereador 85 %. De


acuerdo con esto ha de considerarse que 75 % de la potencia nominal del
motor del aereador superficial es convertido en energa para la transferencia de oxgeno. Ver ejemplo 5.9.
Tabla 5.6. Intervalos tpicos de valores de capacidad de transferencia de
oxgeno para algunos de los tipos de aereadores

Tipo de aereador
Superficial (baja velocidad)
Superficial (alta velocidad)
Turbina
Rotor de cepillo y placas
Difusores de burbuja fina
Difusores de burbuja gruesa

No kg O2 (kWh)1
1,21 3,04
1,21 2,19
1,21 2,25
0,91 2,19
1,80 3,20
0,90 1,50

5.4.4. Eficiencia de la transferencia de oxgeno


La eficiencia de absorcin o de la transferencia de oxgeno viene definida
para difusores y las turbinas como los kg hde oxgeno transferido por kg hde
oxgeno suministrado.
Los fabricantes definen la Eficiencia Estndar de Transferencia de Oxgeno (Eo) (Standard Oxygen Transfer Eficiency, SOTE) de sus implementos de aeracin para agua de acueducto, a 20 0 C, cero oxgeno disuelto y a
nivel del mar.

k L a20 o C Cs20 o C
16,56 Qs

(5.19)

Eo: Eficiencia estndar de transferencia %.


kLa: Coeficiente de transferencia de oxgeno h.
Qs: Flujo de aire m3 min, a 101,3 kPa, 20 0C.
V: Volumen del tanque de aereacin.
Los valores de Eo deben ser rectificados para las condiciones normales
de trabajo, (eficiencia real o actual de transferencia de oxgeno, E), (AOTE),

E0

Cs,m  C 1,02 t  20
Cs20 oC

186

(5.20)

Transferencia de Oxgeno

La eficiencia de absorcin tambin puede ser calculada directamente,

k L a 20 o C Cs,m  C
1,02t  20 V
16 ,56 Qs

(5.21)

En la tabla 5.7 se brinda informacin tpica de la eficiencia de transferencia


de oxgeno de varios tipos de difusores.
Tabla 5.7. Valores tpicos de eficiencia estndar de transferencia de diferentes difusores14

Tipo de difusor
Domos de cermica
Discos de cermica
Placas de cermica
Tubo plstico rgido poroso
Tubo plstico rgido poroso
Difusor no poroso de eje
longitudinal

Profundidad
(m)
4,2
3,7
4,5
4,0
4,5
4,5

E0 (%)
29,0
26,0
30,0
27,0
30,0
12,0

5.4.5. Correlaciones para la transferencia de oxgeno


Eckenfelder15 ha desarrollado una correlacin general para la transferencia
de oxgeno a partir de burbujas que ascienden a travs de una columna de lquido en reposo:

K L d s 13
H
DL

d
C B B

(5.22)

H: profundidad del lquido.


dB: dimetro de la burbuja.
X B: velocidad de la burbuja.
U : densidad del lquido.
P : viscosidad del lquido.
C: constante.
Si se considera que para burbujas de aire en el tanque de aereacin se
cumple que:

A
V

6 Qs H
d B B V
187

(5.23)

Menndez Gutirrez, C. y J. Prez Olmo

la ecuacin 5.20 puede ser expresada en trminos de kLa.


Qs: flujo volumtrico de aire.
A: rea a travs de la cual ocurre la difusin.
V: volumen de lquido.
Para el intervalo de flujo de aire que normalmente se utiliza en la prctica se
cumple que,
dB ~ Qs n
(5.24)
Las ecuaciones 5.20, 5.21 y 5.22 pueden combinarse para obtener una expresin general para la transferencia de oxgeno por difusin

kL a

C c H 2 / 3 Qs 1  n
V

(5.25)

La ecuacin 5.25 puede ser transformada en trminos de masa de oxgeno


transferida por unidad de difusin
G

Cc H 2 / 3 Qs 1  n Cs  C

(5.26)

G = masa de oxgeno transferida en la unidad de tiempo


El comportamiento de todas las unidades de aereacin por difusin puede expresarse utilizando una forma modificada de la ecuacin 5.26
G CQsn H mW  p D(ECs, m  C ) 1,02t  20

(5.27)

G: kg O2 transferido. h (unidad) a t oC y 101,3 kPa.


C: constante.
W: ancho de tanque de aereacin.
n, m, p: caractersticas del implemento de aereacin.
Una correlacin similar a la anterior, pero para el caso de las turbinas es:
G

C N t Qs n d v CsC 1,02 t  20
x

(5.28)

Nt: velocidad perifrica del impelente m s.


d: dimetro del impelente m.
x, n, v: caractersticas del implemento de aereacin.
Es prctica comn utilizar una relacin entre el dimetro del impelente y el
dimetro equivalente del tanque de aereacin entre 0,1 - 0,2 y una velocidad
perifrica del impelente comprendida entre 3,1 - 5,5 m s.

188

Transferencia de Oxgeno

Ejemplo 5.5
Con el propsito de aerear un reactor de 5 000 m3 de volumen efectivo se
requieren 300 kgO2h.
Disee el sistema de aereacin mediante difusores para las siguientes condiciones:
temperatura 28 0C
concentracin de oxgeno disuelto lmite = 2 mgL
E = 0,90
Pa = 101,8 kPa
Presin de aire requerida en la descarga = 156,5 kPa
La informacin que se dispone del proveedor se detalla:
Qs = 0,2 m3 min unidad a 20 0C y 101,3 kPa
D = 0,8
C = 0,2076
n = 1,05
m = 0,70
p = 0,32
Solucin:
Profundidad seleccionada (H) = 4,5 m
ancho = 8,0 m (d 2 H)
longitud = 139 m
Clculo de Cs,m:
a 28 0C: Cs = 7,9 mgL
U = 990 kg m
H = 4,5 m

Cs,m

Cs

Pa  0,5P
h
Po

Ph = 0,0098 Uh
(kPa)
Ph = 0,0098 990 5,0
Ph = 43,66 kPa
Cs,m

7 ,9

101,8  0 ,5 43,66
101,3

9,64 mgL1

Clculo de las condiciones reales de trabajo de los difusores:


G = C Qsn Hm Wp D (E Cs,m - C) 1,02 W
G = 0,2076 0,2 1,05 4,5 0,70 10  >0,8 (0,90 9,64 - 2,0)@ 1,02 8
G = 0,329 kg h unidad a 28 oC
189

Menndez Gutirrez, C. y J. Prez Olmo

Nmero de unidades necesarias:

300 kgh 1
0,329 kgh 1 unidad 1

912 unidades

Espaciamiento entre unidades,

139 m
912 unidades

0,15 m

Flujo de aire del compresor,


Qs = 0,2 x 912 = 182,4 m3 min
Potencia del compresor requerido:
E = eficiencia 70 %
' P = 54,7 kPa (' P = P2P1)
P1 y P2 = presin a la entrada y a la descarga respectivamente.

potencia kW

Q s 3
E

182,4 54,7
0,7 x 60 seg min 1

PT

237 kW

Capacidad de transferencia necesaria en condiciones reales.

GX
PT

0,329 912
1, 27 kgO 2 (kWh) -1
237

Capacidad de transferencia requerida en condiciones estndar.

No

No

N Cs20 0 C

Cs,mC 1,02t 20


1,27 9,2
>0.909,64  2 @ 0,80 1,028

No 1,86 kO 2 kWh 1

Se necesita disponer de un implemento cuya capacidad de transferencia en


condiciones estndar sea superior a 1,86 kg O2 (k w h).
Eficiencia de la transferencia de oxgeno,

190

Transferencia de Oxgeno

O2 absorbido
100
O2 suministra do

Se asume que el oxgeno absorbido es el que se necesita, 300 kg h.


Oxgeno suministrado = 16,56 Qs = 3020 kg h 
E = 9,9 %
La eficiencia de la transferencia en condiciones estndar:

Eo

Cs20 C
(ECsm  C ) D1,02t  20

Eo = 15 %
El dispositivo que se utilice debe tener adems una eficiencia de transferencia de 15 % en condiciones estndard.
Ejemplo 5.6
Para el mismo caso del ejemplo anterior, considere el diseo de un sistema
de aereacin con turbinas.
Informacin adicional del proveedor:
Eo = 20 %
a = 0,85
Pd = 0,95
Solucin:
Cs,m = 9,83 mgL
Se asume como condicin de trabajo que el oxgeno absorbido por el sistema
debe ser igual al oxgeno que se requiere = 300 kg h
Oxgeno suministrado = 16,56 Qs

Eo

oxgeno absorbido
oxgeno suministrado
0,2

0,20

300
16,56 Qs

Qs = 90,5 m3 min (20 C, 101,3 kPa)


Potencia requerida por el compresor (Pc):
e = 70 % 'P = 54,7 k Pa

Pc

90,5 54,7
117,9k W
0,7 60
191

Menndez Gutirrez, C. y J. Prez Olmo

Potencia requerida por la turbina (Pt ):

Pd

Pt
Pc

Pt = 0,95 117,9 = 112 k W


Capacidad de transferencia que se requiere:
300
1
1,25 kgO2 kW h
Pc  Pt

Capacidad de transferencia necesaria en condiciones ideales

N0

N Cs20 0 C

Cs,mC 1,02t 20

No = 1,7 kg O2 (k W h)
Es necesario disponer de un dispositivo cuya capacidad de transferencia en
condiciones de referencia sea superior a 1,7 kg O2 (k W h).

5.5. CONSUMO DE POTENCIA


La potencia terica del compresor o soplador para un sistema de aereacin
por difusin puede calcularse:
Pc = 1,1628 m Wc
(5.29)
Pc: potencia terica del compresor W.
m: flujo msico de aire que se comprime kg h.
Wc: trabajo en el eje del compresor kcal kg.
Asumiendo que para las presiones y temperaturas consideradas en la
compresin el aire se comporta como un gas ideal (lo que es una buena
aproximacin),
Wc

P A
1
A R T1 2  1
P1

29

1

192

(5.30)

Transferencia de Oxgeno

b = constante adiabtica =

Cp
Cv

1,4 (para aire)

Cp, Cv = calores especficos a presin y volumen constantes respectivamente.


P2: presin a la profundidad que se introduce el aire kPa.
R: constante de los gases = 1,987 kcal (mol K).
T1: temperatura de entrada del aire al compresor 0K.
P1: presin del aire a la entrada del compresor kPa.
La ecuacin 5.30 puede expresarse de una forma ms sencilla,
P 0 ,286
1
Wc 0 ,2398T1 2
(5.31)
P1

La potencia real del compresor est determinada por el rendimiento global


del mismo, K ,
K = KT Km
(5.32)
K T: rendimiento termodinmico (desviacin de las condiciones de reversibilidad
adiabtica)
K m: rendimiento considerando las prdidas en el motor y la transmisin

Pc

(5.33)

Para el clculo del requerimiento de potencia en el caso que se empleen


turbinas, ha de considerarse tanto el accionamiento del rotor como el funcionamiento del compresor,
(5.34)
PT = Pc + Pr
PT = requerimiento de potencia total.
Pc = potencia del compresor.
Pr = potencia del impelente o rotor.
Por otro lado, se ha demostrado que la capacidad de transferencia de oxgeno (N) ptima de un aereador de turbina est vinculado a la relacin entre la
potencia del rotor y la del compresor:

Pd

Pr
Pc

(5.35)

Las mejores condiciones de trabajo se presentan cuando el valor de Pd es


cercano a 1.

193

Menndez Gutirrez, C. y J. Prez Olmo

La potencia requerida por el impelente decrece en la medida que aumenta la


cantidad de aire introducido como consecuencia de la disminucin de la densidad de la mezcla bajo aereacin. Por tanto:
Pd  1,0 para altos caudales de aire.
Pd ! 1,0 para bajos caudales de aire
Para evitar la formacin de vrtices se suele emplear tabiques de choques.
Si el tanque de aereacin es circular, se usan 4 tabiques uniformemente distribuidos en el permetro del recipiente. Para tanques cuadrados se colocan 2
tabiques en paredes opuestas. En tanques rectangulares donde la relacin largo:
ancho es mayor de 1,5 no se requiere la colocacin de tabiques de choque.
Ejemplo 5.7
Para un sistema de aereacin por difusin se necesitan 720 kg h de
oxgeno.
Calcular:
a) Rendimiento de la transferencia de oxgeno E.
b) Nmero de unidades de difusin.
c) Potencia real requerida.
d) Temperatura ambiente 28 0C.
e) Presin ambiental 99,96 k Pa.
f) Profundidad de los difusores 4,0 m.
g) Segn el proveedor, el flujo msico de oxgeno entregado por cada
unidad de difusin es 0,8 kgO2 h (20 0C, 101,3 kPa, 0 mgL OD).
D = 0,90
E = 0,92
La concentracin de oxgeno mnima deseada en el tanque de aereacin es
2,5 mgL,
Cs20 C = 7,9 mgL (101,3 kPa)

U agua = 990 g L (28 0 C)


Solucin:
a) Cs,m

Cs

Pa  0,5Ph
Po

Po = 101,3 k Pa.
Pa = 99,96 k Pa.
Ph = 0,0098 U,h = 0,0098 990 4,0.

194

Transferencia de Oxgeno

Ph = 38,81 k Pa.
Cs,m = 9,31 mgL
La relacin entre la transferencia de oxgeno en las condiciones reales y las
de referencia viene dada por:

Cs,m  C 1,02t 20

E
Eo
E

0,8

9,2

0,92 9,31  2,5


9 ,2

0 ,90 1,028

E = 0,556 kg O2 h por unidad de difusin


b) Nmero de unidades de aereacin:
U

720
0,556

1295 unidades

c) Pc = 1,1628 m Wc
Wc

P 0 ,286
 1
0,2398 T1 2
P1

P1 = 99,96 kPa
T1 = 28 0C = 301 0K
P2 = Po + Ph = 101,3 + 38,81 = 140,11 k Pa
Wc = 7,32 kcal kg
m = 720 kg h
Pc = 6128 W (potencia terica)
Ejemplo 5.8
Una planta tiene 3 sopladores centrfugos, cada uno de ellos con una capacidad en condiciones reales de trabajo de 43 m3 min. Dos de ellos estn en
operacin y el tercero en reposo. Determine la cantidad de aire que es capaz de
entregar el sistema.
Se dispone de la siguiente informacin:
Eficiencia estndar de transferencia de oxgeno de los difusores = 12 %
(SOTE)
Profundidad de los difusores en el tanque de aereacin = 3,5 m
Presin = 101, 5 kPa; T = 28 qC;
Densidad del agua = 990 kg m
D = 0,85; E = 0,90
195

Menndez Gutirrez, C. y J. Prez Olmo

Cs 28 C = 7,92 mgLCs 20 C = 9,2 mgL; C = 2,0 mgL


Cs,m
Ph

0,0098 h

C s,m

7 ,92

Po  0 ,5 Ph
101,3

Cs

0,0098 990 3,5

101,5  0,5 33,96


101,3

0 ,12

0 ,90 9 ,262
9 ,2

0,082

8,2%

33,96 kPa

9,26 mgL 1

0,85 1,02

El clculo del flujo msico de oxgeno que entregan los 2 aereadores se


realiza utilizando la eficiencia actual de los difusores y la capacidad estndar de
los sopladores.
Convertir la salida de los sopladores de las condiciones reales a las estndar
(20 oC, 101,3 kPa):

r m3 min 1

Ps
s m min . Tr

Ts Pr

T en o R R

1

9 o
C  491,67
5

Ts

9
20  491,67
5

527 ,67 o R

Tr

9
28  491,67
5

542,07 o R

527,67 101,5
43

542,07 101,3

196

Transferencia de Oxgeno

s = 41,9 m3 min .
El flujo msico de oxgeno que entregan los 2 aereadores se calcula utilizando la eficiencia actual de los difusores y la capacidad de los sopladoes en condiciones estndar (41,9 m3 min ),
kg O2 d  = 2 41,9 m3 min  1440 min.d 0,232 kgO2(kg aire) .
1,29 kg aire m 0,095 = 3430
por tanto, dos sopladores, que cada uno entrega 41,9 m3min  van a transferir
3430 kg d.
Ejemplo 5.9
En una instalacin existen dos aereadores superficiales mecnicos de 40
kW cada uno que son utilizados simultneamente. Segn reporta el fabricante, el SOTR es de 1,8 kg O2 (kW - h) . Calcule el oxgeno transferido si
adems se brinda la siguiente informacin:
D = 0,85; E = 0,90; Csm28 C = 7,92 mgL
C = 2 mgL

T = 28 oC;

N0

Cs  C
Cs20

1,02 t  20

0 ,9 7 ,92  2
N 1,8
0 ,85 1,02 8
9,2

N = 0,99 kg(kW- h)


Por otro lado, se asume que la potencia utilizada para transferir el gas es el
75% de la nominal del motor: (0,90 0,85)
Por lo tanto, la potencia que es utilizada para transferir el gas es:
pkW = 0,75 40 = 30 kW
O sea, el oxgeno transferido es,
kg O2 d  = N pkW = 0,99 60 24 h d  = 1426 kW

Notas bibliogrficas
1 LEWIS, W.K. AND W. G. WHITMAN3ULQFLSOHVRIJDVDEVRUSWLRQInd. Eng.
Chem., vol. 16, no. 8, 1924.
2 DANCKWERTS, D. V.6LJQLILFDQFHRIOLTXLGILOPFRHIILFLHQWLQJDVDGVRUSWLRQ
Ind. Eng. Chem, vol. 43, no. 1, 1951.
197

Menndez Gutirrez, C. y J. Prez Olmo

3 TEWARI, P. K. AND J. K. BEWTRA:$OSKDDQG%HWDIDFWRUVIRUGRPHVWLFZDV


WHZDWHU-RXURIWater Poll. Control Fed., vol. 54, no. 9, 1982.
4 ECKENFELDER, W. W.: Water Quaility Engineering for Practicing Engineers,
CBI Publishing, EE.UU., 1980.
5 MENNDEZ, C. L. Y J. PREZ:&DSDFLGDGGHR[LJHQDFLyQGHUHDFWRUHVELROyJL
FRVHQHOWUDWDPLHQWRGHDJXDVUHVLGXDOHVIng. Hidrulica, vol. 3, no. 2,
1982.
6 ECKENFELDER, W. W. AND J. OcCONNOR: Biological Waste Treatment, Pergamon Press, Oxford, 1981.
7 MENNDEZ, C. L. Y L. GUERRA:/DJXQDDHUHDGDHQHOWUDWDPLHQWRGHUHVLGXD
OHVWHQHURVIng. Hidrulica, vol. 9, no. 4, 1988.
8 HOOVER, S. R. AND S. H. PORGES:$QLQWHUSUHWDWLRQRIWKH%2'WHVWLQWHUPV
RI HQGRJHQRXV UHVSLUDWLRQ RI EDFWHULD Sewage and Industrial Wastes,
vol. 24, no. 7, 1957.
9 CHUBOBA, J., C. MENNDEZ Y J. PREZ: Fundamentos Tericos de Algunos
Procesos para la Purificacin de Aguas Residuales, Ed. ISPJAE, Ciudad de La Habana, 1986.
10 MENNDEZ, C. L.: Wastewater Treatment from the pulp and paper industry, Ph.D. Thesis, Prague, 1986.
11 VASICEK, P.8VHRIDNLQHWLFVWXG\WRRSWLPL]HWKHDFWLYDWHGVOXGJHSURFHVV
Jour. of Water Poll. Control Fed., vol. 54, no. 6, 1982.
12 CLOUGH, G. F.: 3K\VLFDO FKDUDFWHULVWLFV RI PHFKDQLFDO DHUDWRUV -RXU RI
Water Poll. Control Fed., vol. 46, no. 2, 1974.
13 KORMANIK, R. AND ET AL.: Proc. 28 th Ind. Waste Conf. Purdue University,
1973.
14 REDMON, D. T. AND ET AL.7UDQVIHUHIILFLHQF\PHDVXUHPHQWVLQPL[HGOLTXRU
XVLQJ RIIJDV WHFKQLTXHV -RXU RI Water Poll. Control Fed., vol. 55,
no. 11, 1987.

198

Filtros Percoladores

CAPTULO 6

FILTROS PERCOLADORES

6.1. INTRODUCCIN
Los filtros percoladores o ms correctamente denominados lechos
bacterianos, son sistemas de depuracin biolgica de aguas residuales, en
los que la oxidacin de la materia orgnica se produce al hacer pasar, a
travs de un medio poroso cubierto de una pelcula biolgica, aire y agua
residual. El agua residual fluye sobre la superficie del medio poroso o empaque en una delgada capa que est en contacto con la pelcula biolgica por
un lado y con el aire en los espacios intersticiales del empaque por el otro. El
fundamento del proceso est basado en las acciones producidas en todo el
espesor de la pelcula biolgica.
A favor del empleo de los filtros percoladores est el hecho demostrado, que
los cultivos fijos a un soporte o medio son afectados en menor medida, que los
cultivos suspendidos ante cambios de las condiciones ambientales, que prevalecen en un momento dado en las condiciones de trabajo del dispositivo de
tratamiento.1
La pelcula biolgica est constituida por un conjunto complejo de
microorganismos aislados y colonias de ellos, embebidos en una matriz de
polmeros, cuya estructura y composicin es funcin de la edad de la
biopelcula y de las condiciones ambientales. 2
Como sucede con todos los sistemas de tratamiento biolgico de aguas
r esiduales, los tipos y pr opor ciones de las diferentes especies de
microorganismos presentes en la poblacin microbiana dependen de las condiciones de operacin del sistema, especialmente la carga, y la composicin
del agua residual.
Las condiciones aerobias se mantienen por el flujo de aire a travs del
empaque o empaquetadura del lecho. La circulacin del aire se realiza de
forma natural o forzada, a contra corriente o en el mismo sentido del flujo
de agua. Este flujo de aire es inducido por la diferencia entre el peso especfico del aire atmosfrico dentro y fuera de la empaquetadura. En estos
199

Menndez Gutirrez, C. y J. Prez Olmo

sistemas, en muy raros casos, se emplea la ventilacin forzada. En la figura 6.1 se muestra una vista de una planta depuradora que emplea filtros
percoladores.

Fig. 6.1. Vista de una instalacin que emplea filtros percoladores.


Peridicamente existen desprendimientos de la pelcula biolgica. Esto hace
necesario el empleo de sedimentadores secundarios para la separacin del slido de la corriente de lquido tratado. La seccin transversal de un filtro se muestra en la figura 6.2. Tambin es recomendable el uso de sedimentadores primarios
para evitar tupiciones en el material de relleno que dificulten el libre paso de
agua y aire a travs del mismo.

Fig. 6.2. Seccin de un filtro percolador circular de brazo mvil rotatorio.


200

Filtros Percoladores

6.2. PARTES DE LAS QUE CONSTA UN FILTRO


PERCOLADOR. MECANISMO DE REMOCIN
DE LA DBO
6.2.1. Partes de que consta
Los filtros percoladores constan de 3 partes principales:
Sistema de distribucin.
Empaquetadura.
Sistema recolector.
El sistema de distribucin debe proporcionar una carga hidrulica uniforme sobre la superficie del filtro. Por tanto, han de evitarse atascos y paradas. Los aspersores
para la distribucin del agua residual pueden ser fijos o circulares dependiendo de
que la estructura del filtro sea rectangular o circular respectivamente.
Los aspersores fijos requieren un dispositivo ms complejo de distribucin y,
por tanto, una mayor prdida de carga (alrededor de 2 m). Los mviles consisten en brazos giratorios, 2 4, que se disponen radialmente, y son movidos por
carga hidrulica. La prdida de carga prevista para estos casos es aproximadamente de 0,5 m. La velocidad de rotacin es de 0,3 a 5 vueltas por minuto,
dependiendo de la carga hidrulica a la que se desea someter el percolador.

Fig. 6.3. Filtro percolador rectangular de distribuidor mvil no rotatorio.


201

Menndez Gutirrez, C. y J. Prez Olmo

A travs del sistema recolector se extrae el efluente del tratamiento y se


produce adems la circulacin del aire.

Fig. 6.4. Sistema recolector de un percolador.


La recogida de agua residual tratada se efecta por medio de un dispositivo de drenaje en el fondo del lecho bacteriano. Este sistema debe tener
previsto un sistema de canales de recogida, con la caracterstica fundamental de que no debe existir sedimentacin en los mismos, ya que el agua residual contiene los flculos que sedimentarn en el decantador secundario.
Para ello la pendiente ser del 1 % 2 %, y la seccin no ir nunca llena de
agua, para que pueda realizar su funcin como canal de aereacin. Una
recomendacin de diseo indica que la zona de salida al falso fondo de agua
y aire sea 15 % 20 % de la superficie total del filtro.
La empaquetadura o relleno constituye el medio de soporte de la biomasa.
Las dos propiedades ms importantes de los filtros percoladores son la
superficie especfica y el porcentaje de huecos del empaque. La misma se
define como los m2 de superficie de relleno por m3 de volumen total del
empaque o relleno. Cuanto mayor sea la superficie especfica mayor ser la
cantidad de limo biolgico presente por unidad de volumen. Por otra parte
mientras mayor sea la proporcin de huecos en el empaque, se podr trabajar con mayores valores de carga hidrulica y con menor riesgo de que
ocurra inundacin por tupiciones.
El objetivo de los medios de empaque es el de proporcionar un soporte slido
y estable para el limo biolgico, y exponer la mxima rea superficial al flujo del
lquido que se desea tratar, y de superficie mojada al contacto del aire.
El material que se utiliza para el relleno o empaque de los filtros debe poseer:
Resistencia mecnica.
Resistencia qumica.
Alta relacin rea/volumen.
202

Filtros Percoladores

Aunque el material de relleno puede ser de granito, coke o escoria entre


otros materiales, los medios ms comnmente empleados son el estndar de
piedra y material plstico.
Medio de piedra
a)
b)
c)
d)

Tamao de la piedra 3-15 cm.


rea especfica Av = 40 - 80 m2 m.
Peso especfico U = (2 - 3). 103 kg m.
Espacio vaco 50 % del volumen del empaque.

Material plstico
a)
b)
c)
d)

Medio de diferentes formas, tamao y materiales.


rea especfica Av = 20 - 250 m2 m.
Peso especfico U = 50 - 90 kg m.
Espacio vaco 90-95 % del volumen del empaque.

Las principales ventajas que sobre el medio estndar de piedra tiene el medio de plstico son:
a) Alta rea especfica.
b) Mayor espacio vaco.
c) Bajo peso especfico.
El limo que se desarrolla sobre los medios plsticos es, ecolgicamente,
similar al que se forma sobre los medios minerales convencionales, que contienen bacterias del tipo zoogleas, protozoos y macroinvertebrados.
Teniendo en cuenta que generalmente los filtros percoladores con empaques plsticos operan comnmente a mucha mayor carga hidrulica que
los convencionales de piedra, la eliminacin del exceso de limo o pelcula
biolgica es el resultado de la accin de lavado por el flujo de agua ms que
por la accin de la actividad de los organismos macroinvertebrados.
La principal desventaja del medio plstico es su alto costo, aunque compensado en cierta medida por el menor costo de las paredes y piso de la estructura que
lo contiene, al ser ms sencillos que si el medio soporte empleado es la piedra.
Los materiales de plstico de los que ms comnmente se dispone pueden
ser de distribucin ordenada o distribucin aleatoria, tambin llamada catica o
desordenada.
El empaque que se oferta para colocar en disposicin ordenada se presenta,
generalmente, en paquetes (estructuras modulares) de configuracin
paraleleppeda, de hojas conformadas y encoladas (por ejemplo, Flocor o Plasdek).
Tambin pueden ser de forma tubular (tubos de 8 a 10 cm de dimetro) que
llevan tabiques internos para aumentar la superficie especfica; estos tubos se
203

Menndez Gutirrez, C. y J. Prez Olmo

colocan a lo largo de toda la altura del filtro por capas (por ejemplo, el Bionet).
Figura 6.7.

Fig. 6.5. Colocacin del material de empaque en un filtro.


La manera ms econmica de contener los empaques aleatorios es dentro
de estructuras de seccin circular, y los sistemas modulares en estructuras
rectangulares. Por lo tanto, el tipo de empaque plstico prcticamente define
tambin el sistema de distribucin a emplear.
La altura del empaque dentro del filtro percolador comnmente oscila entre
1,5 y 8 metros. Cuando se emplea un medio convencional de piedra la altura
ms utilizada es la de 2 m.

Fig. 6.6. Filtro percolador tipo torre con empaque plstico.


204

Filtros Percoladores

Los tamaos de rido recomendados, como ya fue expresado, oscilan de


3 a 15 cm de dimetro. Especial atencin debe prestarse a la uniformidad
del medio, ya que mientras ms uniforme sea su tamao, mayor proporcin de
huecos o espacios vacos tendr el empaque, para un tamao dado del mismo.
Los materiales disponibles para su distribucin dentro de la estructura del
filtro de forma aleatoria estn constituidos por elementos individuales de un tamao de 4 a 10 cm, dispuestos directamente en el lecho sin ninguna combinacin (por ejemplo, Norpac). Figura 6.7.

Fig. 6.7. Algunos tipos de empaques plsticos: a) Bionet. b) Norton.


Actifil. c) Norpac. d) Plasdek.
205

Menndez Gutirrez, C. y J. Prez Olmo

6.2.2. Mecanismo de remocin de la DBO


El mecanismo de remocin de los compuestos orgnicos en el filtro
percolador es similar al del proceso de lodo activado.
Una fraccin del volumen del lquido que se aplica al filtro pasa rpidamente
a travs del empaque y otra parte del flujo percola ms lentamente ponindose
en mayor contacto con el limo biolgico. Los contaminantes suspendidos son
rpidamente removidos por adsorcin y coagulacin. La oxidacin ocurre fundamentalmente en la fraccin del flujo que tiene mayor tiempo de retencin.3
La materia orgnica contaminante del agua es degradada en la pelcula biolgica. Esta pelcula no debe tener ms de 3 mm de espesor ya que no se puede
asegurar la accin del oxgeno en espesores mayores.4,5
La pelcula biolgica se forma por adherencia de los microorganismos al
rido y a las partculas orgnicas, formando la biopelcula . Al aumentar el espesor de la biopelcula, las capas ms internas de biomasa van tornndose anaerobias
al irse reduciendo paulatinamente la concentracin de oxgeno disuelto disponible. En la capa ms cercana al medio soporte, completamente anaerobia, se
produce desprendimiento de gases y rotura de la pelcula, perdiendo la capacidad de adherencia al medio poroso. Se desprende la pelcula, siendo arrastrada
por el agua residual y conducida a la decantacin secundaria, donde se producir la sedimentacin. Figura 6.8.

Fig. 6.8. Representacin esquemtica de la pelcula biolgica sobre un


elemento de empaquetadura.

6.3. PARMETROS BSICOS DEL PROCESO.


DEFINICIONES
Los principales parmetros que caracterizan la operacin de los filtros
percoladores son la carga hidrulica, carga orgnica, la eficiencia de purificacin y la relacin de recirculacin.
206

Filtros Percoladores

Carga hidrulica (m3 de agua residual aplicada por m2 por da)

Qo
3
 
Af (m m d )

(6.1)

Q0: Flujo (m3 d)


Af: rea de la seccin transversal del filtro (m2)
Carga orgnica (kg DBO5 aplicada por m3 por da)
Bv

Qo So
Af H

So
 
H (kg m d ),

(6.2)

S0: DBO5 del afluente


H: Profundidad o altura del empaque (m)
Eficiencia de purificacin

S o  S2
100 (%)
So

(6.3)

S2: DBO5 del efluente


Relacin de recirculacin R

Qr
Qo

Qr: Flujo de recirculacin


Un diagrama de flujo tpico se representa en la figura 6.9.

Fig. 6.9. Diagrama de flujo de un sistema de filtro percolador. 1) Sedimentador primario. 2) Filtro. 3) Sedimentador secundario.

6.4. CLASIFICACIN DE LOS FILTROS PERCOLADORES


Los filtros pueden clasificarse, de acuerdo con los parmetros tecnolgicos
bajo los cuales se operan, en cuatro grupos:
a) De baja velocidad.
b) De velocidad media.
207

Menndez Gutirrez, C. y J. Prez Olmo

c) De alta velocidad.
d) De velocidad super alta.
Filtros de baja velocidad:
Generalmente no emplean recirculacin
X P3 m d
Bv = 0,2 kg m d
Filtros de velocidad media:
La dosificacin es continua, con recirculacin
X P3 m d
Bv NJP d
Filtros de alta velocidad
Usualmente de empaque sinttico y recirculacin
X P3 m d
Bv NJP d
Filtros de velocidad super alta

Empaque sinttico y recirculacin


Profundidades mayores a los 6 m
X P3 m d
Bv NJP d

Los filtros con empaque de piedra no deben operarse con cargas hidrulica y orgnica con valores en el intervalo de 5,0 a 15 m3 m d y 0,2 a 0,7
kgm d respectivamente. Al operar un filtro de piedra en este intervalo se
corre el riesgo de tupicin del mismo. Esto no ocurre con los filtros con
empaquetaduras plsticas. Estos ltimos pueden ser operados dentro de todo
el intervalo de valores de cargas. 6

6.5. RECIRCULACIN
Una de las razones que justifican la introduccin de la recirculacin es
para evitar la tupicin del filtro. Para flujos elevados el limo o cieno biolgico se remueve ms fcilmente del filtro a travs de la autolimpieza.
La aplicacin de la recirculacin tiene como consecuencia adems una
dilucin de la concentracin de las sustancias orgnicas del agua residual.

208

Filtros Percoladores

La concentracin de DBO de la mezcla puede calcularse por un balance de


materiales a la entrada del filtro,
S m Qo  Qr

Qo So  Qr S 2

(6.4)

So  R S2
1 R

Sm

Para el empleo del empaque de piedra en el tratamiento de aguas


residuales urbanas usualmente se recomienda que la DBO5 del agua residual a la entrada al filtro no sobrepase de 100 a 150 mg/l. Puede considerarse que un agua residual urbana decantada tiene una DBO5 media del orden
de 200 mg/l. Una recirculacin R del 100 al 150 % es suficiente, en general,
para respetar esta regla. 7
Para aguas residuales con slidos suspendidos debe ser utilizado un
sedimentador primario a la entrada del filtro, fundamentalmente cuando la
empaquetadura de este sea de piedra. Si el agua residual contiene bajo nivel de
slidos suspendidos y el medio filtrante es de material plstico puede obviarse
en ocasiones el uso de la sedimentacin primaria.

6.6. FACTORES QUE INCIDEN EN LA EFICIENCIA


DE PURIFICACIN
Entre los factores que inciden en la eficiencia de purificacin pueden citarse, como los principales, el tiempo de contacto con el limo biolgico, la temperatura, la cantidad y actividad de los microorganismos en el limo y la transferencia
de oxgeno a la pelcula de limo.

6.6.1. Tiempo de contacto


El tiempo de contacto promedio a travs del filtro puede expresarse por la
relacin: 8

H
n

donde:
H: Profundidad del filtro.
X: Carga hidrulica.
C y n: Constantes que varan con el tipo de empaque.

209

(6.5)

Menndez Gutirrez, C. y J. Prez Olmo

La experiencia demuestra que el tiempo de contacto promedio en el filtro


vara entre 5 y 60 minutos en dependencia de la carga hidrulica.9 La existencia
del limo tambin incrementa el tiempo de retencin. Este tiempo tiende a
incrementarse debido al almacenamiento de agua por capilaridad. No obstante,
Mehta y col.,10 sealan que los anlisis del tiempo de retencin en los percoladores
carecen de importancia y solo dificultan distinguir los aspectos fundamentales
en discusin.
El efecto de la carga hidrulica del filtro sobre la eficiencia se ilustra en la
figura 6.10.
En la figura 6.10 se observa que para un valor de carga hidrulica constante,
la eficiencia de remocin de DBO se mantiene tambin constante, independientemente de la concentracin del afluente. Por otro lado, para un valor fijo de
DBO de entrada, la eficiencia de remocin disminuye cuando se incrementa la
carga hidrulica.11
Como se ver ms adelante, esta propiedad es reflejada en el modelo de
primer orden que describe el comportamiento de los filtros percoladores.

Fig. 6.10. Influencia de la carga hidrulica sobre la eficiencia.

6.6.2. Temperatura
El efecto de la temperatura sobre el funcionamiento de los percoladores es el
resultado de dos efectos opuestos. Por un lado, la difusividad de los contaminantes
y el oxgeno aumenta al elevarse la temperatura, as como tambin se incrementa
la actividad biolgica; por otra parte, la solubilidad del oxgeno en el agua disminuye. El efecto neto resultante de los dos ha sido reflejado en diferentes expresiones
empricas que establecen la relacin entre la temperatura y la eficiencia del proceso. Tal es el caso de Shriver y Bowers12 entre otros.
210

Filtros Percoladores

Sin embargo, teniendo en cuenta que el efecto de la temperatura sobre los


filtros percoladores es similar al que tiene este parmetro sobre todos los procesos
biolgicos, el mismo puede expresarse mediante la ecuacin 6.6, de base tambin
emprica, pero a la que adems puede llegarse a travs de la ecuacin de Arrhenius:
kT = k20 - t 

(6.6)

- usualmente se toma como 1,035 13, 14 1,047.


Un criterio conservador es asumir el valor de 1,045.15

6.6.3 Cantidad y actividad de la capa de limo


La cantidad de la pelcula biolgica en el empaque del filtro vara con la
profundidad y con la temperatura.16,17 El espesor de la biopelcula disminuye en
la medida que se desciende a travs de la altura del empaque, ya que disminuye
la concentracin de materia orgnica biodegradable.
Se estima que para filtros de baja velocidad la cantidad de biopelcula
vara de 4,7 a 7,1 kg por cada metro cbico de empaque, mientras que para
filtros de alta carga la masa de limo que se logra est comprendida entre 3,3
y 6,5 kg m . 6
La eficiencia de remocin de DBO no est tan directamente relacionada
con la cantidad de biomasa presente como con la fraccin de esta que acta
como un oxidante activo.
Por otra parte, el espesor de la biopelcula normalmente est comprendido
entre 0,1 y 2,0 mm. Existe un efecto perjudicial en la operacin del filtro percolador
si dicho espesor es superior a los 2,0 mm, pudiendo presentarse una obstruccin
del relleno, dificultando el flujo del agua residual y la transferencia de oxgeno a
los microorganismos aerobios. Jenkins18 determin que el espesor de la biopelcula
que proporciona la mxima eficiencia est alrededor de 0,25 mm. Sin embargo,
en este sentido hay resultados muy diversos. Mientras que Bruce19 sita la profundidad aerobia activa en un espesor entre 0,05 y 0,1 mm, Kornegay y
Andrews20 sealan profundidades crticas entre 0,07 y 0,15 mm, La Motta21 las
sita entre 0,012 y 0,065 mm. Por su parte, Williams, 22 reporta buenos resultados tratando aguas residuales del procesamiento de alimentos, con espesores de
biopelcula de hasta 8 mm de espesor.
La cantidad de fraccin activa vara con la carga orgnica, la carga hidrulica y la temperatura.16 El espesor de pelcula activa depende adems de la
profundidad de la pelcula a travs de la cual penetra el oxgeno para mantener
las condiciones aerobias.
Adems del efecto que posee la carga hidrulica sobre el espesor de la
biopelcula, existe un control natural como consecuencia de la accin depredadora
211

Menndez Gutirrez, C. y J. Prez Olmo

ejercida por macroinvertebrados, larvas y gusanos que coexisten con las bacterias, protozoos y hongos, en el limo acumulado.

6.6.4. Transferencia de oxgeno a la pelcula del limo


El oxgeno necesario para la oxidacin biolgica en el seno del filtro, es
suministrado por el aire que fluye a travs del mismo.
El fenmeno de la aereacin natural, fue estudiado por Halvorson, Savage y
Piret,23 quienes llegaron a relacionar la diferencia de temperatura entre el aire y
agua, con el caudal y direccin del aire, formulando una ley lineal del siguiente tipo:
4D . 7D7/7
(6.7)
siendo:
Qa: Caudal de aire en m3md.
Ta: Temperatura del agua en el interior del lecho en C.
TL: Temperatura exterior del aire en C.
T: Constante de 2 C para lechos convencionales.
K: Constante del lecho.
Para una diferencia de temperatura 7D7/ = 6 C, el caudal de aire garantizado es aproximadamente 18 m3mh.
El esquema de la circulacin del aire puede verse en la figura 6.11, deducido
de la siguiente expresin para filtros convencionales,
Va = 0,075 (Ta-TL) - 0,15
(6.8)

siendo Va = Velocidad del aire (en mmin ).

Fig. 6.11. Circulacin del aire en un lecho.


212

Filtros Percoladores

Halvorson determin que es necesaria una corriente de aire mnima


de 0,3 mmin para que se realice una buena oxigenacin. Por tanto, las
diferencias ideales entre la temperatura del agua en el interior del filtro y la
GHO DLUH HQ HO H[WHULRU VHUiQ DTXHOODV VXSHULRUHV D   y D   & (Q HO
intervalo definido por estos valores la aereacin es deficiente y por tanto se
pueden producir procesos anaerobios indeseables.
Tal como se afirma anteriormente, la fraccin activa de la biopelcula depende en cierta medida del nivel de penetracin del oxgeno a travs del espesor
de la misma. Esta profundidad puede ser estimada por las relaciones que siguen.
Durante la operacin en estado estacionario de un filtro percolador, la velocidad de transferencia de oxgeno del lquido a la pelcula biolgica puede
obtenerse por:6

DL
A C1  C2 kgh
h

(6.9)

donde:
C1 y C2: Concentraciones de oxgeno a la profundidad 1 y 2 de la pelcula.
DL: Difusividad a travs del espesor de la pelcula (m2 h) h.
A: rea de la pelcula.
La velocidad de utilizacin de oxgeno por los microorganismos en la pelcula puede expresarse:
M

k I W A h

(6.10)

donde:
W: Gravedad especfica de la pelcula.
kI: Constante de velocidad (h).
En condiciones de estado estacionario la velocidad de consumo de oxgeno
es igual a la velocidad de transferencia, por lo tanto,

DL
A C1  C2 kI W A h
h

(6.11)

La penetracin mxima de oxgeno ocurrir cuando la concentracin de


oxgeno C2 a la profundidad h sea cero, y
h

DL C1
kI W

La mxima profundidad de la zona aerbia es de 2 a 3 mm.


213

(6.12)

Menndez Gutirrez, C. y J. Prez Olmo

6.7. DISEO DE FILTROS PERCOLADORES


En la actualidad son reconocidos muchos modelos para el diseo de filtros
percoladores. En este texto se abordar el estudio de dos de los ms reconocidos. El modelo de Eckenfelder (primer y segundo orden),24 que tiene su fundamento en los estudios de Velz 25 y Howland,8 y el modelo emprico del National
Research Council (NRC).26

6.7.1. Modelo de Eckenfelder


Con el objetivo de describir el comportamiento de los filtros percoladores, se
han desarrollado diferentes modelos matemticos. Muchos de estos modelos se
basan en la suposicin de que la velocidad de remocin de compuestos orgnicos biodegradables, siguen una cintica de primer orden, asumiendo el modelo
de flujo tubular o de pistn.
El modelo de reaccin de primer orden puede obtenerse como sigue:

dS
dt

k1 S

(6.13)

Asumiendo el comportamiento de flujo a pistn:

S2
So

e  k1 t

(6.14)

S0: DBO del afluente.


S2: DBO del efluente.
k1: constante de velocidad del proceso.
Sustituyendo para el tiempo t la ecuacin (6.5) en (6.14),

S2
So

 k1 C

H
n

(6.15)

La constante de velocidad depende de la temperatura de acuerdo con la


ecuacin 6.6:
kT

k20 t  20

La constante de velocidad tambin depende de la cantidad de


microorganismos en la pelcula. Esta cantidad es proporcional a la superficie
especfica del medio utilizado.
214

Filtros Percoladores

Sustituyendo la ecuacin (6.6) para k1 en (6.15) y combinando todas las


constantes en una sola:
S2
So

 k -t  20

20

Av H  n

(6.16)

AV: rea especfica del empaque, m2m.


El modelo descrito por la ecuacin (6.16) es conocido como modelo de
Eckenfelder.24
El exponente n vara entre 0,5 y 1,0 en dependencia del medio de
empaquetadura empleado. Benjes 27 ha fijado el valor de n en 2/3 para empaque
de piedra y de 1/2 para los medios plsticos. Para empaque plstico aleatorio,
Porter y Smith28 recomiendan el valor de 0,44 para n.
De la ecuacin 6.16 puede obtenerse una expresin para calcular el volumen necesario de empaque. As:
a) Cuando no hay recirculacin,

S0
ln S
2

kAV H

n

Q0 H

(6.17)

Q0  Qr H

(6.18)

b) Cuando hay recirculacin,

Sm
ln S
2

kAV H

Sm

n

So  R S2
1 R

(6.19),

Qr
Qo

(6.20)

Sin embargo, la mayora de las aguas residuales contienen sustratos complejos con diferentes velocidades de remocin. Esto hace necesario utilizar
una forma retardada de la ecuacin que describe la remocin global del

215

Menndez Gutirrez, C. y J. Prez Olmo

proceso. Esta ecuacin retardada puede ser obtenida del modelo de reaccin de segundo orden como sigue:

dS
dt

k2 S 2

(6.21)

Asumiendo el comportamiento de flujo a pistn:

S2
So

1
1  k2 S o t

(6.22)

Utilizando el mismo procedimiento que para el modelo de primer orden:


S2
S0

1
1  So k 20 t  20 Av H  n

(6.23)

El exponente n vara en el intervalo comprendido entre 0,85 y 1,25 en dependencia del medio de empaquetadura empleado.
Para el modelo de segundo orden el volumen de la empaque se calcula
segn:
a) Sin recirculacin,

n
So  S2

2
Q H
S kA H
o
o V

(6.24)

b) Con recirculacin,

n
Sm  S2

2
Q Q H
S kA H
r
0
0. V

(6.25)

Ejemplo 6.1
Se desea tratar 3217 m3d de un agua residual industrial con una DBO de
850 mgL hasta obtener un efluente de 280 mgL. Se emplear una torre
de 6 m de altura cuyo empaque tiene un rea especfica de 100 m2m.
216

Filtros Percoladores

Determine la car ga hidrulica, r ea de columna y r elacin de


recirculacin. La mxima DBO que debe existir a la entrada del filtro
percolador es de 570 mgL .
Considere que se cumple el modelo de primer orden y,
k = 0,008 md
t = 28 qC

n = 0,5

Clculo de R,

S  R S
o
1 R

850  R 280
1 R

570

R = 0,96 | 1,0
Clculo de la carga hidrulica, Q,

S
Sm

exp  k Av H  n

280
570

exp  0,008100 H  0 ,5 , n = 44 m3md

rea,

Q  R Q
o
o

3217  1.3217
146,23
44

A = 146,23 m2
Volumen de empaque, V = HA = 877,38 m3
Carga orgnica,
Bv

Sm Qo  Qr 103
V

4 ,18 kgmd

Los valores de carga hidrulica y orgnica chequean para el tipo de filtro


deseado.
Ejemplo 6.2
Determine el volumen de relleno necesario para tratar 1200 m3d de un
agua residual cuya DBO 5 es de 200 gm si se desea obtener 90 % de
remocin.
Considere el modelo de primer orden, empaque plstico,

217

Menndez Gutirrez, C. y J. Prez Olmo

Av = 150 m2m

t = 26 qC
H = 3,0 m
k20 qC = 0,071 md

n = 0,5

Asumiendo que kt k 20 1,047 t  20


Se tendr que k26qC = 0,01
a) Sin recirculacin,

S0
ln S

kAV H

n

Q0 H

200

ln

20

0 ,01 150 3,0

 0 ,5

1200 3,0

V = 940 m3
Si H = 3 m,
entonces el dimetro de filtro a utilizar ser: 20 m
Comprobando los valores de carga hidrulica y carga orgnica:
Carga hidrulica:

Q0
A

Carga orgnica: BV

S 0 Q0
V

1200
313

3,8 m3md

200 1200 103


940

BV = 0,25 kgmd
Aun cuando el valor de la carga hidrulica est dentro del intervalo recomendado para filtros sin recirculacin, la carga orgnica tiene un valor algo
superior al recomendado. No obstante, podra considerarse que no es necesario
el uso de la recirculacin a menos que se desee una mayor flexibilidad en la
operacin. Hoy da lo ms comn, aun en casos como este, es concebir la
recirculacin para seguridad de la operacin y obtener una mayor flexibilidad de
trabajo.
Adems de la ventaja que ofrece la recirculacin en cuanto a evitar tupiciones
en el filtro, puede citarse la de la posibilidad de utilizar mayores alturas de filtro.
Esta ventaja puede hacerse ms evidente en la medida que el material utilizado
en el empaque sea ms ligero.

218

Filtros Percoladores

b) Con recirculacin,

n

Sm
ln S

kAV H

Q0  Qr H

Sm = 140 mgL

Si R = 0,5

140

ln

20

0,01150 3,0

0 ,5

1200  600 3,0

V = 1009 m3
Si H = 3,0 m, entonces el dimetro de filtro a utilizar ser: 21 m
Comprobando los valores de carga hidrulica y carga orgnica:
Carga hidrulica:

Q0  Qr
A

Carga orgnica: Bv

Sm Q0  Qr 140 1800 103


1009
V

1800
336

5,3 m3md

BV = 0,25 kgmd
Tal como se aprecia, los valores de carga hidrulica y carga orgnica obtenidos coinciden ahora con los recomendados para un filtro de velocidad media.
Si se incrementara la altura de empaquetadura, se reduce el rea y como
consecuencia aumenta la carga hidrulica sobre el filtro.

6.7.1.1. Relacin volumen - altura


La relacin entre la altura de la empaquetadura en el filtro y el volumen que
ocupa esta es funcin del valor de n. Figura 6.12. En la tabla 6.1 se ofrece el
factor por el que se afecta el volumen de la empaquetadura al aumentar la
profundidad para distintos valores de n.

219

Menndez Gutirrez, C. y J. Prez Olmo

Tabla 6.1. Factor por el que se afecta el volumen del filtro al aumentar la
altura H, para 5 valores de n
ALTURA

n
0,4
0,5
0,7
0,8
1

1,5 H
0,54 V
0,67 V
0,84 V
0,91 V
V

2H
0,35 V
0,5 V
0,75 V
0,83 V
V

3H
0,19 V
0,33 V
0,62 V
O,75 V
V

Fig. 6.12. Variacin del volumen de empaque con la profundidad para


distintos valores de n.

6.7.1.2. Efecto de la recirculacin


Es importante destacar el efecto de la recirculacin sobre la eficiencia de
depuracin as como sobre las cargas hidrulica y orgnica del filtro. Estos efectos
pueden apreciarse en las figuras 6.13 y 6.14.
La eficiencia del filtro, disminuye al aumentar la recirculacin, cuando esta
es medida con respecto a la DBO de entrada (Sm) segn la figura 6.9. Sin
embargo, cuando se calcula sobre la base de S0, se mantiene constante, independientemente del flujo de recirculacin. Figura 6.13.
Por otro lado, tal como se observa en la figura 6.14, el efecto de la
recirculacin sobre la carga hidrulica es ms notable que sobre la carga
orgnica. Esto se debe a que en esta ltima el incremento del flujo de entrada es compensado en parte por la disminucin de la concentracin que experimenta la DBO a la entrada.
220

Filtros Percoladores

Fig. 6.13. Efecto de la recirculacin sobre la eficiencia. 1. Calculada a


partir de S0. 2. Calculada a partir de Sm.
En la tabla 6.2 se destaca el efecto de la recirculacin para un caso particular.

Fig. 6.14. Efecto de la recirculacin sobre las cargas hidrulica y orgnica.


Tabla 6.2. Eficiencia, carga hidrulica y orgnica para distintos valores de R

Q0 = 4500 m3d-1
H=2m
S0 = 300 mgL-1
k = 0,02
R
Ef/S0
Ef/Sm
0
0,5
1,0
1,5
2,0
2,5

85,0
84,7
84,9
85,3
85,6
86,0

85,0
78,0
74,0
69,8
66,5
63,8
221

AV = 72 m2m-3
V = 3905 m3
BV
X
kgm3d-1
m3m-2d-1
0,34
2,30
0,37
3,46
0,40
4,61
0,42
5,76
0,44
6,92
0,47
8,07

Menndez Gutirrez, C. y J. Prez Olmo

6.7.2. Modelo del NRC


Ms que un modelo que pretenda explicar la respuesta de un proceso
ante la incidencia de diferentes factores, el mtodo del NRC trata de un
sistema de clculo.
Este mtodo es el resultado de un estudio realizado en la dcada de los
aos cuarenta del siglo pasado en bases militares en Estados Unidos de
Norteamrica. En el mtodo son obviados algunos parmetros como el rea
especfica de la empaquetadura y la influencia de la carga hidrulica en la
eficiencia de la depuracin.
De acuerdo con el NRC, el volumen de la empaquetadura de un filtro
percolador puede calcularse segn,
2

0,443 E W

100  E F

(6.26)

donde:
V: volumen m3.
W: carga de DBO kgd.
E: eficiencia de depuracin %.
F: factor asociado a la recirculacin,
F

1 R
R

1 
10

(6.27); R

Qr
Q0

Ejemplo 6.3
Estime el volumen de empaque necesario para tratar 2500 m3 d de un
agua residual urbana cuya concentracin de DBO5 es 250 mgL si se requiere un efluente con una DBO de 25 mgL  . Considere una altura de empaque de 2,0 m.
E = 80 %
Asumiendo inicialmente R = 0,
F=1
W = 250 2500 10 = 625 kgm3d
2

0,443 E W

100  E F

222

Filtros Percoladores
2

0,443 80 625
1962,5 m3


100 80 1

Si H = 2,0 m, entonces el rea ser,


A = 981,25 m2
Verificando los valores de carga hidrulica y carga orgnica:
2500
981,25

BV

2 ,55 m3m d

625
1962,5

0,32 kgm3d

La carga hidrulica est dentro del intervalo que corresponde a los sistemas sin recirculacin. Sin embargo, el valor de la carga orgnica es superior
al valor mximo de 0,2 kgm3 d recomendado para esta variante de R = 0.
Asumiendo R = 1,
F = 1,65
2

0,443 80 625
1189 m3


,
100
80
1
65

Si H = 2,0 m, entonces el rea ser,


A = 594,5 m2
Verificando los valores de carga hidrulica y carga orgnica:

BV

5000
594,5

625
1189

8,41 m3m d

0,52 kgm3d

Ahora tanto la carga hidrulica como la orgnica se corresponden con los


valores del intervalo para filtros de velocidad media. Por lo tanto, el volumen a
utilizar de empaque es de 1189 m3 y R = 1.

223

Menndez Gutirrez, C. y J. Prez Olmo

6.8. CLCULO DE LAS CONSTANTES DEL MODELO


DE PRIMER ORDEN
6.8.1. Determinacin de n y k a temperatura constante
La mayor dificultad en el uso del modelo de primer orden para el diseo de
filtros percoladores radica en la necesidad de conocer los valores de los
parmetros n y k. En el caso de n, esta informacin puede obtenerse de las
firmas que ofertan el material de empaque. El valor de k puede conocerse de
diferentes fuentes bibliogrficas.
Por otro lado, los valores de k y n pueden ser calculados en pruebas realizadas en plantas piloto o semipiloto.
En la figura 6.15 se presenta el diagrama de una instalacin tipo semipiloto
de filtro percolador.

Fig. 6.15. Filtro percolador semipiloto.


Esta planta puede operarse tanto con empaque de piedra como de material
plstico. Este ltimo permite mayor profundidad de empaque.
El procedimiento consiste en realizar pruebas a varios valores de carga
hidrulica y diferentes alturas de empaque. Para ello el filtro debe tener
previsto tomar muestras a diferentes profundidades. Generalmente se utili224

Filtros Percoladores

zan entre 3 y 5 valores diferentes de carga hidrulica, en un intervalo que


depender del tipo de empaque disponible, y 4 alturas o profundidad de empaque.
Una de las mayores dificultades para la realizacin de estas pruebas se
encuentra en la formacin del limo. Esta se puede lograr en un trmino de 3 a 4
semanas. Debe conocerse de antemano el rea especfica (AV) del material que
se utilizar como empaque.
Procedimiento
1. Para la puesta en marcha de la instalacin, con la salida del filtro cerrada, se
alimenta una vez al da el agua residual que se desea tratar, y se recircula
durante 24 horas.
2. Transcurridas 24 horas se extrae el agua residual y se alimenta una nueva carga. Esta operacin se repite de 2 a 3 semanas. Se sugiere que
la carga hidrulica sobre el filtro en esta etapa no debe ser mayor
de 4 m3 m d . Si en ese periodo se logra la formacin de limo sobre el
empaque, se debe cerrar la recirculacin e iniciar la operacin normal del
filtro a la menor carga hidrulica de las que se seleccione para trabajar
en las siguientes etapas.
3. Determinar los valores de carga hidrulica de trabajo as como las alturas a
las que se tomarn las muestras.
4. Para cada valor de carga hidrulica se tomarn muestras a las diferentes
alturas.
5. Los anlisis de las muestras se expresan en por ciento de DBO remanente
(S/S0 100).
Para cada valor de carga hidrulica a las que se realizan las experiencias, y
teniendo en cuenta la ecuacin:
S
S
0

 kA H
V
e

n

(6.27)

se representa en papel semilogartmico el % de DBO remanente en funcin de


la profundidad, H:
S
ln
S
0

 kAV H n

 kA
V

n
H

(6.28)

De esta manera puede obtenerse una familia de rectas, dependiendo de la


cantidad de cargas hidrulicas seleccionadas, y cuya pendiente en cada caso
ser, en valor absoluto, kAVXn. Figura 6.16 del ejemplo 6.4.
225

Menndez Gutirrez, C. y J. Prez Olmo

7. En un grfico doble logartmico (log-log) se representan los valores absolutos


de las pendientes de las rectas obtenidas en el paso 6, (kAVX n), en funcin
de la carga hidrulica que le dio origen:
log pendiente

log kAV  n log

(6.29)

De la pendiente de la recta correspondiente se obtiene el valor de n.


8. El valor de k se obtiene de la ordenada, considerando como valor de origen,
el valor de abscisa correspondiente a la menor carga hidrulica. Figura 6.17.
El valor de la ordenada, en el punto tomado como origen es igual a kAVX n.
Del valor de n as como del valor de la menor carga hidrulica de las utilizadas, puede obtenerse la constante k.
Ejemplo 6.4
Para determinar los valores de los parmetros n y k se opera una instalacin
semipiloto con un material sinttico de empaque cuya rea especfica es
de 200 m2m. Las pruebas se realizaron a 5 cargas hidrulicas diferentes, tomando muestras a 4 profundidades de filtro. Las mediciones de DBO en cada
caso arrojaron los resultados que aparecen en la tabla 6.1, reportados como
S/S0. Calcule n y k.
Tabla 6.3. Valores de S/S0 100 a diferentes cargas hidrulicas y alturas
Ejemplo 6.4.

Altura m
1,5
3,0
4,5
6,0

15
70
49
33
23

Carga hidrulica m3m-2d-1


27
36
46
64
62
58
41
38
33
26,5
23
19
17
14
11

70
48
23
11

1. Para cada carga hidrulica se grafica el % DBO remanente en funcin de la


profundidad,
S
ln
S
0

 kA
V

n
H

obteniendo la familia de rectas de la figura 6.16. Al calcular la pendiente


(-kAVX n) de cada una de las rectas,

226

Filtros Percoladores

Tabla 6.3.a. Valor de la pendiente a diferentes cargas hidrulicas


X m3m-2d-1
Pendiente (kAVX -n)

15
-0,490

27
-0,370

36
-0,330

46
-0,295

70
-0,247

S
100
S0

Fig. 6.16. % de DBO remanente en funcin de la profundidad.


2. En log-log se grafica el valor absoluto de las pendientes en funcin de la
carga hidrulica. Figura 6.17.
log pendiente

log kAV  n log

La pendiente de esta recta es la constante n = 0,44


3. Para obtener el valor de k: se toma la ordenada que corresponde con X = 15
m3md, que al ser la menor de las cargas hidrulicas, representa el origen,
kAVX n = 0,49
como: X n = 15  = 0,30 y AV = 200, entonces,
k = 0,008 md
227

Menndez Gutirrez, C. y J. Prez Olmo

Fig. 6.17. Representacin grfica de kAVH en funcin de X .

6.8.2. Clculo de la dependencia de k con la temperatura


En la ecuacin 6.16 qued expresada la relacin que existe entre el valor de
k a una temperatura de referencia, 20 qC, y su valor a cualquier otra temperatura. Esta relacin permite que pueda conocerse el valor de las constantes de un
filtro percolador a cualquier temperatura, y a partir de l, determinarla a una
temperatura de inters.
En este epgrafe se describe, mediante el ejemplo 6.5 un procedimiento para
calcular, experimentalmente el valor del parmetro - de la ecuacin 6.16. En el
propio ejemplo se aplica una variante del procedimiento descrito en el epgrafe
6.8.1 para el clculo de k y n cuando se dispone de informacin de una sola
profundidad de empaque.
Ejemplo 6.5
Los datos de la tabla 6.4 fueron obtenidos para tres temperaturas diferentes,
20 qC, 25 qC, y 30 qC.
Determine los parmetros n y kt, para el modelo de primer orden, as como
una expresin que permita conocer la influencia de la temperatura sobre la eficiencia de depuracin. Profundidad del filtro 1,83 m; AV = 72 m3md.
R=0
228

Filtros Percoladores

La ecuacin (6.16) puede escribirse:

S
ln ln 0
S2

ln kt AV H  n ln

(6.30)

S0
graficando ln ln S en funcin de lnQ ha de obtenerse una lnea recta cuya
2

pendiente es el parmetro n y su intercepto con el eje de ordenadas ktAVH.


En la figura 6.18 se muestra, a modo de ejemplo, la recta correspondiente a
la temperatura de 20 qC. Para las restantes 2 temperaturas se procede de manera similar.
Tabla 6.4. Modelo cintico de primer orden
No. Temp qC
1
2
3
4
5
6
7
8
9
10
11
12
13
14
15
16
17
18
19
20

20

S0
(mgL-1)
213

25

197

30

212

S2
(mgL-1)
44
64
87
102
26
55
61
72
96
95
107
112
22
42
49
64
88
110
114
129

X
(m3m-2d-1)
2,176
4,901
6,529
8,705
2,176
4,901
6,529
8,705
10,881
13,602
16,322
21,762
2,176
4,901
6,529
8,705
10,881
13,602
16,322
21,762

Ln X
0,777429
1,589386
1,876321
2,163933
0,777429
1,589386
1,876321
2,163933
2,387035
2,610246
2,792500
3,080182
0,777429
1,589386
1,876321
2,163933
2,387035
2,610246
2,792500
3,080182

S0
Cuando se grafica ln ln vs ln X a cada temperatura, se obtienen 3
S2
juegos de valores de n y kt. Tabla 6.5.
229

Menndez Gutirrez, C. y J. Prez Olmo

Tabla 6.5. Procesando los datos

Temperatura qC
20
25
30

n
0,537
0,579
0,701

kt
0,0189
0,0246
0,0351

Valor promedio de n = 0,6


Con el valor promedio de n y aplicando la ecuacin 6.30, se recalculan los
valores de kt para cada uno de los puntos experimentales.

Fig. 6.18. Ejemplo del clculo de n para la temperatura de 20 C.


Procesando los datos de la Tabla 6.4, para el modelo cintico de primer
orden, se obtienen las tablas 6.6 a 6.8.
Tabla 6.6. Procesando datos
Temp.
20 qC
n = 0,6
1
2
3
4

Valor de kt obtenido
considerando n = 0,6
y aplicando la ec. 6.30
0,0189655
0,0236824
0,0209484
0,0204717
0,0210170

S
ln ln 0
S2

0,4493926
0,1843271
-0,1105025
-0,3060914
promedio

230

Filtros Percoladores

Tabla 6.7. Procesando los datos


Temp.
25 qC
n = 0,6
5
6
7
8
9
10
11
12

Valor de kt obtenido
considerando n = 0,6
y aplicando la ec. 6.30
0,0245044
0,0251293
0,0274279
0,0279839
0,0228488
0,0265038
0,0247442
0,0272058
0,0257240

S
ln ln 0
S2

0,7056226
0,2436287
0,1589931
0,0065163
- 0,3300949
- 0,3156333
- 0,4936819
- 0,5714521
promedio

Tabla 6.8. Procesando los datos


Temp.
30 qC
n = 0,6
13
14
15
16
17
18
19
20

Valor de kt obtenido
considerando n = 0,6
y aplicando la ec. 6.30
0,0274138
0,0318858
0,0342697
0,0332994
0,0279469
0,0238429
0,0251501
0,0239331
0,0284677

S
ln ln 0
S2

0,8178148
0,4817572
0,3816955
0,1804057
- 0,1286866
- 0,4214331
- 0,4774105
- 0,6996203
Promedio

De esa manera se obtiene que para el sistema bajo estudio:


n = 0,6 y los valores de kt obtenidos son:
0,0210170
a 20 qC
0,0257240
a 25 qC
0,0284677
a 30 qC
Para calcular la influencia de la temperatura, se hace uso de una ecuacin
similar a la 4.33,

ln kt

a p

231

1
T

Menndez Gutirrez, C. y J. Prez Olmo

Tabla 6.9. Procesamiento de datos


Temperatura
Kelvin
293
298
303

1 /T

kt

ln kt

0,003412969
0,003355705
0,003300330

0,021017
0,025724
0,028467

- 3,8624236
- 3,6603309
- 3,5589852

Aplicando estos datos a la ecuacin anterior se obtiene,

ln kt

 12,4497  2693,9

1
T

para 2 temperaturas K diferentes, por ejemplo, 298 y T,


k
T  298
ln T 2693,9
298T
k298

298 T | (298)2 = 88 804, entonces,


k
ln T 0,030 T  298
k 298
kT

k 298 1,030t  20

Notas bibliogrficas
1 PEDERSEN, K.:%LRILOPGHYHORSPHQWRQVWDLQOHVVVWHHODQG39&VXUIDFHV
in drinking water, Water Res., vol. 24, no. 1, 1990.
2 LAZAROBA, V. AND MANEM:%LRILOPFKDUDFWHUL]DWLRQDQGDFWLYLW\DQDO\VLVLQ
ZDWHUDQGZDVWHZDWHUWUHDWPHQWWater Res., vol. 29, no. 10, 1995.
3 ECKENFELDER, W.W. AND'-2&ONNOR: Biological waste Treatment, Pergamon Press Ltd., London, 1964.
4 VASEL, J. AND P. SCHROBITGEN:2[\JHQWUDQVIHULQWULFNOLQJILOWHUVWater
Res., vol. 25, no. 1, 1991.
5 FRUHEN, C.6SDWLDOGLVWULEXWLRQRIVSHFLHVLQELRILOPWater Sci. Technol., vol. 23, no. 8, 1991.
6 CHUDOBA, J., C. MENNDEZ Y J. PREZ: Fundamentos Tericos de Algunos
procesos para la depuracin de Aguas Residuales, Edit. ISPJAE,
Cuba, 1986.
232

Filtros Percoladores

7 IMHOFF , K. W.J. MULLER, AND T HISTLETHWAYTE : Disposal of Sewage and


other Waterborne Wastes, Edit. Butterworths, 1971.
8 HOWLAND, W.E.)ORZRYHUSRURXVPHGLDDVLQWULFNOLQJILOWHUProc. 12th
Ann. Ind. Waste Conf. Purdue Univ., 1958.
9 BLOODGOOD, D.E., G.H. TELETZKE, AND F.G. POHLAND:)XQGDPHQWDOK\GUDXOLF
SULQFLSOHVRIWULFNOLQJILOWHUVSewage and Ind. Wastes, vol. 31, no. 3, 1959.
10 MEHTA, D.S., H.H. DAVIES, AND R.P. KINGSBURY: Oxygen theory in biological treatment plant design, J. San. Eng. Div. ASCE, 98 (SA 3),
1972.
11 BRUCE, A.M. AND J.C. M ERKENS:)XUWKHUVWXGLHVRISDUWLDOWUHDWPHQWRI
VHZDJH E\ KLJKUDWH ELRORJLFDO ILOWUDWLRQ Water Poll. Control, vol.
72, no. 5, 1973.
12 SHRIVER , L.E. AND D.M. BOWERS2perational practices to upgrade filter
SODQW SHUIRUPDQFH -RXU Water Poll. Contro Fed., vol. 47, no. 11,
1975.
13 SHULZE , K.L/RDGDQGHIILFLHQF\RIWULFNOLQJILOWHUV-RXUWater Poll.
Control Fed., vol. 32, no. 3, 1960.
14 LAMB , R. A6XJJHVWHGIRUPXODIRUWKHSURFHVVRIELRORJLFDOILOWUDWLRQ
Water Poll. Control, vol. 69, no. 2, 1970.
15 WEF AND ASCE 'HVLJQ RI PXQLFLSDO ZDVWHZDWHU WUHDWPHQW SODQWV
vol. I. Water Environment Federation, Alexandria, 1992.
16 HAWKS , H.A.: The Ecology of Waste Treatment, Ed. Pergamon Press,
London, 1963.
17 CHANG, G. AND J. HUANG:(IIHFWRIELRILOPWKLFNQHVVGLVWULEXWLRQRQVXEV
WUDWHLQKLELWHGNLQHWLFWater Res., vol. 28, no. 8, 1994.
18 JENKINS, D. 6HZDJH WUHDWPHQW &DS  HQ Biochemistry of Industrial Microorganisms, Eds. Rainbow, C and A.H. Rose, Academic
Press, 1963.
19 BRUCE , A.M.:3HUFRODWLQJILOWHUVProcess Biochem., vol. 4, no. 4, 1963.
20 KORNEGAY, B.H. AND J.F. ANDREWS:.LQHWLFVRIIL[HGILOPELRORJLFDOUHDF
WRUV-RXUWater Poll. Control Fed., vol. 40, no. 2, 1968.
21 LAMOTTA, E.J.:,QWHUQDOGLIIXVVLRQDQGUHDFWLRQLQELRORJLFDOILOPVEnv.
Sci. Technol., vol. 10, no. 8, 1976.
22 WILLIAMS, I.L.::DWHU3ROOControl, vol. 78, no. 2, 1979.
23 HALVORSON, H.O., G.M.S AVAGE AND E.L. P IRET:6RPHIXQGDPHQWDOIDF
WRUVFRQFHUQHGLQWKHRSHUDWLRQRIWULFNOLQJILOWHUVSewage Works J.,
vol. 8, no. 6, 1936.
233

Menndez Gutirrez, C. y J. Prez Olmo

24 ECKENFELDER, W.W.: Industrial Water Pollution Control, Mc Graw Hill,


1989.
25 VELZ, C.J.: $ EDVLF ODZ RI SHUIRUPDQFH RI ELRORJLFDO EHGV Sewage
Works J., vol. 20, no. 4, 1948.
1DWLRQDO5HVHDUFK&RXQFLO7ULFNOLQJ)LOWHUV 7UHDWPHQWDW0LOLWDU\,QV
WDOODWLRQV Sewage Works J., vol. 18, no. 5, 1948.
27 BENJES , H.H.: Handbook of Biological Wastewater Treatment, Garland STPM Press, New York, 1980.
28 PORTER, E.K. AND E. SMITH:3ODVWLFPHGLDELRORJLFDOILOWHUVWater Poll.
Control, vol. 78, no. 2, 1979.

234

Lagunas de Estabilizacin

CAPTULO 7

LAGUNAS DE ESTABILIZACIN

7.1. CARACTERSTICAS GENERALES


El empleo de lagunas de estabilizacin para el tratamiento de aguas residuales
que contengan compuestos orgnicos biodegradables est muy difundido.
Las lagunas son estructuras simples de fcil operacin y mantenimiento que
se basan en el proceso de autopurificacin. Generalmente estn constituidas
por embalses naturales o artificiales, en tierra, expuestos al aire y al sol, por lo
que las condiciones climticas influyen significativamente en el funcionamiento
de este dispositivo de tratamiento. Por esta razn el diseo de las lagunas es
posiblemente, de todos los procesos de tratamiento biolgico, el menos definido.
De acuerdo a la naturaleza de la actividad biolgica que tiene lugar en la
misma, y por tanto al metabolismo que prevalece durante su funcionamiento, las
lagunas de estabilizacin se clasifican en:
Aerobias.
Anaerobias.
Facultativas.

7.2. LAGUNAS AEROBIAS


Bajo esta denominacin se incluyen aquellas lagunas en las que los compuestos biodegradables, sean estos suspendidos o disueltos, son estabilizados
por la accin de microorganismos aerobios. El oxgeno es suministrado por la
accin fotosinttica de las algas y por el que se difunde del aire. Debido a que la
luz solar es esencial para la produccin de oxgeno mediante algas, la profundidad de estas lagunas est limitada de acuerdo con la penetracin de la luz, y
normalmente es menor de 50 cm. En el caso de aguas residuales domsticas
tpicas y algunos residuales industriales, se producen aproximadamente
entre 0,5 y 0,6 kg de biomasa por kilogramo de DBO removido.
Aunque estas lagunas admiten cargas de DBO relativamente altas, su poca
profundidad hace que se requieran grandes reas para su construccin y hoy da
tienen uso limitado.
235

Menndez Gutirrez, C. y J. Prez Olmo

Las principales reacciones que pueden llevarse a cabo durante la estabilizacin en este tipo de lagunas son:
degradacin aerobia
6 C 6H12O6  16 O 2  4NH 3 o 4 C5 H 7 NO 2  16CO 2  28 H 2O

(7.1)

a su vez, las algas sintetizan materia orgnica que incorporan a su propio


protoplasma.
fotosntesis
NH 3  8 CO 2  4,5 H 2 O o C8 H12 O3 N  8,75 O2

(7.2)

nitrificacin


N  org o NH3 o NO 2 o NO3

(7.3)

7.3. LAGUNAS ANAEROBIAS


Las lagunas anaerobias se construyen preferentemente para reducir la carga orgnica sedimentable.
En las lagunas de este tipo, la materia orgnica es estabilizada mediante un
mecanismo similar al que existe en los tanques de digestin anaerobios (formacin de cidos orgnicos y de metano).
En las condiciones de climas tropicales la carga de estas lagunas puede
variar entre 60 y 300 g DBO m d. La profundidad est limitada por elementos prcticos: nivel fretico, tipo de suelo y facilidades para la limpieza eventual
de los lodos o fangos que se depositan en el fondo con el transcurso del tiempo.
Las ecuaciones simplificadas que representan las transformaciones que tienen lugar en una laguna anaerobia son:

CH 2O X

o X HCH 3COO

(7.4)

HCH 3COO o CH 4  CO 2

(7.5)

N  org o NH 4

(7.6)

7.4. LAGUNAS FACULTATIVAS


En las lagunas facultativas se distinguen dos zonas de trabajo bien diferenciadas: una regin aerobia en la superficie y cercana a esta y una regin
anaerobia en el fondo. Entre ambas existe una zona, no muy bien delimitada,
facultativa. Figura 7.1.
236

Lagunas de Estabilizacin

Una laguna de estabilizacin facultativa es una estructura simple de relativa


poca profundidad (1 a 3 m) y con periodos de retencin entre 1 y 40 das.
Las lagunas facultativas se caracterizan porque:
ocurre la autodepuracin o estabilizacin natural,
la DBO es el parmetro que se utiliza para evaluar las condiciones de la
laguna,
para cargas orgnicas (CO) bajas 5-20g DBO 5 md y temperaturas
entre 20 y 30 C, el estrato superior se cubre de algas microscpicas
(clorellas y euglenas), llegando el agua a sobresaturarse de oxgeno y
la penetracin de la luz es limitada (5 a 15 cm) y el estrato inferior es
anaerobio.

7.1.b
Fig. 7.1. a) Esquema de los mecanismos responsables de la depuracin
en lagunas facultativas. b) Simbiosis alga-bacteria en las lagunas.
237

Menndez Gutirrez, C. y J. Prez Olmo

No existe un lmite bien definido de carga para garantizar si una laguna es


facultativa o anaerobia. Para temperaturas entre 15 y 30 C hay una zona de
 
transicin entre 30 y 60 gm d .
En Cuba hay centrales azucareros con refinera que en pocas en que no
 
hay zafra la carga de las lagunas es de 45 gm d y son anaerobias.1

7.4.1. Factores que afectan el proceso depurador de las lagunas


pH,
luz solar,
temperatura,
nutrientes y sustancias txicas y
La influencia de la luz solar est relacionada con la fotosntesis.

pH
Producto de la respiracin bacteriana se libera CO2, que tiende a bajar el
pH. Por otro lado, la accin fotosinttica provoca consumo del CO2 con el consiguiente aumento del pH. Por tanto el pH esta sujeto a variaciones horarias que
se relacionan con la energa luminosa incidente.2

Radiacin Solar
Segn Thirumurthi, para un buen funcionamiento de las lagunas facultativas
se requiere una radiacin solar mnima de 4 langleysd (4 cal cmd) por cada
(g DBQ aplicadom) (luz visible).3 De acuerdo con Eckenfelder,4 este valor es
de 9 langleys.d. Ildeu Duarte, citado por Senz, reporta el valor de 6 langleysd
1
de radiacin visible como valor de radiacin mnima.5 En Cuba, los meses de
menor radiacin son diciembre y enero, con un valor mnimo calculado como
promedio de 250 cal cmd.6 Esta es una de las razones de las excelentes
condiciones de los pases tropicales para el funcionamiento de las lagunas facultativas.
No toda la energa solar que llega a la laguna es utilizada. Despus de
un cierto valor de intensidad de luz donde las algas se saturan, el resto no
es utilizado excepto para elevar y mantener la temperatura. As, por ejemplo, el valor de saturacin estimado para un cierto tipo de clorella 5 es
de 60 cal cm d .
Debido a que la intensidad de la luz vara a lo largo del da y de acuerdo a la
poca del ao, la velocidad de crecimiento de las algas vara tambin de la
misma forma. Este fenmeno da lugar a dos efectos fundamentales: el oxgeno
238

Lagunas de Estabilizacin

disuelto y el pH del agua presentan valores mnimos en las ltimas horas de la


noche, y aumentan durante las horas de luz solar hasta alcanzar valores mximos a media tarde. A partir de ese momento, los valores decrecen al arribar
nuevamente a la noche.2 En la figura 7.2 se refleja esta variacin cclica del
oxgeno y el pH.

Fig. 7.2. Variacin diaria de oxgeno y pH.


Para lagunas aerobias, la produccin de O2 por algas, est relacionada a la
eficiencia de conversin de la luz y de su intensidad. As,
O2 = 0.28 F IL

(7.7)

O : produccin de O .
2
2
F: eficiencia de conversin de la luz (generalmente se toma como 4%).
, L: intensidad de luz (cal cmd ) (langley).
Para estas lagunas la carga orgnica permisible del afluente en base a DBO,
(gmd) puede relacionarse con la intensidad de la luz. La carga hidrulica se
OLPLWDDFPG . La carga orgnica superficial (CO) que se recomienda
debe ser menor de 25 g md . La mxima produccin de algas se obtiene
cuando la profundidad no rebasa el intervalo comprendido entre 15 y 30 cm. Si
lo que se busca es produccin de O2, las profundidades deseables no deben ser
mayores de 1 a 1.3 m.
La produccin de algas se estima:
Ac=0.017 F. IL
(7.8)
 
Ac-produccin de algas (g m d )

239

Menndez Gutirrez, C. y J. Prez Olmo

Temperatura
Como ocurre en todos los procesos biolgicos, la temperatura ejerce una
influencia marcada sobre las lagunas. En general, y para los intervalos de temperatura normales en las lagunas, se puede decir que la velocidad de la depuracin aumenta con la temperatura, en especial en lo que concierne a la actividad
de las bacterias.
Sin embargo, y en lo que respecta a las algas, se han detectado disminuciones importantes en su actividad fotosinttica a temperaturas superiores a 30 C,
relacionadas con la estimulacin del crecimiento de algas menos productivas
(verdiazules) que las algas verdes (clorofceas) a las que sustituyen. Puesto que
este fenmeno coincide con una gran actividad de las bacterias, y por tanto,
grandes consumos de oxgeno, puede ampliarse la zona anaerobia en las lagunas
facultativas durante la poca de calor, y muy especialmente si el calentamiento
se produce de forma brusca. Normalmente esta situacin es solo transitoria y
las lagunas vuelven a funcionar correctamente en la medida que la temperatura
disminuye.

7.5. RGIMEN DE FLUJO EN LAS LAGUNAS


Uno de los aspectos ms controvertidos en las lagunas lo constituye el rgimen de flujo que prevalece en las mismas. Hay varias hiptesis:
a) mezcla completa,
b) flujo a pistn y
c) flujo disperso.
Las dos primeras hiptesis describen condiciones de flujo extremas e ideales. La tercera, una intermedia entre ambas, se ajusta a las condiciones de flujo
que realmente prevalecen en las lagunas. No obstante, condiciones muy cercanas al comportamiento de flujo de pistn o tubular se han encontrado en lagunas
con una relacin largo ancho mayor de 3:1.
Ha sido demostrado que an cuando se sabe que el rgimen de flujo que
prevalece en las lagunas es el de flujo disperso, no se comete mucho error, a los
efectos de diseo, asumiendo un modelo de flujo de mezcla completa.7

7.5.1. Lagunas en serie y paralelo


El mayor argumento para la construccin de una laguna grande est relacionada con el beneficio que ocasionan los vientos. Sin embargo, hoy se reconocen
las ventajas de las bateras de lagunas, sean estas en serie o en paralelo.
240

Lagunas de Estabilizacin

En serie:
mejora la calidad bacteriolgica del agua,
se obtiene mayor economa del rea.
En paralelo:
no mejora la calidad del efluente pero trae ventajas en su operacin,
mejor para la puesta en operacin de las lagunas al ser estas ms pequeas, ya que se llenan ms rpido y
mayor facilidad para la limpieza.

7.6. DISEO DE LAGUNAS DE ESTABILIZACIN


Numerosas experiencias realizadas durante ms de 40 aos han estimulado
los intentos para desarrollar modelos matemticos para el diseo de lagunas de
estabilizacin. Esto obedece al hecho ya expresado, de que el diseo de este
tipo de dispositivo de tratamiento es, de todos los procesos de tratamiento biolgico, el menos definido.
Entre los diferentes mtodos desarrollados para el diseo de lagunas pueden
citarse los empricos por carga superficial reportados por McGarry y Pescod,8, 9 el
de Arceivala (WHO),10 y el emprico volumtrico de Herman y Gloyna.11 Otros
mtodos son el modelo racional de Marais12 y el de flujo disperso de Thirumurthi.2
Por su parte, Polprasert y Batharai 13 realizaron un aporte importante
correlacionando el nmero de dispersin con las formas geomtricas de la laguna y propiedades del fluido, para facilitar el empleo de este ltimo modelo.
Ninguno de los mtodos de diseo de lagunas tiene una aceptacin universal. Existe mucha literatura en ese sentido, muchas veces contradictoria, que
puede inclusive llegar a confundir.
Para ilustrar la evolucin de los criterios de diseo para lagunas facultativas,
sern considerados, brevemente los temas desarrollados por Hermann y Gloyna,
emprico volumtrico, el de McGarry y Pescod, emprico superficial y por Marais
y Shaw, racional de primer orden.

7.6.1. Criterio americano


Dentro de ciertos lmites, la velocidad de las reacciones de degradacin
aumenta con el aumento de la temperatura. Si to es el tiempo de degradacin a
la temperatura To y T la temperatura para el tiempo tT, se tiene que:

tT
t0

T0

 T

241

(7.9)

Menndez Gutirrez, C. y J. Prez Olmo

Aplicando la ecuacin anterior, Gloyna y col., realizando una serie de experimentos, concluyeron que, para obtener una reduccin de 80 a 90 % de la DBO
a partir de un afluente de 200 mg L, se hace necesario:
a) tiempo de retencin de 7 das,
b) temperatura de 35oC,
c) considerar Q = 1.085 y
d) aguas negras domsticas.
Teniendo en cuenta todo lo anterior, la ecuacin 7.9 se transforma:

tr
7

1,085 35  T

(7.10)

Por tanto, el tiempo de retencin, en das, necesario para una reduccin


de 80-90% de un afluente cualquiera de aguas domsticas de concentracin
y0 ser:

t=

y0
tT
200

7 y0
1,085 35T
200

(7.11)

V
Q

(7.12)

t=

si
Sustituyendo 7.12 en 7.11:
V

3,5 10 2 Q y0 1,085

35  T

(7.13)

donde:
V: Volumen de la laguna, m3.
Q: Flujo, m3d.
yo: DBO afluente, mg L.
T: Temperatura promedio del mes ms fro, oC.

7.6.2. McGarry y Pescod


Este mtodo destaca la importancia que tiene la temperatura sobre la carga
orgnica que es capaz de asimilar una laguna facultativa, expresada en funcin
del rea,
Carga orgnica mxima admisible:
MAX

6,03 1,0993

OMAX: gmd
242

(7.14)

Lagunas de Estabilizacin

T: temperatura mnima del ao


Carga orgnica mnima recomendable:
O MIN = 2 T - 24

(7.15)

O MIN: gmd
La carga orgnica de proyecto (O), ha de cumplir la relacin,
OMIN < O > OMAX

7.6.3. Mtodo racional. Marais y Shaw


Marais y Shaw basan su mtodo de diseo en la suposicin de que se logra
una mezcla instantnea entre el afluente y el volumen total de la laguna. De
hecho se supone un comportamiento de mezcla completa en las lagunas. Esta
suposicin se basa en los grandes volmenes que generalmente tienen las lagunas, el relativo alto tiempo de retencin, y el efecto de mezcla proporcionado por
las brisas. De esta forma, haciendo un balance de DBO alrededor de la laguna
en estado estacionario (Figura 7.3), se obtiene que:

Qo So - Qo S + V

dS
dt

(7.16)

donde:

dS
: Velocidad de degradacin de la DBO.
dt
Q0: Flujo de agua a la laguna, m3d.
S0, S: DBO de entrada y salida respectivamente, mgL.
V: Volumen efectivo de la laguna, m3.

Fig. 7.3. Esquema para el balance de DBO en una laguna.


asumiendo una cintica de remocin de primer orden con respecto a la DBO,

dS
dt

kS

sustituyendo 7.17 en 7.16 y transformando se obtiene,


243

(7.17)

Menndez Gutirrez, C. y J. Prez Olmo

S
S0

1
1 k

(7.18)

donde:
S: DBO efluente (mg L).
So: DBO afluente (mg L).
k: Constante de la reaccin de primer orden (d).
T : Tiempo de retencin hidrulico (d).
Para lagunas en serie, la ecuacin de Marais puede representarse segn,

Sn

S0
k

k
1

1

1 1 2 2 .... 1  knn

(7.19)

De acuerdo a la ecuacin 7.17, se ha supuesto que la cintica de remocin


de contaminantes en la laguna es de primer orden.
La relacin de las constantes de degradacin a distintas temperaturas es
igual a la relacin de los tiempos de retencin

k 35

kt

35

35  t

(7.20)

La ecuacin 7.18 puede escribirse:

So
S

1 k

(7.21)

Cuando a escala experimental se opera una laguna a distintos tiempos de


retencin (T), la constante de remocin puede calcularse como la pendiente
de la ecuacin 7.21. Figura 7.4.
Generalmente el empleo de la ecuacin de primer orden de Marais y Shaw
arroja como resultado lagunas con sobre diseo. Sin embargo esto no ha invalidado su empleo.

7.6.3.1. La constante k
Como en toda constante cintica, el valor de k depende, de entre otros factores, de la temperatura.
Cuando no se conoce el verdadero valor de la constante de reaccin, un
criterio conservador recomendado por Marais es el de 1,2 da 35 qC. Marais12
recomienda asumir este valor, debido a que para tiempos de retencin hasta de

244

Lagunas de Estabilizacin

40 das, una constante con ese valor corresponde a reducciones de la DBO


inferiores a las observadas experimentalmente en lagunas primarias.
kt

k35 t  35
Q | 1,085

(7.22)

Fig. 7.4. Clculo de la constante de remocin k.

7.6.3.2. Influencia del tiempo de retencin en la eficiencia


de remocin
El tiempo de retencin influye en la eficiencia de las lagunas. Sin embargo, a partir de un determinado valor, un incremento de ste influye poco
en la remocin de DBO. En la tabla 7.1 se aprecia que, independientemente de la temperatura de trabajo, el % de remocin tiene un lmite, y este se
alcanza a un menor tiempo de retencin en la medida que la temperatura
es mayor.
En la prctica no deben esperarse las eficiencias de la tabla 7.1, ya que
las condiciones para las que se estimaron presuponen condiciones ideales:

mezcla instantnea y completa del afluente,


pocos slidos sedimentables,
pequeas variaciones de temperatura,
predominio de condiciones aerobias y
baja DBO en el fondo.

245

Menndez Gutirrez, C. y J. Prez Olmo

Tabla 7.1. Eficiencia de remocin terica de DBO para distintos tiempos de


retencin y valores de k (segn la temperatura)
-1

t(d)
7
10
15
20
30
40

0.24
62.7
70.6
78.2
82.7
87.8
90.6

VALORES DE k (d )
0.35
0.53
0.80
71.0
78.7
84.8
77.8
84.1
88.9
84.0
88.8
92.3
87.5
91.4
94.1
91.3
94.1
96.0
93.3
95.5
97.0

1.2
89.4
92.3
94.7
96.0
97.3
98.0

Ejemplo 7.1
Dimensione una laguna facultativa para tratar 560 m3 d de un agua residual de un proceso y cuya constante de velocidad de remocin es de 0,15 d.
La DBO de entrada a la laguna es de 350 mg L y la deseada en el efluente
debe ser menor de 50 mg L.

0,14

Aplicando la ecuacin 7.18

1
1  0,15

T = 41 d
Volumen = 560 41 = 22 960 m3
Asumiendo una profundidad de 1,8 m,
rea de la laguna

22 960
12 756 m 2
1,8

Comprobando la carga orgnica:

CO

350 22 960
15 g m  2 d 1
41 12 752

por tanto la carga orgnica tiene un valor que corresponde al intervalo caracterstico para las lagunas facultativas.
Ejemplo 7.2
Determine el rea total requerida para las condiciones del ejemplo anterior,
pero suponiendo 2 lagunas en serie.
Considere que k1 = k2 = 0,15 d y T1 = T2
1
0,14
1  0,15 2
Para 2 lagunas en serie:

246

Lagunas de Estabilizacin

T = 11,15 d
Tiempo total de retencin para las 2 lagunas: 22,3 d
AT = A1 + A2 = 3 468 + 3 468 = 6 936 m2
Comprobando la carga orgnica
para la primera laguna CO

350 6 244
11.15 6 936

para la segunda laguna CO

175 6 244
14 g m 2 d 1
11,15 6 936

28 g m 2 d 1

La carga orgnica de la primera laguna es algo alta, pero se puede ver


compensada con la de la segunda.
En la prctica, k1 ! k2, y como consecuencia de ello, no siempre es tan
evidente la ventaja del empleo de 2 o ms lagunas en serie para la remocin de
DBO. La mayor justificacin en el uso de este arreglo se encuentra en la remocin de patgenos.

7.7. RELACIN REA: VOLUMEN PARA LAS LAGUNAS


La carga orgnica superficial de la laguna determina su rea en la superficie,

AS

QS
CO

(7.23)

Q: flujo de agua residual, m3d.


S: DBO en el efluente, mgL.
CO: carga orgnica, gmd.
Una vez determinada el rea en la superficie de la laguna, el rea en el
fondo depende de la inclinacin del talud, y la profundidad que se seleccione
para la laguna. Si la inclinacin es 1:P,
AS  2 . P H L  a  2 P H

AF

(7.24)

donde:
H: profundidad efectiva.
L: longitud de la laguna en la superficie.
a: ancho en la superficie.
El volumen efectivo de la laguna ser,

H
AS  AF 
3
247

AS AF

(7.25)

Menndez Gutirrez, C. y J. Prez Olmo

7.8. REMOCIN DE PATGENOS


Una de las ventajas de las lagunas radica en la posibilidad de eliminacin de
patgenos. Mientras que la remocin de coliformes mediante otros tratamientos
biolgicos alcanza hasta 95 %, en las lagunas se aprecian frecuentemente valores de este indicador hasta (99,999,999) %.14
Las mayores remociones de patgenos en las lagunas se logran cuando se
est en presencia de altos tiempos de retencin, baja turbiedad, alto pH y bajos
valores de DBO.
Hoy da se reconoce que la incidencia de la radiacin solar puede ser un
elemento de consideracin en la muerte de las bacterias.11 Sin embargo,
teniendo en cuenta que la penetracin de la luz en las lagunas solo alcanza
unos (10 a 15) cm y que en la superficie raras veces se encuentran altas
concentraciones de bacterias, la posibilidad de incidencia de la radiacin
solar sobre la reduccin de patgenos es mayor en la medida que es menor
la turbiedad, de manera que se favorezca la penetracin de la luz.
Altos valores de pH aceleran la muerte de las bacterias al exponerlas a un
medio hostil. Una laguna que opere normalmente puede llegar a tener valores de
pH entre 9 y 10 durante determinadas horas del da debido a la accin fotosinttica
de las algas.12,13 Estos valores favorecen la reduccin de patgenos.
Las bajas concentraciones de DBO en las lagunas favorecen la muerte de
patgenos al carecer del sustrato necesario para su subsistencia. Es por ello
importante mantener concentraciones de DBO 20 mgL en las lagunas para
propiciar una disminucin apreciable de patgenos. El empleo de dos o ms
lagunas en serie tambin favorece este propsito.
El empleo de altos tiempos de retencin no est asociado solamente con
la obtencin de bajos valores de concentracin de DBO en el efluente, sino
con la intencin de favorecer la eliminacin de patgenos a travs de la
sedimentacin.

7.9. BALANCE HDRICO DE LAS LAGUNAS


Un aspecto importante de las lagunas de estabilizacin, no siempre tenido en
cuenta, es el balance hdrico.
El balance hdrico permite conocer si existe o no percolacin en el dispositivo de tratamiento.
El balance de flujo en la laguna puede formularse,
4H 4D 3U3F  (3H
(7.26)
Qe: Caudal efluente.
248

Lagunas de Estabilizacin

Qa: Caudal afluente.


Pr: Precipitacin sobre la laguna.
Pc: Infiltracin de agua hacia la laguna.
E: Evaporacin.
Pe: Prdidas por percolacin.
Las variables anteriores pueden trabajarse en m3d.
La precipitacin y evaporacin en la laguna pueden medirse experimentalmente
o tener informacin de ellas a travs de estaciones meteorolgicas.
Aun considerando el mes ms critico del ao (el de menos lluvia, el nivel
fretico ms bajo, de mayor evaporacin), el valor de Qe tiene que ser positivo.
En aquellos casos en los que Qe sea negativo, una alternativa de solucin consiste en impermeabilizar la laguna. Esto ltimo puede resultar ms
econmico si se emplean lagunas anaerobias primarias. Estas ltimas pueden tener 60 % del rea de una equivalente facultativa, lo cual es muy importante desde el punto de vista hidrulico.
La evaporacin que ocurre en una laguna puede estimarse a travs de la
ecuacin de Meyer,

15 Va -V H r 1 

16

(7.27)

E: Evaporacin mensual, mm.


Va: Presin de vapor del agua a la temperatura del agua, mmHg.
V: Presin de vapor del agua a la temperatura del aire, mmHg.
Hr: % humedad relativa.
Q : Velocidad del viento km.h.
Por otro lado, en aquellos casos en los que se disponga de informacin necesaria, las prdidas por percolacin pueden ser estimadas aplicando la ley de
Darcy,
Q=PIA
(7.28)
3 
Q: Caudal que percola m s .
P: Permeabilidad del suelo m s.
A: rea expuesta m2.

I=

h
L

gradiente hidrulico

L: Distancia del fondo de la laguna al nivel fretico.


h: Distancia de la superficie del agua en la laguna al nivel fretico.
por tanto, la profundidad de la laguna es igual a K/ 
249

(7.29)

Menndez Gutirrez, C. y J. Prez Olmo

Cuando se emplea como impermeabilizante algn tipo de suelo (arcilla), por


supuesto de menos permeabilidad que el suelo original, los valores de L y h son:
L = espesor de la nueva capa de arcilla
h = L + la altura del agua en la laguna.
Ejemplo 7.3
Una laguna de estabilizacin de 10 000 m2 va a ser construida en un
suelo cuya permeabilidad es de 10 m.seg . El nivel de agua fretica se
encuentra 3 metros por debajo del nivel del fondo de la laguna. La mxima
altura del agua en la laguna ser de 1,5 m. Determine las prdidas por
percolacin que tendr la laguna en las condiciones anteriores, y en caso
que se sustituyan 40 cm de material del fondo por una arcilla con una permeabilidad de 10 m.seg .

P.I.A

h
L

9 ,5
3

Para el suelo natural:

Q 106 3,17 10 4

3,17

3,17 102 m3 seg 1

Sustituyendo 40 cm de suelo con una capa de arcilla cuya permeabilidad es


10 mseg ,


250

Lagunas de Estabilizacin

1,9
0 ,4

Q 1010 4,75 10 4

4,75

4,75 10 6 m3 seg 1

7.10. CRITERIOS PARA LA OPERACIN


Y MANTENIMIENTO DE LAS LAGUNAS
DE ESTABILIZACIN
No obstante la simplicidad de estos dispositivos de tratamiento, no estn
exentos de un conjunto de requisitos a tener en cuenta para su buena operacin, as como medidas a considerar para su mantenimiento. Desconocer
esto es frecuentemente la causa por la que no se obtienen las eficiencias de
depuracin que deban esperarse.

7.10.1. Lagunas facultativas


Caractersticas de una buena operacin
Dos caractersticas muy fciles de observar y que son indicadoras de que la
laguna facultativa opera adecuadamente son:
Poseer oxgeno disuelto en toda la masa lquida alcanzada por la radiacin
solar.
El efluente presenta una coloracin verde intensa, parcialmente transparente y sin slidos flotantes. Una coloracin ceniza claro indica que
la carga a la que est siendo sometida la laguna es mayor a la admitida
para mantener su condicin de facultativa.
Problemas frecuentes que pueden presentarse
Aparicin de natas. Puede deberse a:
flotacin del lodo de la laguna,
floracin de algas (super produccin de algas) y
exceso de detergentes.
Olores desagradables. Puede deberse a:
sobre carga de materia orgnica, tiempo nublado o baja temperatura,
presencia de sustancias txicas,

251

Menndez Gutirrez, C. y J. Prez Olmo

corto circuito y
presencia de materia flotante.
Aparicin de moscas e insectos. Puede deberse a:
crecimiento y vegetacin de los taludes,
slidos flotantes.

7.10.2. Lagunas anaerobias


Caractersticas de una buena operacin
Elementos distintivos de una buena operacin de las lagunas anaerobias y
que se evidencian de manera inmediata lo constituyen la ausencia de oxgeno
disuelto y la coloracin oscura del agua.

Problemas frecuentes que pueden presentarse


Los problemas que suelen presentarse en la operacin de estas lagunas son
muy similares a los que se observan en las facultativas, aunque se suman otras
causales. Por ejemplo, debido a las caractersticas de los procesos que ocurren
en el metabolismo anaerobio, los malos olores pueden ser producidos adems
por deficiencia de alcalinidad del medio.

7.10.3. Operaciones de control y mantenimiento


Parmetros o elementos que deben ser controlados de manera rutinaria

Demanda Qumica y Bioqumica de Oxgeno en el afluente y efluente.


Concentracin de oxgeno disuelto.
Apariencia.
Caudal y pH de afluente y efluente.

Acciones de mantenimiento

Limpiar las obras de entrada y salida.


Mantener los taludes libres de vegetacin mediante la chapea regular.
Retirar slidos flotantes de la superficie.
Romper la nata que se forma, manualmente o mediante el empleo de
chorro de agua.

252

Lagunas de Estabilizacin

Notas bibliogrficas
1 MENNDEZ, C.3ULQFLSLRVGHGLVHxRGHODJXQDVGHR[LGDFLyQIng. Hidrulica, vol. 10, no. 3, 1989.
2 MENNDEZ, C. Y L. GUERRA: Relaciones de alcalinidad y Ph en lagunas facultativas, Reporte de Investigacin, ISPJAE, Ciudad de La Habana, 1990.
3 THIRUMURTHI, D.:'HVLJQFULWHULDIRUZDVWHVWDELOL]DWLRQSRQGV-RXUWater
Poll. Control Fed., vol.49, no. 9, 1974.
4 ECKENFELDER, W.W.: Industrial Water Pollution Control, 2d. Ed. Mc Graw
Hill, 1989.
5 SAENZ, R.: Lagunas de Estabilizacin, Manual DTIAPA, no. 14, CEPIS,
Per, 1988.
6 MARTN, M. Y C. MENNDEZ:'LVHxRGHODJXQDVGHR[LGDFLyQIngeniera en
Procesos Qumicos, vol. 3, no. 2, 1978.
7 MENNDEZ, C. Y L. GUERRA:/DV/DJXQDVGH2[LGDFLyQ\OD3RVLELOLGDGGHO
8VR GHO 0RGHOR GH 0H]FOD &RPSOHWD Tecnol. del Agua, vol. 15, no.
136, 1995.
8 MCGARRY Y PESCOD: Stabilization Pond. Design criteria for tropical Asia.
Missouri Basin Egn. Health Council, Kansas City, 1970.
9 FINNEY, B. AND E. MIDDLEBROOKS)DFXOWDWLYH:DVWH6WDELOL]DWLRQ3RQG'H
VLJQ-RXUWater Pull, Control Fed, vol. 52, no. 1, 1980.
10 WHO: :DVWHZDWHU6WDELOL]DWLRQ3RQGV3ULQFLSOHVRISODQQLQJDQGSUDFWL
FH:+2 EMRO Tech. Publication no. 10. World Health Organization. Regional Office for the Eastern Mediterranean, Alejandra, 1987.
11 HERMANN, E.R. AND E.F. GLOYNA::DVWHVWDELOL]DWLRQSRQGV,,,)RUPXODWLRQ
of design equations, Sewage and Ind. Wastes, vol. 30, no. 8, 1958.
12 MARAIS, G. VR.:1HZIDFWRUVLQWKHGHVLJQRSHUDWLRQDQGSHUIRUPDQFHRI
ZDVWHVWDELOL]DWLRQSRQGV%XOOHWLQWorld Health Organization, vol. 34,
no. 5, 1966.
13 P OLPRASERT, C. AND K. BHATTARAI0RGHOIRU:DVWH6WDELOL]DWLRQ3RQGV
Jour. Env. Eng. ASCE III, no. 1, 1985.
14 JAMES, A.$QDOWHUQDWLYHDSSURDFKWRWKHGHVLJQRIZDVWHVWDELOL]DWLRQSRQGV
Int. Conf. On Waste Stabilization Ponds, IAWPRC, Lisbon, Portugal,
1987.

253

Digestin Anaerobia

CAPTULO 8

DIGESTIN ANAEROBIA

8.1. INTRODUCCIN
La descomposicin anaerobia conduce a la degradacin de la materia orgnica y su transformacin, en ausencia de oxgeno libre, en compuestos ms
simples como metano y dixido de carbono.

Fig. 8.1. Secuencia de procesos en la digestin anaerobia.


Se trata de un proceso complejo que es consecuencia de muchas reacciones diferentes, tal como se esquematiza en la figura 8.1.
La principal aplicacin de este tratamiento se halla en la estabilizacin de
lodos con alto contenido de materia orgnica, as como en el tratamiento de
255

Menndez Gutirrez, C. y J. Prez Olmo

residuales de alta carga. Sin embargo, a travs de los aos se han desarrollado
nuevas tecnologas basadas en el propio metabolismo anaerobio que facilitan el
tratamiento ya no solo de los lodos, sino que pueden ser tratados exitosamente
residuales orgnicos diluidos.

8.2. MECANISMO DE LA DIGESTIN ANAEROBIA


Los microorganismos causantes de la descomposicin anaerobia generalmente
son clasificados en dos grandes grupos. El primer grupo hidroliza y transforma
los compuestos orgnicos en cidos voltiles, de los cules los ms comunes son
el actico y el propinico. El segundo grupo convierte los cidos voltiles formados en CH4 y CO2. Ambos grupos de microorganismos son necesarios para la
descomposicin anaerobia. Las bacterias formadoras de cido no son capaces
de producir CH4 y las formadoras de metano no pueden por s mismas descomponer la materia orgnica original en compuestos ms sencillos.

Fig. 8.2. Mecanismo de la digestin anaerobia.


256

Digestin Anaerobia

Analizando en mayor detalle la descomposicin de la materia orgnica,


puede plantearse que mediante la accin hidroltica, los microorganismos
degradan los polmeros como los polisacridos y protenas, a monmeros.
Posteriormente estos monmeros son convertidos en cidos voltiles. Finalmente
es que se produce la formacin de CH4 y CO2 . As en la digestin anaerobia
se distinguen cuatro niveles trficos: hidrlisis, acidognesis, acetognesis y
metanognesis. El metano se produce tanto a partir del cido actico como
del H2 y CO2 . 1 Figura 8.2.
No en todas las etapas de la digestin ocurre disminucin de la DBO. As, la
hidrlisis ocurre sin variacin detectable de la DBO. En general durante la acidificacin la disminucin de la DBO es mnima. Por tanto es en la metanognesis
donde ocurren las mayores disminuciones de la DBO.2

8.3. DISTINTAS VARIANTES DE LOS PROCESOS


DE DIGESTIN ANAEROBIA
Hoy en da son muchos y variados los procesos de digestin anaerobia que
se utilizan, y son diversos los factores que inciden en que se emplee una u otra
variante.

8.3.1. Reactores de primera generacin


Son aquellos en que la biomasa se encuentra en relativo reposo, con un
mnimo contacto con el sustrato, o tambin en suspensin, pero sin
recirculacin de slidos. Por esta razn la relacin (edad del lodo/tiempo de
retencin hidrulico) es igual a 1, como es el caso de los reactores completamente mezclados.
Ejemplos de reactores de primera generacin son las fosas spticas, los
tanques Imhoff, los digestores convencionales, las lagunas anaerobias y los reactores completamente mezclados aplicados a la estabilizacin de los lodos de
extraccin de plantas de tratamiento de aguas residuales.

8.3.1.1. Digestores convencionales


Estos digestores trabajan a flujo discontinuo, y debido a que no son agitados,
estn estratificados: una capa superior de nata, relativamente inactiva; una capa
intermedia de la que sedimentan los slidos; y una capa inferior de slidos digeridos. Figura 8.3.

257

Menndez Gutirrez, C. y J. Prez Olmo

Fig. 8.3. Esquema de un digestor de lodo convencional.


Los digestores convencionales son apropiados para el tratamiento de
lodos orgnicos y residuales lquidos de alta demanda bioqumica de oxgeno
(mayor de 20000 mgL ). Se requiere de altos tiempos de retencin.
Aunque en general, como ya fue mencionado, los reactores de primera generacin fueron inicialmente concebidos sin recirculacin de
lodos, existe una variante de estos que tiene incluida la recirculacin para
producir un incremento de la relacin edad del lodo  tiempo de retencin
hidrulico y hacer un proceso ms estable. Esta variante se conoce como
contacto anaerobio. Figura 8.4. En la tabla 8.1 se reflejan algunas de las
caractersticas del contacto anaerobio.

8.3.1.2. Digestores completamente mezclados


En este tipo de digestor la carga de slidos es mayor que en el convencional.
El lodo se mezcla ntimamente mediante la recirculacin del propio gas que se
genera en el proceso o a travs del empleo de agitadores mecnicos.

258

Digestin Anaerobia

Fig. 8.4. Esquema de un digestor de contacto anaerobio.


Exceptuando que trabaja con cargas mayores, y la existencia de la agitacin, son pocas las diferencias entre este tipo de digestor y el convencional.
Tabla 8.1. Caractersticas del reactor de contacto anaerobio
Ventajas
x Soporta aguas con slidos en
suspensin.
x Puede admitir aguas que
forman precipitados.
x Soporta picos orgnicos.
x Arranque rpido con inculo
adecuado.
x Se puede incorporar carbn
activado en polvo para tratar
aguas con compuestos
inhibidores.

Desventajas
x Recibe relativas bajas cargas
orgnicas (6 kgm3d).
x Grandes tiempos de
retencin y volmenes de
reactor.
x Altamente dependiente de la
calidad de la sedimentacin
del lodo anaerobio.
x Costos energticos asociados
al mezclado y la
recirculacin.

8.3.2. Reactores de segunda generacin


Se caracterizan porque los microorganismos son retenidos en el reactor por
la presencia de un soporte al que se adhieren o bien por su sedimentacin. La
primera versin de esta generacin es el filtro anaerobio (Young y McCarty). 3
259

Menndez Gutirrez, C. y J. Prez Olmo

Posteriormente se desarroll una variante ms eficiente con la introduccin de


un soporte tubular, ordenndose de esta manera el empaque (Van D Berg y
Lentz).4 Lettinga (1980)5 desarroll el sistema UASB (Upflow Anaerobic Sludge
Blanket) o Reactor Anaerobio de Manto de Lodos y Flujo Ascendente, con
caractersticas de pelcula fija pero sin material de soporte.

8.3.2.1. Filtro anaerobio


Este dispositivo consiste en un reactor empacado con material inerte en
el que los microorganismos crecen sobre la empaquetadura. No obstante, la
experiencia demuestra que no toda el rea disponible de empaquetadura es
cubierta por la biopelcula. En el filtro anaerobio usualmente se presenta una
relativamente alta fraccin de biomasa que no se adhiere al medio soporte,
sino que permanece atrapada en los espacios libres de la misma. 6 Los filtros
pueden operarse a flujo ascendente o descendente. Figura 8.5.

Fig. 8.5. Esquema de un filtro anaerobio de flujo ascendente.


Estos reactores son estratificados como consecuencia del poco mezclado
que existe en su interior. Por lo tanto, se aprecia una tendencia de los
microorganismos a distribuirse de acuerdo a sus requerimientos y las caractersticas del sustrato. La zona de mayor actividad es la del afuente, existiendo
mayor fraccin de microorganismos saprofitos facultativos y anaerobios que
hidrolizan y degradan la materia orgnica compleja, que en la regin ms prxima al efluente.
260

Digestin Anaerobia

La elevada concentracin de microorganismos dentro del reactor hace que


los tiempos de retencin alcancen valores comprendidos entre 3 h y 6 h
obtenindose altas eficiencias y elevada produccin de biogs.
Teniendo en cuenta el alto tiempo de retencin de los slidos, en esta variante no se utiliza recirculacin de biomasa. Jennett y Dennis7 han reportado remociones de DQO entre 80 y 98 % en el tratamiento de residuales lquidos de la
industria farmacutica con tiempos de retencin entre 12 y 48 horas operando
con cargas de 0,22 y 3,5 kg DQO md respectivamente.
Algunas de las caractersticas de los filtros anaerobios se resumen en la
tabla 8.2.
Tabla 8.2. Caractersticas de los filtros anaerobios
x
x
x
x
x
x

Ventajas
Soporta altas cargas, hasta
(15 kg DQO/m3d).
Con recirculacin es
resistente a picos orgnicos o
txicos.
Simple construccin.
Verstil. Aplicable a pequea
y mediana escala.
Operacin simple.
Re-arranque rpido.

x
x
x
x

Desventajas
Arranque lento.
Riesgo de tupicin.
Muy sensible a la presencia
de slidos suspendidos en el
afluente.
Presencia de slidos
suspendidos en el efluente.

8.3.2.2. Reactores de manto de lodo de flujo ascendente


La carga permisible en los procesos anaerobios est limitada por la edad del
lodo en los reactores. La necesidad de mantener altas edades de lodo ha sido
siempre el mayor problema prctico de la aplicacin de estos procesos, especialmente para residuales con demanda bioqumica de oxgeno relativamente
baja (~ 5000 mgL). Obviamente, en general, un proceso es ms econmico en
la medida que los tiempos de retencin requeridos son menores.
La bsqueda de mayores edades de lodo en los reactores para lograr tratamientos ms intensivos ha conducido a las diferentes variantes de reactores
anaerobios. El digestor de manto de lodo busca este propsito. Este sistema fue
descrito inicialmente por Coulter,8 pero debe su desarrollo actual y popularidad a
Lettinga.5 Figura 8.6. Su caracterstica principal est dada por la capacidad que
poseen de retener biomasa sin necesidad de un soporte, como consecuencia de
la formacin de granos o pellets. Las principales caractersticas de este reactor
se resumen en la tabla 8.3.
261

Menndez Gutirrez, C. y J. Prez Olmo

Fig. 8.6. Esquema de reactor de manto de lodo.


En este digestor se distinguen 3 zonas bien definidas:
Zona de lecho de lodos.
Zona de los microorganismos dispersos.
Zona de separacin gas - lquido - slido.
Entre las condiciones que favorecen la buena operacin de este tipo de
reactor pueden citarse:
Elevada concentracin de biomasa, limitada solo por las condiciones del
separador gas-lquido-slido de la parte superior del reactor. La concentracin de slidos totales puede alcanzar hasta 150 g.L .
Elevada actividad de la biomasa.
Altos tiempos de retencin (edad del lodo) de la biomasa.
Bajos tiempos de retencin hidrulicos. Para ello el sistema de distribucin del afluente debe garantizar el ntimo contacto residual-biomasa.
Produccin de biogs tal, que exista una buena distribucin del lodo sobre
el lecho y en la zona que corresponde a la biomasa dispersa.

262

Digestin Anaerobia

Tabla 8.3. Caractersticas del reactor de manto de lodo

Ventajas
x Soporta altas cargas
(20 kg DQO/m3.d).
x Su construccin es
relativamente simple.
x Con inculo apropiado tiene
arranque inmediato.
x Aplicable a todas las escalas.
x Operacin sencilla.

Desventajas
x La granulacin es lenta y no
controlable.
x No todas las aguas favorecen
la granulacin.
x Sensible a las grasas en el
afluente.
x Sensible a las aguas que
forman precipitados.

En los reactores de manto de lodo de flujo ascendente el agua residual


se alimenta por el fondo, se distribuye uniformemente por toda el rea y
asciende a travs del manto biolgicamente activo en el que experimenta
las transformaciones tpicas de la descomposicin anaerobia. El CH 4 y
CO 2 formados ascienden a lo largo del reactor y son extrados por el domo
superior.
Su principio de funcionamiento se basa en la capacidad de sedimentacin de la biomasa producida en el reactor. Esta se aglomera en forma de
granos de 1 a 3 mm de dimetro y presenta elevada actividad metanognica.
El lavado o escape de los flculos ms pequeos de la zona del manto,
generalmente se minimiza creando dentro del propio reactor una zona tranquila que posibilita que estas partculas vuelvan a sedimentar. 9
En parte el xito de este tipo de reactor se debe a los relativamente altos
valores de concentracin de slidos que se obtienen en el lodo, entre 100 y
150 gL . Han sido reportadas eficiencias de remocin de DBO mayores del
90 % con cargas orgnicas e hidrulicas del orden de 80 kgmd y 5 m3md
respectivamente, con tiempos de retencin hidrulico de 8 h.10

8.3.3. Reactores de tercera generacin


Estos reactores contienen microorganismos en forma de biopelcula adherida a un soporte que se expande o fluidiza. Utilizan una corriente de
recirculacin para provocar un flujo ascendente y mantener fluidizado el
lecho de partculas de soporte. Estas partculas pueden ser arena, material
plstico o cermico. El material soporte brinda una gran rea superficial
sobre la que se adhiere la biopelcula y mantiene una buena sedimentabilidad,
garantizando la retencin celular. 11 Figura 8.7. Las caractersticas de este
reactor se reportan en la tabla 8.4.
263

Menndez Gutirrez, C. y J. Prez Olmo

En la tabla 8.5 se resumen adems datos comparativos de diferentes procesos anaerobios.12

Fig. 8.7. Esquema de un reactor de lecho fluidizado.


Tabla 8.4. Caractersticas del reactor de lecho fluidizado

x
x
x
x

Ventajas
Asimila muy altas cargas
(40 kg DQO/m3d).
Pequeos tiempos de
retencin.
Instalaciones compactas.
Asimila slidos
suspendidos en el afluente.

Desventajas
x Arranque lento y difcil.
x Requiere energa para
mantener la fluidizacin por
bombeo.
x Presencia de slidos
suspendidos en el efluente.

Tabla 8.5. Datos de rendimientos tpicos de procesos anaerobios empleados


en el tratamiento de residuales industriales. Adaptados12
Proceso
De
contacto
UASB
Lecho fijo
Lecho
fluidizado

Afluente
DQO
mgL-1
1500
5000
5000
15000
10000
20000
5000
10000

Tiempo de
retencin
T h
2-10

BV
kg DQOm-3d-1
0,5-6,0

%
remocin
DQO
75 - 90

4-12

4,0-20,0

75 - 85

24-48

1,0-15,0

75 - 85

5-10

5,0-10,0

80 - 85

264

Digestin Anaerobia

8.4. FACTORES QUE CONTROLAN EL PROCESO


DE DIGESTIN
Los factores que controlan los procesos de digestin pueden clasificarse en
dos grandes grupos: Factores bsicos y factores ambientales.

8.4.1. Factores Bsicos


Dentro de los factores bsicos que intervienen en el control de los procesos
anaerobios se destacan:
Bacterias.
Sustrato.
Tiempo de retencin.

8.4.1.1. Bacterias
Las formadoras de metano son las bacterias clave en la digestin anaerobia.
Crecen ms lentamente y son ms sensibles a los cambios ambientales que las
formadoras de cido.
La fermentacin metnica es la etapa limitante de la digestin. Las
formadoras de metano son bacterias estrictamente anaerobias, y por tanto, subsisten en completa ausencia de oxgeno molecular. Estas bacterias son poco
abundantes en un medio dado, como consecuencia de que la mayor fraccin de
la energa que se libera en los procesos de oxidacin anaerobia queda asociada
al CH4 que se forma, por lo que resta menos energa para ser utilizada en el
crecimiento.

8.4.1.2. Sustrato
El sustrato para las bacterias est constituido por los compuestos orgnicos
a ser estabilizados. Los compuestos orgnicos que se someten a digestin
anaerobia comnmente son de estructura muy compleja, que como ya fue mencionado, son transformados a travs de sucesivas etapas hasta la obtencin de
los productos finales, constituidos fundamentalmente por CH4 CO2 y otros compuestos estables y de bajo contenido energtico.
El menor crecimiento de las bacterias anaerobias comparados con el de las
aerobias para igual cantidad de sustrato consumido causa menor requerimiento
de nutrientes. Los sistemas anaerobios producen aproximadamente 20 %
menos lodo que los aerobios para el mismo sustrato, y los rquerimientos de
265

Menndez Gutirrez, C. y J. Prez Olmo

nutrientes decrecen en la misma proporcin. Se han reportado relaciones DQO:N


tan altas como 700:5. El valor de 250:5 parece ser razonable como relacin para
SURFHVRVFRQDOWDV FDUJDV  NJ'42 NJ669G . Para procesos que
operan a cargas ms bajas, la relacin DQO:N puede incrementarse de forma
conservadora multiplicando la relacin 250:5 por un factor igual al valor de la
carga, dividido por 1,2.13

8.4.1.3. Tiempo de retencin


En el anlisis del tiempo como factor que interviene en el control de los
procesos anaerobios, hay que considerar tanto la edad del lodo como el tiempo
de retencin hidrulico.
La edad del lodo tiene mucha aplicacin en el diseo y control de los reactores
completamente mezclados, pero en pelcula fija o reactores en los que la biomasa
no es homognea no tiene uso. Esto se debe a la dificultad prctica de medir la
concentracin de slidos suspendidos voltiles (SSV) dentro del reactor.
El tiempo de retencin hidrulico para la estabilizacin anaerobia depende
de la temperatura y la edad del lodo. Actualmente se conoce que los digestores
pueden operar con tiempos de retencin de hasta de un da o menos en la medida que la edad del lodo sea mayor que un cierto valor crtico (T X,cr). Esta edad
del lodo crtica es el tiempo por debajo del cual la digestin se inhibe como
resultado del lavado de las bacterias, cuyo crecimiento es lento.

X,cr

1
Yk  k b

(8.1)

donde:
T X,cr: edad del lodo crtica.
Y: rendimiento.
k: velocidad especfica de remocin de sustrato, d.
kb: constante de auto-oxidacin, d.
Para edades de lodo menores de 10 das la produccin de CH4 comienza a
disminuir y puede detenerse completamente para valores de tres o cuatro das.14

8.4.2. Factores ambientales


Los factores ambientales para el control de la digestin anaerobia, al igual
que los bsicos, han de ser tomados en cuenta adems para el diseo. Entre los
ms importantes se destacan:
Temperatura.
266

Digestin Anaerobia

pH.
Sustancias txicas.

8.4.2.1. Temperatura
El crecimiento de las bacterias metnicas es lento en comparacin con la
mayora de los otros microorganismos. El tiempo requerido por estas bacterias
para regenerar es funcin de la temperatura. A 35 qC algunas especies requieren cuatro das para duplicar su nmero, mientras que otras necesitan diez das
o ms. Una disminucin de la temperatura implica un mayor tiempo de regeneracin.
Intervalos de temperatura de inters pueden ser los correspondientes al niYHOPHVyILOR qC, con ptimo entre 28 y 33 q& \HOWHUPyILOR qC,
con ptimo entre 50 y 60 qC).
Considerando la produccin de gas como indicador de la degradacin de
los compuestos orgnicos, podra decirse que el intervalo termfilo es superior. De hecho, pueden obtenerse resultados similares con tiempos de retencin hidrulico de 7 a 14 das en el intervalo termfilo, a los que se obtienen
en el mesfilo con retenciones de 30 das. No obstante la ventaja de trabajar
en el intervalo mesfilo en los pases tropicales radica en el hecho que no es
necesario el uso del calentamiento para mantener la temperatura dentro de
los valores del intervalo.
Harris y Dague15 han reportado la posibilidad de incrementos notables de la
carga de los filtros anaerobios a 56 C con resultados de eficiencia de remocin
similares a los alcanzados a 32 C con la mitad de la carga.

8.4.2.2. pH
Las bacterias metnicas ejercen adecuadamente su funcin en un intervalo
de pH comprendido entre 6,6 y 7,6, con un ptimo cercano al pH 7.16 Cuando la
velocidad de la produccin de los cidos voltiles es mayor que la de su transformacin en CH4, el proceso puede desequilibrarse, resultando en una disminucin del pH, disminucin de la produccin de gas y aumento de las fraccin de
CO2 en el gas producido. El control del pH es por tanto esencial para garantizar
una buena operacin de los procesos anaerobios.

8.4.2.3. Sustancias txicas


Aunque se reconoce una relacin funcional entre algunos de los problemas
de operacin de la digestin anaerobia y la presencia de sustancias txicas, no
267

Menndez Gutirrez, C. y J. Prez Olmo

est clara la incidencia en las transformaciones bioqumicas que tienen estos


compuestos txicos, especialmente el nitrgeno amoniacal y los cidos voltiles
a altas concentraciones.
La toxicidad de los cidos voltiles se debe no tanto a su concentracin
como al efecto del pH, ya que se ha demostrado17 que la toxicidad es originada por la fraccin no ionizada de ellos. Kroeker 18 ha concluido que una
concentracin de cidos voltiles no ionizados, en el intervalo comprendido
entre 30 y 60 mgL (como cido actico) es txica e inhibe los procesos
anaerobios. En la figura 8.8 se muestra que aunque la concentracin de
cidos voltiles sea elevada, si el pH se mantiene relativamente alto, la concentracin no ionizada de cidos voltiles es pequea y la digestin puede
ocurrir sin dificultad. Por tanto, aunque altas concentraciones de por si no
son dainas. Sin embargo, su combinacin con bajos valores de pH pueden
producir situaciones indeseables.
Hay sales inorgnicas que a bajas concentraciones favorecen la digestin,
pero a valores por encima de un cierto lmite pueden inhibir el proceso. Entre
estos pueden citarse el Na+, K+, Ca2+, y Mg2+. En la tabla 8.6 se ofrecen intervalos de concentraciones a las cuales estos cationes pueden ser inhibitorios. Otros
cationes como el zinc, cobre y nquel son txicos an a bajas concentraciones.
El amoniaco es txico a concentraciones superiores a 3000 mgL e inhibitorio a 1500 mgL. La toxicidad de amoniaco est relacionada con el pH, ya
que este compuesto es ms nocivo en su forma molecular que como catin
amonio.14

Fig. 8.8. Relacin entre la forma molecular y disociada de los cidos


voltiles y su incidencia en la digestin.
268

Digestin Anaerobia

Tabla 8.6. Concentraciones inhibitorias de distintos cationes 20


CATIN
Na+
K+
Ca2+
Mg2+

Concentracin mgL-1
Moderadamente
Fuertemente
inhibitorio
inhibitorio
1500 3000
3000
3500 5500
8000
2500 4500
12000
1000 - 1500
3000

8.5. PARMETROS INDICADORES DEL CURSO


DE LA DIGESTIN
Entre los diferentes parmetros que son de utilidad prctica para indicar si
un digestor est operando o no en buenas condiciones, y aun para predecir
futuras complicaciones en la operacin, pueden citarse:
La produccin de gases.
Contenido de cidos voltiles y pH.
La alcalinidad del medio.

8.5.1. Produccin de gases


El grado de estabilizacin de los compuestos orgnicos est directamente
relacionada con la produccin de gases y su composicin. As por ejemplo, la
produccin de gases puede disminuir abruptamente si el tiempo de retencin cae
por debajo de un determinado valor. Figura 8.9. Esto est ntimamente vinculado
con la velocidad de crecimiento de las bacterias formadoras de metano, que
como ya se ha reiterado, es menor que el de las formadoras de cido.

Fig. 8.9. Influencia del tiempo de retencin en la produccin de biogs.


269

Menndez Gutirrez, C. y J. Prez Olmo

En condiciones de operacin normal la produccin de gases, a 101,3 kPa y 0 qC,


est en el intervalo comprendido entre 0,2 y 0,7 m3 por cada kg de DQO
removido 1 m3 por cada kg de slido voltil estabilizado.
En la tabla 8.7 se muestran algunos rendimientos tericos de gas y su produccin por kg de slidos voltiles para sustancias que frecuentemente se encuentran presentes en muchos residuales.
Por otro lado, la composicin de los gases producidos tampoco es constante.
En un digestor anaerobio que funcione en ptimas condiciones, los gases producidos estn constituidos en 75 a 85 % por CH4 y el resto fundamentalmente por
CO2. Un digestor operando en condiciones desfavorables puede mantener durante un tiempo constante la el volumen de gas producido y estar variando la
fraccin relativa de CH4 d CO2.
Tabla 8.7. Rendimientos tericos de biogs en funcin de los constituyentes
del residual
Compuesto
Carbohidratos
Protenas
Grasas

% CH4 en el gas
50
70
84

m3(kg SV digerido)-1
0,89
1,30
0,60

En la figura 8.10, donde se representa la variacin de la relacin de la composicin de los gases con el pH del medio, puede apreciarse cunto llega a
cambiar esa composicin ante condiciones desfavorables de operacin. En este
caso, del pH.

Fig. 8.10. Variacin de la composicin de los gases con el pH.

8.5.2. Contenido de cidos voltiles, pH y alcalinidad


Estas tres caractersticas de un digestor estn muy relacionadas entre s.
270

Digestin Anaerobia

La alcalinidad desempea un papel importante en los procesos anaerobios,


compensando no solo la produccin de los cidos voltiles, sino tambin la de
CO2. La alcalinidad natural de estos procesos es debida al bicarbonato, y da
lugar a un sistema tampn de pH comprendido entre 6,8 y 7,2.
En la figura 8.11 se aprecia que en los lmites de trabajo normal de un tratamiento anaerobio se requiere disponer de una alcalinidad de aproximadamente
1500 mgL cuando el gas tiene en su composicin 30 % de CO2. Cuando la
alcalinidad necesaria no existe en el medio, debe ser adicionada.

Fig. 8.11. Alcalinidad del medio y composicin de los gases en funcin


del pH.
Aunque la alcalinidad es esencial para una digestin balanceada, su
valor en s mismo no es un indicador adecuado que permita predecir posibles
dificultades actuales o futuras del proceso. Esto se debe a que en la medida
en que la alcalinidad al bicarbonato es consumida y destruida, se forma
alcalinidad vinculada a los cidos voltiles. Las determinaciones de alcalinidad
y cidos voltiles pueden utilizarse simultneamente como indicadores de la
condicin de la digestin.21
La relacin entre la alcalinidad total, la debida al bicarbonato y la concentracin de cidos voltiles es:
AT = AB + 0,71 AV
(8.2)
donde:
AT: Alcalinidad total, mgL como CaCO3.
AB: Alcalinidad debida al HCO3, mgL como CaCO3.
AV: Concentracin de cidos voltiles, como (HCH3 COO).
271

Menndez Gutirrez, C. y J. Prez Olmo

Es deseable que la capacidad tampn del proceso de digestin sea alta.


Esto se traduce en que la relacin cidos voltiles / alcalinidad sea baja
(entre 0 y 0,1). Valores de esta relacin cercanos a 0,5, es una seal de la
existencia de problemas en el proceso. El pH del digestor comienza a
descender rpidamente si esa relacin llega a valores tan altos como 0,8.

8.6. MTODOS DE DISEO


Esencialmente hay dos enfoques para el diseo de reactores anaerobios: los
que se basan en mtodos empricos, y los que utilizan criterios cinticos.

8.6.1. Mtodos empricos


Los mtodos empricos se utilizan fundamentalmente cuando se necesita
digerir lodos producidos en alguna etapa anterior del tratamiento.
A modo de criterio, en la tabla 8.8 se brinda la composicin, en slidos voltiles y fijos, de lodos crudos y digeridos.
Tabla 8.8. Composicin de los lodos

Lodo
Crudos

Slidos voltiles % Slidos fijos %


70 a 75
30 a 25

Digeridos

50

50

8.6.1.1. Mtodo de las cargas


El mtodo de las cargas o tradicional para el diseo de digestores de lodo
brinda buenos resultados en la medida que los criterios que se utilicen se basen
en la experiencia previa, sin extrapolar resultados anteriores a una nueva situacin. Este mtodo determina el volumen requerido de reactor en funcin de una
carga asumida.
El factor de carga ms utilizado es que se basa en los kg de slidos voltiles
aadidos por cada m3 de reactor por da. La carga recomendada para los reactores normales est entre 0,03 y 0,10 kg de slidos voltiles (SV) por m3 por da,
con tiempos de retencin de 30 a 90 das. En el caso de reactores de alta carga
es posible utilizar cargas de hasta 1,6 a 6,4 kg de SV por cada m3 por da, para
tiempos de retencin de 10 a 20 das.

272

Digestin Anaerobia

8.6.1.2. Mtodo de la reduccin de volumen


A medida que la digestin tiene lugar, el lodo experimenta una reduccin de
volumen. Teniendo en cuenta esta caracterstica, el volumen del reactor puede
calcularse,

V f  3 V f  Vd . t

(8.3)

donde:
V: volumen del digestor.
Vf: volumen diario de lodo crudo.
Vd: volumen diario de lodo digerido.
t: tiempo de retencin.
Ejemplo 8.1
Estime el volumen de un digestor para tratar 5000 kgd (base seca) de un
lodo orgnico producido en otra etapa de tratamiento. Asuma que 75 % de los
slidos es voltil. Utilice el mtodo de las cargas.
Carga: 4 kg.md
Slidos voltiles en los 5000 kg de lodo = 5000 0,75 = 3750 kg

Volumen del digestor

3750 kg d  1
4 kg m -3 d  1

937 ,5 m 3

Ejemplo 8.2
Determine el volumen de un reactor para digerir 3500 kgd (base seca) de
un lodo que contiene 95 % de humedad. Asuma que 75 % de los slidos es
voltil. Considere que hay una reduccin del 60 % de los slidos voltiles, con
una humedad de 92 %. Tiempo de retencin de 25 das.
Si el lodo que contiene 95 % de humedad, su peso especfico es 1,02 kgL,
por tanto el volumen de lodo a digerir es:

Vf

3500
1,02 . 0,05 . 10 3

68,63 m 3

La masa de lodo despus de la digestin ser 40 % de los voltiles que


entraron ms los fijos que no se degradan (25 % de los slidos totales).
Slidos voltiles en los 3500 kg de lodo = 3500 0,75 = 2625 kg
Slidos fijos = 3500 0,25 = 875 kg
273

Menndez Gutirrez, C. y J. Prez Olmo

El lodo digerido, con 92 % de humedad, tiene un peso especfico de


1,04 kgL

Vd

875  2625 . 0,4


1,04 . 0,08 .10
V

23,14 m 3

V f  3 V f  Vd

68,63  3 68,63  23,14 25

V = 958 m3

8.6.2. Ecuaciones de diseo basadas en criterios cinticos


El uso principal que han recibido los modelos basados en criterios cinticos,
llamados en ocasiones modelos conceptuales, es en el control y simulacin de
los procesos.

8.6.2.1. Tiempo de retencin y volumen del digestor


Para un digestor anaerobio que opere a flujo continuo y mezcla completa, el
tiempo de retencin viene dado por

S S0

Xk S

(8.4)

donde:
' S = S06 mgL.
S: DBO DQO en el efluente.
S0: DBO DQO en el afluente al reactor.
X: concentracin de slidos suspendidos voltiles en el reactor.
k: constante especfica de remocin de sustrato, d.
El volumen efectivo del digestor se obtiene,
V = q Q m3
(8.5)
3 

Q: flujo de entrada al digestor, m d .

274

Digestin Anaerobia

8.6.2.2. Edad del lodo y produccin de slidos


Aunque la produccin de biomasa durante la digestin anaerobia es mucho
menor que en los procesos aerobios, siempre hay que considerarla, pues ella va
a garantizar en cierta medida que el reactor no se lave con la consecuente
inhibicin del proceso.
X

Y %V  k b X kgmd

(8.6)

donde:
' X: produccin de slidos, kgmd.
Y: rendimiento.
' BV: razn de carga volumtrica, kgmd.
Valores tpicos de Y y kb.
Y: 
kb: 
Cuando la produccin de biomasa es ' X, y se desea una concentracin de
SSV en el reactor igual a X, la edad de lodo requerida es,

X
d
;

(8.7)

El valor de la edad de lodo siempre debe ser superior al valor crtico definido
por la ecuacin 8.1.

8.6.2.3. Volumen de gas producido


La produccin de gases en los procesos de digestin es muy variable y
depende en alguna medida de la composicin del agua residual.22 En trminos
generales, y tomando como base la DQO, puede estimarse la produccin total
de gas a TPN como,
(8.8)
GT = 0,5 (' BV1,42 X) V m3d

Bv

donde:
Si lo que se desea es el volumen de CH4,
G = 0,35 (' BV 1,42 DX)V m3d

275

(8.9)

Menndez Gutirrez, C. y J. Prez Olmo

8.6.2.4. Nutrientes
El N y P que se requiere en los procesos anaerobios es menor que los que
exigen los aerobios,
(8.10)
N = 0,12 'XV kgd
P = 0,025 ' XV kgd

(8.11)

Ejemplo 8.3
Determine las dimensiones de un digestor completamente mezclado para
obtener 87 % de remocin de DQO de un agua residual cuyo flujo es de
750 m3 d .
DQO del afluente = 16000 mgL
Y = 0,1
kb = 0,02 d
k = 6 d
X = 6000 mgL
Edad del lodo mnima = 15 d
Clculo del tiempo de retencin a partir de la informacin cintica:

6 S 0

kX S

13920 16000

6 . 6000 2080

3d

Volumen efectivo del digestor:


V = q Q = 3 750 = 2250 m3
Comprobacin de la edad del lodo:

X
X

0 ,1

X
;

Y %V  k b X

13920
 0,02 6000 344
3

g m - 3 d 1

Por tanto,

6000
17 ,4 d
344

La edad del lodo es superior a la mnima fijada por las condiciones de


diseo.
276

Digestin Anaerobia

Produccin de CH4 a TPN:


G = 0,35 ('BVDX)V
G    2250
G = 3269 m3d

8.6.3. Mtodo experimental para determinar los parmetros


cinticos
El procedimiento de laboratorio que se detalla a continuacin es en esencia el descrito por Eckenfelder y Ford.23

Procedimiento
Para las pruebas en laboratorio se emplea un reactor con un volumen
efectivo de 1 a 5 litros. El montaje del equipamiento necesario est
esquematizado en la figura 8.12.
1. Tomar un volumen de lodo digerido activo de una instalacin en operacin. Filtrar el lodo a travs de una malla gruesa para eliminar partculas
como semillas, paja, etc. Si fuera necesario, diluir con agua corriente.
2. Introducir el lodo en el digestor. Mantener la temperatura constante a 35 C
y agitado en contenido del reactor durante todo el tiempo que dure la
experiencia.
3. No alimentar agua residual ni extraer licor mezclado del reactor hasta
que se aprecie produccin de gas. Asegurar que el dispositivo est hermtico para evitar fuga de gas.
4. Una vez que se aprecie produccin de gas, introducir residual aplicando
una carga a 4 kg DBO m d , por un periodo no menor a un tiempo
equivalente a un tiempo de retencin. Es aconsejable, durante este periodo inicial de aclimatacin, alimentar el reactor 2 3 veces en el da, de
manera tal que la carga total no sobrepase los 4 kg DBO m d .
5. El pH es un parmetro que siempre debe ser vigilado, pero ms cuidadosamente durante la aclimatacin. En caso de disminuir por debajo de 6,
aadir alcalinidad para mantenerlo entre 6,5 y 7,5. El licor mezclado debe
tender a crear su propio sistema tampn.
6. Proceder a la alimentacin del residual con un flujo continuo, con una
extraccin diaria de un volumen igual al alimentado.

277

Menndez Gutirrez, C. y J. Prez Olmo

Fig. 8.12. Digestor anaerobio a escala de laboratorio.


7. Registrar la siguiente informacin:
Tabla 8.9. Informacin a obtener en el laboratorio
Residual crudo (afluente)
pH
DBO DQO

Efluente (licor mezclado)


PH
DBO DQO
Alcalinidad
Acidez
SSV
cidos voltiles

8. Registrar el volumen de gas producido diariamente y su composicin (CH4,


CO 2 )
9. Tabular los datos obtenidos segn se muestra en la tabla 8.10 del ejemplo 8.4.
10.Calcular las constantes k,Y y kb graficando la data segn la informacin de
las tablas 8.10 y 8.11. En las figuras 8.13 y 8.14 aparecen los grficos correspondientes.
Ejemplo 8.4
En pruebas de laboratorio realizadas en buenas condiciones de operacin,
se obtuvieron los datos de la tabla 8.10 para un digestor anaerobio.
Determine:
a) Velocidad especfica de remocin de DBO.
b) El rendimiento y la constante de autooxidacin.
278

Digestin Anaerobia

Tabla 8.10. Tabla de trabajo del ejemplo 8.4


T
(d)
0,25
0,50
2,0
3,0
4,5
5,0

S0
(mgL-1)
15000
15000
15400
14800
15150
14750

S
(mgL-1)
13500
12000
9000
7250
4500
3750

TX
(d)
2
3
4
6,25
20
28

X
(mgL-1)
1200
1500
1164
1260
3156
4400

a) Considerando que se cumple que

dS
dt

kX

G
(m3d-1)
14
15
17
24
27
25

S
S0

se prepara la tabla 8.11


1

Tabla 8.11. Tabla de trabajo del ejemplo 8.4

' BX
(kgkg-1d-1)
S
S0

Se grafica B X

5,0

4,0

2,75

2,0

0,75

0,50

0,9

0,8

0,58

0,49

0,30

0,25

contra

S
S0

segn la figura 8.13. La pendiente da el

valor de k = 6,7 d

1
b) Para el clculo de Y y kb, se grafica
X
a) contra ' BX de acuerdo con la tabla 8.12 y la figura 8.14
Tabla 8.12. Tabla de trabajo del ejemplo 8.4

' BX
(kgkg-1d-1)

5,0

4,0

2,75

2,0

0,75

0,50

1
X

0,5

0,33

0,25

0,16

0,05

0,036

279

Menndez Gutirrez, C. y J. Prez Olmo

'BX

Fig. 8.13. Clculo de la constante de velocidad ('BX vs S/So).

Fig. 8.14. (1/X vs BX ).

8.6.4. Consideraciones de diseo para reactores UASB


El diseo de los reactores de manto de lodo de flujo ascendente en
general se basa en elementos hidrulicos para DQO menores de 1000 mgL 
y tiempos de retencin inferiores a 8 h. La velocidad ascensional en la
zona de paso del dispositivo separador superior debe ser mantenida menor
que 3 m3 m h para lodos floculentos (no granulares). Esta velocidad puede llegar hasta 6 m3 m h para los lodos granulares tpicos. En todos los
casos se recomienda que en la zona de sedimentacin la velocidad sea
menor que 1 mh .

280

Digestin Anaerobia

Cuando el criterio de diseo es hidrulico el volumen efectivo del reactor se


calcula,
(8.12)
V = T Q
Para aguas residuales con elevada DQO, el criterio de diseo se fundamenta en la carga orgnica volumtrica (BV). Por lo tanto, el clculo del volumen
ser,

S0 Q
BV

(8.13)

En la figura 8.15 se refleja la variacin de la carga volumtrica recomendada en el diseo de reactores UASB, en funcin de la temperatura, para reactores con 25 kg SSVm.
Para el tratamiento de aguas residuales diluidas, similares a las domsticas, la altura recomendada para los reactores es de 3 a 5 m. Para DQO
entre 1000 y 5000 mgL , los mejores resultados se obtienen con alturas de
reactor de 5 a 6 m.

Fig. 8.15. Carga orgnica volumtrica en funcin de la temperatura


para el diseo de reactores UASB. 10
En los reactores UASB el sistema de distribucin del afluente y el separador
gas-slido-lquido, revisten especial importancia.
Del sistema de distribucin depende que el manto de lodos tenga un contacto ms ntimo con el agua residual, evitando las zonas muertas y canalizaciones.
La distribucin se hace a travs de boquillas. Las boquillas pueden distribuirse
en el fondo del reactor con una densidad que puede variar desde 1 2 por m2
para aguas con DQO menores de 1000 mgL, y hasta 6 por cada m2 en el caso
de BV 4 kg DQOmd.

281

Menndez Gutirrez, C. y J. Prez Olmo

Entre los objetivos del separador gas-slido-lquido pueden sealarse:


Facilitar la separacin y descarga del biogs.
Prevenir, mediante decantacin, el lavado de flculos y granos.
Como ya fue destacado, en el separador se considera de importancia la
velocidad en los pasos entre colectores y en la zona de sedimentacin. Se recomienda que el rea de paso entre los colectores sea aproximadamente 20 % del
rea superficial del reactor.

8.7. PUESTA EN MARCHA DE LOS DIGESTORES


ANAEROBIOS
Sin duda, uno de los aspectos ms complejos de la operacin de los digestores
es la puesta en marcha.
Aun cuando cada digestor tiene sus especificidades y por lo tanto, su propia
estrategia de puesta en marcha, se destaca un conjunto de aspectos que de
manera general ha de considerarse en todos ellos. Estos son:
Inculo.
Carga orgnica.
Control de los cidos grasos voltiles.
Como inculo de los digestores es preferible el empleo de una poblacin
de microorganismos que haya sido previamente adaptada al sustrato que va
a ser utilizado. Adems se prefiere el empleo de una mezcla de diferentes
fuentes con biomasa activa que emplear una sola. 24 El volumen inicial de
inculo nunca debe ser menor del 10 % del volumen efectivo del reactor.
Una vez cargado el reactor con el inculo y el sustrato de inters, se recomienda no iniciar el flujo continuo, sino mantener el sistema a no flujo hasta
que se aprecie produccin de gas.
Teniendo en cuenta que en la etapa inicial del proceso la poblacin de
microorganismos necesita adaptarse a las condiciones de operacin que se imponen, la carga inicial debe ser menor a la de la operacin normal que se pretende en el futuro. Puede iniciarse la puesta en marcha con un valor aproximado de
carga orgnica de 0,15 kg DBO.(kg SSV.d) . Esta carga deber ser
incrementada en la medida que se observe una adecuada autoregulacin del pH
as como la alcalinidad del medio, y exista control sobre la concentracin de los
cidos voltiles. Debe propiciarse adems un determinado nivel de mezcla dentro del reactor para facilitar el contacto contaminante-biomasa. El funcionamiento adecuado del digestor puede ser estimulado si junto al sustrato se adiciona
algn otro residual de fcil degradacin.
282

Digestin Anaerobia

Considerando que en un inicio el sistema buffer o tampn de pH que estos


sistemas poseen no est desarrollado, lo ms comn es que se aprecie una
tendencia a decrecer el pH debido a la formacin de los cidos grasos. El pH ha
de mantenerse en un valor cercano a 7 mediante la adicin de una base.

Notas bibliogrficas
1 HENZE0)XQGDPHQWDOVRIDQDHURELFGLJHVWLyQWater Sci. Tech., vol. 15,
no. 3, 1983.
2 ECKENFELDER, W.W.: Water Quality Engineering for Practicing Engineers,
CBI Pub. Co. Inc., EU.
3 YOUNG, J.C. AND P.L. MCCARTY:7KHDQDHURELFILOWHUIRUZDVWHWUHDWPHQW
Jour. Water Poll, Control Fed., vol. 41, no. 1, 1969.
4 VAN DER BERG, L. AND C.P. LENTZ:&RPSDULVRQEHWZHHQXSDQGGRZQIORZ
anaerobic fixed film reactors of varying surface-to-volume ratios for the
WUHDWPHQWRIEHDQEODQFKLQJZDVWH3URFth Purdue Ind. Waste Conf.,
1979.
5 LETTINGA, G. ET AL.8VHRI8SIORZ6OXGJH%ODQNHW 86% UHDFWRUFRQFHSW
IRU ELRORJLFDO ZDVWHZDWHU WUHDWPHQW Biotechnol. Bioeng, vol. 25,
no. 10, 1980.
6 YOUNG,-&)DFWRUVDIIHFWLQJWKHGHVLJQDQGSHUIRUPDQFHRIXSIORZDQDH
URELFILOWHUVWater Sci. Tech., vol. 24, no. 8, 1991.
7 JENNET, J.C. AND H.D. DENNIS$QDHURELFILOWHUWUHDWPHQWRISKDUPDFHXWLFDO
ZDVWHJour. Water Poll, Control fed., vol. 47, no. 1, 1975.
8 COULTER, J.E. AND M.B. ETTINGER $QDHURELF FRQWDFW SURFHVV IRU VHZDJH
GLVSRVDOSewage and Ind. Wastes, vol. 29, no. 4, 1957.
9 GUIOT, R.S., Y. ARCAND AND C. CHAVARIE$GYDQWDJHVRIIOXLGL]DWLRQRQJUDQX
OHVL]HDQGDFWLYLW\GHYHORSPHQWLQXSIORZDQDHURELFEHGUHDFWRUVWater
Sci. Tech., vol. 26, no. 3-4, 1992.
10 LETTINGA, G. AND P.L. HULSHOFF8$6%SURFHVVGHVLJQIRUYDULRXVW\SHVRI
ZDVWHZDWHUVWater Sci. Tech., vol. 24, no. 8, 1991.
11 IZA, J.:)OXLGL]HGEHGUHDFWRUVIRUDQDHURELFZDVWHZDWHUWUHDWPHQWWater
Sci. Tech., vol. 24, no. 8, 1991.
12 METCLAF AND EDDY: Ingeniera de Aguas Residuales. Tratamiento, Vertido
y Reutilizacin, 3ra. Edic., vol. 1, p. 487, Mc Graw Hill, 1996.
13 HENZE, M. AND P. HARREMOES$QDHURELFWUHDWPHQWRIZDVWHZDWHULQIL[HG
ILOP UHDFWRUV$ /LWHUDWXUH 5HYLHZ Water Sci. Tech., vol. 15, no. 8/9.
1983.
14 SPEECE, R.E.:7R[LFLW\LQDQDHURELFGLJHVWLRQ3URFth Int., Symposium on
anaerobic digestion, Guanghou, China, 1985.
283

Menndez Gutirrez, C. y J. Prez Olmo

15 HARRIS, W. L. AND R. DAGUE&RPSDUDWLYHSHUIRUPDQFHRIDQDHURELFILOWHUV


DWPHVRSKLOLFDQGWKHUPRSKLOLFWHPSHUDWXUHVWater Environment Res.,
vol. 65, no. 6, 1993.
16 JEWELL : $QDHURELF 6HZDJH 7UHDWPHQW (QY Sci. Tech., vol. 21,
no. 1, 1987.
17 MC CARTY, P.L.:$QDHURELFZDVWHWUHDWPHQWIXQGDPHQWDOV3DUW7R[LF
Materials and their Control, Pub. Works, vol. 91, no. 11, 1964.
18 KROEKER, E.J. ET AL$QDHURELF WUHDWPHQWSURFHVV VWDELOLW\-RXU Water
Poll. Control, vol. 51, no. 4, 1979.
19 GUPTA, A.R., R. FLORA, M. GUPTA, G. SAYLES AND M. SUIDAN0HWKDQRJHQH
sis and sulphate reduction in chemostats I. Kinetic studies and experiPHQWVWater Res., vol. 28, no. 4, 1994.
20 MCCARTY, P.L. ET AL.:,QGLYLGXDOYRODWLOHDFLGLQDQDHURELFWUHDWPHQW-RXU
Water Poll. Control Fed., vol. 35, no. 12, 1963.
21 JENKINS, S.R., J.M. MORGAN AND X. ZHANG0HDVXULQJWKHXVDEOHFDUERQDWH
DONDOLQLW\RIRSHUDWLQJDQDHURELFGLJHVWHUV5HV-RXUWater Poll. Control Fed., vol. 63, no. 1, 1991.
22 ZEPENG, C. ET AL.:7KHDSSOLFDWLRQRIWZRSKDVHDQDHURELFGLJHVWLRQRQWKH
disposal of organic wastewater Proc. 4th Int. Symposium on anaerobic
GLJHVWLRQ*XDQJKRX&KLQD
23 ECKENFELDER, W.W. AND D.L. FORD: Water Pollution Control, Jenkins Book
Pub. Co. E.U., 1970.
24 WEILAND, P. AND A. ROZZI7KHVWDUWXSRSHUDWLRQDQGPRQLWRULQJRIKLJK
UDWHDQDHURELFWUHDWPHQWV\VWHPVWater Sci. Tech., vol. 24, no. 8, 1991.

284

Manejo de Lodos

CAPTULO 9

MANEJO DE LODOS

9.1. INTRODUCCIN
Un aspecto importante en el tratamiento de los residuales industriales lquidos (RIL) lo constituye la manipulacin de los lodos o fangos que se obtienen,
sean estos de naturaleza orgnica o inorgnica.
Los lodos producidos en los tratamientos biolgicos de aguas residuales pueden provenir de los sedimentadores primarios y secundarios as como de los
tratamientos terciarios. Estos lodos estn constituidos fundamentalmente de
materia orgnica, con una fraccin voltil entre 60 y 80 %.
En los tratamientos fsico-qumicos como puede ser la coagulacin, los lodos
que se producen son eminentemente de naturaleza inorgnica.
Uno u otro lodo posee en su composicin agua en un alto porcentaje. Esto
dificulta tanto su manipulacin como su disposicin.
La magnitud del problema se evidencia en un clculo sencillo al considerar
los residuales de una industria cuyos lodos corresponden a una poblacin equivalente a 500000 personas. En ese caso se producen aproximadamente 0,09 kg
de lodo por habitante. Si se asume un contenido de humedad de 97 %, esto
significa que se manejan diariamente 3000 m3 de lodo.
La naturaleza de los lodos, orgnica o inorgnica, define el tratamiento al
que deben ser sometidos antes de su disposicin final, pero en uno y otro caso
deben ser transportados y manipulados antes y despus del tratamiento que se
emplee para los mismos.

9.2. CONTENIDO DE HUMEDAD Y VOLUMEN


DE LOS LODOS
La gravedad especfica de los slidos en los lodos puede ser determinada
mediante la expresin:

Ws
Ss

Wf Wv

Sf
Sv
285

(9.1)

Menndez Gutirrez, C. y J. Prez Olmo

donde:
Ws: masa de slido (slido seco), kg.
Ss: gravedad especfica de los slidos secos.
Wf: masa de slidos fijos (fraccin no voltil de los slidos), kg.
Wv: masa de slidos voltiles, kg.
Sf: gravedad especfica de los slidos fijos.
Sv: gravedad especfica de los slidos voltiles.
Teniendo en cuenta que la fraccin filtrable de los lodos es relativamente
pequea comparada con la no filtrable, los slidos pueden ser tomados como el
residuo total despus de secada la muestra a 105 C.
Si en determinada circunstancia la fraccin filtrable es de tal magnitud que
no puede ser despreciada, debe trabajarse con los slidos suspendidos o no
filtrables.
La gravedad especfica de la materia orgnica puede ser tomada como
1,2 a 1,4, mientras que la de la materia inorgnica vara entre 1,5 y 2,5.
Una vez que se conoce la gravedad especfica de los slidos secos, puede
calcularse la de los lodos mediante la relacin:

Ws  Ww
Ws
Ww 
Ss

(9.2)

donde:
S: gravedad especfica de los lodos.
Ws: masa de slido seco.
Ww: masa de agua.
Ss: gravedad especfica de los slidos secos.
No es difcil demostrar que para los lodos orgnicos cuyo contenido de humedad sea del 90 % o mayor, su gravedad especfica puede ser tomada como
1,0 sin cometer mucho error.
El volumen de lodo, calculado a partir de su contenido de slido y % de
humedad es,

Ws
100  P

W S
100

V: volumen de lodo, m3.


P: contenido de humedad, %.
U w: densidad del agua, kgm-3.
286

(9.3)

Manejo de Lodos

A los efectos prcticos puede asumirse sin mucho error, que la densidad del
agua es 1000 kgm-3 en el intervalo de temperatura entre 20 y 40 C.
La ecuacin 9.3 puede ser utilizada tambin en aquellos casos en los que se
desee calcular el flujo de lodo, Qs (m3d-1), si el trmino Ws est dado como flujo
msico (kgd-1).

9.3. LODOS PRIMARIOS Y SECUNDARIOS. CRITERIOS


DE ESTIMACIN
Siempre que no se disponga de toda la informacin necesaria para el clculo
preciso de los lodos en un sistema dado, se hace necesario acudir a mtodos de
clculo aproximados.
La produccin de slidos en los sedimentadores primarios y secundarios
puede estimarse de manera aproximada utilizando las siguientes ecuaciones,
Ws = Wsp + Wss
(9.4)
-1
Ws: produccin total de slido seco, kgd .
Wsp: produccin de slido seco en el sedimentador primario, kgd-1.
Wss: produccin de slido seco en el sedimentador secundario, kgd-1.
Por otro lado,
(9.5)
Wsp = f SS Q 10-3
-1
SS: slidos suspendidos en el agua sin sedimentar, mg L .
Q: flujo de agua residual que entra al sedimentador primario, m3 d-1.
f: fraccin de los slidos suspendidos que es removida en el sedimentador
primario y
(9.6)
:VV I'%24 10-3
DBO: concentracin en el agua residual que entra a la unidad de tratamiento
secundario, mg L-1.
I: fraccin de la DBO aplicada que aparece como exceso de lodo en la
unidad de tratamiento secundario.
El clculo estimado se basa, fundamentalmente, en la experiencia de trabajo
que se tenga con cada residual en particular, estando la mayor variacin en la
IUDFFLRQHVI\ISDUDORVUHVLGXDOHVLQGXVWULDOHV
Para residuales domsticos: f = 0,4;
I 
Para residuales industriales f depende de los slidos sedimentables.
Para residuales domsticos:
Filtros percoladores

de baja velocidad
de alta velocidad
287

I 
I 

Menndez Gutirrez, C. y J. Prez Olmo

Lodo activado

aeracin extendida
convencional

Para residuales industriales:


Lodo activado convencional papel
azucarero
cervecera
tenera

I
I
Ia
Ia
Ia
Ia

Ejemplo 9.1
Una industria del papel arroja diariamente 3000 m3 de un agua residual
con una DBO de 300 mg L-1 y una concentracin de slidos suspendidos de
270 mg L-1 . Para el tratamiento de esos residuales est concebido una sedimentacin primaria donde se elimine 65 % de los slidos suspendidos y 45 %
de la DBO.
El % de agua de los lodos primarios ser de 94 %.
La concentracin esperada de los lodos secundarios es de 15000 mg L-1.
Calcule el estimado de lodo a extraer en aquel caso en que se decida unir y
espesar los lodos primarios y secundarios de conjunto hasta obtener una concentracin de slidos de 9 %. Estime adems el flujo de lodo a extraer.
Wsp = f SS Q 10-3
= 0,65 270 3000 10-3
= 526,5 kgd-1

Qsp

Qsp

Wsp
100  P

W S
100

526,5
8,77 m3 d-1
0,06 1000 1

:VV I'%24 10-3


= 0,55 300 0,35 3000 10-3 VHDVXPLyI 
= 173,25 kg d-1

Q ss

Qss

Wss
100  P

W S
100

173,25
11,55 m3d-1
0,015 1000 1
288

Manejo de Lodos

Volumen de la mezcla de lodos primarios y secundarios,


Qs = Qsp + Qss
= 8,77 + 11,55
= 20,32 m3 d-1
Masa de slidos en la mezcla,
Ws = Wsp + Wss
= 526,5 + 173,25
= 699,75 kg d-1
% en peso de slidos en la mezcla antes del espesamiento:
%peso

699 ,75 100


x
20 ,32 1000

3,4 %

Flujo de lodo espesado a extraer

Qs

699,75
7 ,77 m3d-1
0,09 1000 1

Observe que un aumento en la concentracin de slidos por espesamiento


del 3,4 % al 9 %, da lugar a una disminucin en el volumen de lodos a manipular
de 20,32 a 2,77 m3 d-1.
La variacin de la concentracin de slidos que puede obtenerse en una
secuencia de tratamiento se muestra a continuacin.
Tabla 9.1. Secuencia de tratamiento

Conc.
Slidos (%)

sedimentacin espesamiento deshidratacin


0,5 6,0
20 12,0
18,0 50,0

Los lodos primarios y los inorgnicos, generalmente tienen una mayor concentracin en cada una de las etapas de la secuencia de tratamiento que los
obtenidos de la coagulacin mediante aluminio o la de aquellos que proceden de
lodos activados.
Los procesos que normalmente se utilizan en la secuencia de deshidratacin
dependen de:
Naturaleza y caractersticas del lodo.
Mtodo que se aplicar en la disposicin final.

289

Menndez Gutirrez, C. y J. Prez Olmo

El espesamiento por gravedad y por flotacin, la filtracin al vaco o por


presin, el tratamiento mediante calor, y el secado en lechos al aire libre, son
algunos de los tratamientos que hoy da se utilizan para disminuir el contenido de
agua de los lodos.
En los pases en desarrollo los procesos ms comnmente empleados son el
espesamiento por gravedad y los lechos de secado debido a su relativo bajo
costo de inversin, operacin y mantenimiento. No obstante en ocasiones los
volmenes de lodos son tales que se hace necesario acudir a los procesos ms
intensivos y tecnolgicamente ms complejos.

9.4. ESPESAMIENTO POR GRAVEDAD


Los principios que rigen este proceso as como los elementos fundamentales
que facilitan el diseo y operacin de los espesadores pueden ser tratados con
ms detalle. Sin embargo, aqu se dan algunos elementos de carcter prctico
para el trabajo ms cotidiano.
Debe tenerse presente que los espesadores se disean sobre la base del
rea unitaria m2kg-1d-1, mediante la relacin del rea superficial con los slidos
que entran y salen del espesador.
La carga msica de lodo, kgm-2d-1 recomendada para el trabajo de los
espesadores vara con la facilidad que posea el lodo de perder el agua, y puede
variar entre 20 kgm-2d-1 para lodos digeridos procedentes de un lodo activado,
hasta 150 kgm-2d-1 para un lodo primario.
El flujo por unidad de rea que normalmente se recomienda para los
espesadores vara entre 15 y 40 m3m-2d-1. Cargas superficiales menores pueden
tener como consecuencia malos olores al producirse condiciones spticas. En
tales casos es recomendable el empleo de agua de dilucin para aumentar el
flujo por unidad de rea.
En los espesadores continuos es usual dejar una capa de lodo en el fondo del
mismo para propiciar el aumento de su concentracin. Normalmente el tiempo
de retencin de los slidos oscila entre 0,5 y 2 das. Este tiempo de retencin,
llamado por algunos relacin de volumen de fango (RVF), se calcula dividiendo
el volumen ocupado por el manto de lodo en el espesador, entre el flujo diario de
lodo extrado.
Ejemplo 9.2
El lodo primario que se obtiene en una planta de tratamiento de aguas
residuales es 16 m3 d-1 con una concentracin de slidos de 45000 mg L-1.
Calcule el rea requerida del espesador, asumiendo una carga msica de
lodo de 50 kgm-2d-1.
290

Manejo de Lodos

Considere adems que en el sobrenadante sale 5 % de los slidos que entran al espesador y que los lodos de fondo tienen una concentracin de 8 %.
Determine el agua de dilucin que se necesita adicionar para obtener una
carga superficial de 15 m3 m-2 d-1.
Carga de lodo al espesador = 16 x 45000 x 10-3 = 720 kg d-1
rea de tanque requerida = A

720
14,4 m2
50

A
Dimetro = D

14 ,4 4
D

0 ,5

4,28 m

Volumen de lodo espesado = V (ec. 9.3)

720 0,95 8,55


8,55
0,08 1000 1

m3 d-1
Flujo por unidad de rea = q
q

16
1,11
14,4

m3 m-2 d-1
El flujo de agua suplementario que se necesita para lograr una carga superficial de 15 m3m-2d-1 ser,
  P3d-1
Si el manto de lodo espesado tiene una profundidad de 1,25 m, estime el
tiempo de retencin de los lodos en el espesador,

1,25 14,4
8,55

2 ,1 d

9.5. LECHOS DE SECADO


El mtodo ms comn de secado de lodos espesados es el de los lechos de
secado, teniendo en cuenta su economa y que no requiere de personal especializado. Se utiliza fundamentalmente en plantas pequeas de tratamiento de albaales y para algunos residuales industriales.
291

Menndez Gutirrez, C. y J. Prez Olmo

El secado en los lechos se produce a travs de los procesos de percolacin


y evaporacin. Por esta razn su diseo y empleo est influido por las condiciones climticas prevalecientes.
La fraccin de agua que se elimina por percolacin vara de 0,2 a 0,5 en
dependencia el contenido inicial de slidos y de sus caractersticas, as como de
la radiacin solar que incide sobre el mismo.
En el tema no se insistir en el diseo emprico de los lechos de secado. No
obstante, a continuacin se resumen algunos criterios que mejor se ajustan al
diseo de estos dispositivos en pases que reciben una buena radiacin solar
durante la mayor parte del ao.
Tabla 9.2. Parmetros empricos para el diseo de lechos de secado

Tipo de lodo digerido


Primario
Primario y filtro percolador
Primario y lodo activado
Lodo de precipitacin qumica

rea m2 (persona)-1
0,040
0,044
0,112
0,076

Cuando los lodos a secar proceden de un tratamiento de residuales industriales es recomendable, siempre que se pueda, realizar pruebas de secado de
los mismos antes de proceder a disear con criterios que quizs no se ajustan a
un caso particular.
Swanwick1,2 ha desarrollado una metodologa para el estudio de la deshidratacin de los lodos en lechos de secado. Esta metodologa se describe a continuacin:
1. Utilice un tubo de vidrio de 2,50 cm de dimetro con arena en el fondo.
2. Introduzca en el tubo el lodo a secar hasta tener una altura de aproximadamente de 30 a 45 cm.
3. Permita el drenaje del agua durante un periodo entre 12 y 48 horas, dependiendo del contenido de humedad.
4. Extraiga la torta drenada y determine su contenido de humedad.
5. Exponga la torta a la evaporacin natural en contacto directo con el aire.
Controle peridicamente el contenido de humedad hasta que se alcance
el valor deseado.
6. Determine la humedad. La diferencia entre el valor obtenido aqu y en el
paso 4 es el agua a evaporar.
7. Infrmese de la evaporacin y el rgimen de lluvia anual de la zona a
partir de datos meteorolgicos.
8. Se acepta que la evaporacin que ocurre en el lodo hmedo es 75 % de la
que ocurre en el agua as como que 43 % del agua de lluvia es drenada y
75 % restante se evapora.
292

Manejo de Lodos

9. Calcule, mes a mes, la evaporacin acumulativa multiplicada por el


factor 0,75. Grafique este resultad en funcin del tiempo. Haga lo
mismo con la lluvia cada, pero multiplicndola por el factor 0,57.
10.Estime, para cada mes, teniendo en cuenta los resultados en 9 y 6, el
tiempo requerido para el secado del lodo.
11. Tabule, para cada mes, el rea total requerida.
12.El rea de diseo ser la mxima hallada en el paso 11.

9.6. VOLUMEN DE AGUA PRODUCIDA


Como consecuencia del proceso de deshidratacin de los lodos, por cualquier mtodo que se utilice, se produce un volumen dado de agua que es preciso
conocer. Esta agua no siempre est libre de slidos. Si se trata de un espesador
por gravedad operando en ptimas condiciones, el lquido claro obtenido puede
contener entre 5 y 10 % de los slidos iniciales del lodo. Con otros procedimientos el contenido de slidos puede ser mayor o menor.
El volumen de agua producida puede calcularse como sigue:
Se sabe que,

Cw
Ww

Cs
Ws

(9.7)

Cs: contenido de slidos del lodo, %.


Cw: contenido de agua en el lodo, %.
Ws: masa de slido, kg.
Ww: masa de agua, kg.
Entonces,

100  Cs
Ww

Cs
Ws

(9.8)

De aqu,

100
Ww
1 Ws
Cs
Si se acepta que la densidad del agua es 1000 kg m-3,
Vw = Ww 10-3
donde:
Vw: volumen de agua, m3.

293

(9.9)

(9.10)

Menndez Gutirrez, C. y J. Prez Olmo

Por tanto
100

Vw
 1 Ws 10 3
Cs

(9.11)

El agua que est asociada al cambio de concentracin de un lodo desde (Cs)1


a (Cs) 2 es:

Vw

1 1
1


(Cs)2
10 (Cs)1

Ws

(9.12)

Ejemplo 9.3
100 m3 d-1 de un lodo con un contenido de slidos del 5 % es deshidratado
por diversos tratamientos hasta alcanzar 20 %. Determine el volumen de lquido
claro producido.
(Cs)1 = 5 %
(Cs)2 = 20 %
Ws = 100 50000 10-3 = 5000 kg d-1
Aplicando la ecuacin 9.12,

Vw

1 1
1
 5000
10 5 20
Vw = 75 m3 d-1

Notas bibliogrficas
1 SWANWICK, J.D.: Advances in Water Pollution Research, vol. II, Pergamon
Press, N.Y., 1963.
2 _____________ : Water Poll. Research, +HU 0DMHVW\V 6WDFLRQDU\ 2IILFH
London, 1966.

294

ANEXOS

ANEXO 1

VOLMENES MNIMOS DE MUESTRAS


Y CRITERIOS PARA SU CONSERVACIN

Parmetro

Aceites y
grasas
Acidez
Alcalinidad
DBO
DQO
Carbono
orgnico, total
Nitrato
Nitrito
Amoniaco
N-org
Fosfato
Oxgeno
disuelto
pH

Volumen
Tiempo
Conservacin
mnimo de
mximo de
muestra, mL
conservacin
28 d
1000
aadir H2SO4 hasta pH
2 y refrigerar
refrigerar
24 h
100
refrigerar
24 h
200
refrigerar
6h
1000
7d
100
aadir H2SO4 hasta pH
2 y refrigerar
refrigerar y aadir HCl
7d
100
hasta pH 2
48 h
100
refrigerar
12 h
100
refrigerar
7d
500
Aadir H2SO4 hasta pH
2 y refrigerar
7d
500
aadir H2SO4 hasta pH
2 y refrigerar
48 h
100
refrigerar
Analizar inmediatamente
300
Analizar inmediatamente

297

ANEXO 2

DISTRIBUCIN NORMAL ACUMULATIVA

N
0,00
0,05
0,10
0,15
0,20
0,25
0,30
0,35
0,40
0,45
0,50

P(X)
0,5000
0,5160
0,5398
0,5596
0,5793
0,5987
0,6179
0,6368
0,6554
0,6736
0,6915

N
0,55
0,60
0,65
0,70
0,75
0,80
0,85
0,90
0,95
1,00

P(X)
0,7088
0,7257
0,7422
0,7580
0,7734
0,7881
0,8023
0,8159
0,8289
0,8413

298

N
1,05
1,10
1,15
1,20
1,25
1,30
1,35
1,40
1,45
1,50

P(X)
0,8531
0,8643
0,8749
0,8849
0,8944
0,9032
0,9115
0,9192
0,9265
0,9332

S-ar putea să vă placă și